FMGE (MCI Screening test) preparation material by Gankidi Raghavender Reddy

232

Click here to load reader

description

MEDICAL FMGE ENTRANCE TEST ,INDIACOLLECTED BY Gankidi Raghavender Reddy (MBBS)student,ZSMU,Ukraine For more study materials contact : [email protected] May be this material is used for FMGE(MCI Screening test) preparation also just practice it…….!!!!!!!!!!!!!!!!!Best of LuckQuestion 1:Which of the following stimulus does not induce visceral pain1. Distension 2. Pressure 3. Cauterisation 4. Cutting 5. Correct Answer: 4.Question 2: Lymphatics of ovary drains into 1. Paraaortic LN 2. Internal iliac LN 3. External iliac LN 4. Obtura

Transcript of FMGE (MCI Screening test) preparation material by Gankidi Raghavender Reddy

Page 1: FMGE (MCI Screening test)  preparation material by Gankidi Raghavender Reddy

MEDICAL PG ENTRANCE TEST ,INDIACOLLECTED BY Gankidi Raghavender Reddy (MBBS) Student

For more study materials contact : [email protected]

May be this material is used for FMGE preparation also just practice it…….!!!!!!!!!!!!!!!!!

Question 1: Which of the following stimulus does not induce visceral pain

1. Distension

2. Pressure

3. Cauterisation

4. Cutting

5.

Correct Answer: 4.

Question 2: Lymphatics of ovary drains into

1. Paraaortic LN

2. Internal iliac LN

3. External iliac LN

4. Obturator LN

5.

Correct Answer: 1.

Page 2: FMGE (MCI Screening test)  preparation material by Gankidi Raghavender Reddy

Question 3: All the following are derivatives of the neural crest except

1. Melanocyte

2. Adrenal medulla

3. Sympathetic ganglia

4. Cauda equina

5.

Correct Answer: 4.

Question 4: The term neuropraxia means

1. Complete division of nerves

2. Functional disruption

3. Division of nerve fibers with intact nerve sheath

4. Anatomical disruption of nerve sheath

5.

Correct Answer: 2.

Question 5: All the following are true about dorsal root ganglia except

1. Occurs in the cranial nerves

2. Derivative of neural crest

3. Contains multipolar cells

4. Contains lipofuscin granules

5.

Correct Answer: 3.

Question 6: Urethra of female

1. Has only connective tissue in its upper third

2. Has only smooth muscle in its wall

3. Is shorter than in male

4. Is longer than in male

5.

Correct Answer: 3.

Question 7: Left renal vein crosses the Aorta:

1. Anteriorly, above the superior mesenteric artery

2. Anteriorly, below the superior mesenteric artery

3. Posteriorly, at the level of superior mesenteric artery

4. Anteriorly, below the inferior mesenteric artery

5.

Correct Answer: 2.

Question 8: Superficial fatty fascia between umbilicus & pubis is

1. Camper's fascia

2. Scarpa's fascia

3. Colle's fascia

Page 3: FMGE (MCI Screening test)  preparation material by Gankidi Raghavender Reddy

4. Cilli's fascia

5.

Correct Answer: 1.

Question 9: What disorder is caused by having three copies of chromosome 21?

1. Phenylketonuria

2. Down syndrome

3. Sickle-cell anemia

4. Hemophilia

5.

Correct Answer: 2.

Question 10: The first costochondral joint is a

1. Fibrous joint

2. Synovial joint

3. Syndesmosis

4. Synarthrosis

5.

Correct Answer: 4.

Question 11: Clavicular part of deltoid is associated with

1. Lateral rotation

2. Medial rotation

3. Abduction

4. Adduction

5.

Correct Answer: 2.

Question 12: The coracobrachialis assist in which movement of arm

1. Flexion

2. Extension

3. Rotation

4. Supination

5.

Correct Answer: 1.

Question 13: Transpyloric plane passes through

1. T12- L1

2. L5- S1

3. T10

4. L1-L2

5.

Correct Answer: 4.

Question 14: The transfer of body weight in vertebral column is through

Page 4: FMGE (MCI Screening test)  preparation material by Gankidi Raghavender Reddy

1. Laminae

2. Pedicle

3. Vertebral body

4. Neural arch

5.

Correct Answer: 3.

Question 15: Pancreatico splenic lymph nodes receive lymphatics from the part of the stomach which is supplied by

1. Left gastric artery

2. Short gastric arteries and left gastro epiploic artery

3. Right gastro epiploic artery

4. Right gastric artery

5.

Correct Answer: 2.

Question 16: Nerve supply of pyramidalis muscle is

1. Ilioinguinal nerve

2. Subcostal nerve

3. Genitofemoral nerve

4. None

5.

Correct Answer: 2.

Question 17: Posterior boundary of foramen of Winslow is formed by

1. Aorta

2. Inferior vena cava

3. Portal vein

4. Liver

5.

Correct Answer: 2.

Question 18: Perforators are not present at the

1. Ankle

2. Distal calf

3. Mid thigh

4. Below the inguinal ligament

5.

Correct Answer: 4.

Question 19: Bunion is

1. Fluid filled enlarged bursa over the bony prominences

2. Tumor of sesamoid bone

3. Infected Bursa

4. Blood filled bursa

5.

Page 5: FMGE (MCI Screening test)  preparation material by Gankidi Raghavender Reddy

Correct Answer: 1.

Question 20: Regarding the vessels in the umbilical cord which of the following are true

1. There are two arteries and two veins

2. There are two arteries and the right vein

3. There are two arteries and left vein

4. There is only one artery and one vein

5.

Correct Answer: 3.

Question 21: Urinary bladder and urethra are derived from the

1. primitive urogenital sinus (a ventral part of the cloaca) and adjacent splanchnic mesenchyme.

2. Metanephros

3. Paramesonephric duct

4. Mesonephric duct

5.

Correct Answer: 1.

Question 22: Intrinsic muscles of tongue develops from ?

1. cervical somite

2. mesoderm of pharyngeal pouch

3. Occipital somite

4. None of these

5.

Correct Answer: 3.

Question 23: The attachment of the mesentery of the small gut is

1. Left transverse process of L2 to Right sacroiliac joint

2. Right transverse process of L2 to Right sacroiliac joint

3. Left transverse process of T1 to Right sacroiliac joint

4. Right transverse process of T1 to Left sacroiliac joint

5.

Correct Answer: 1.

Question 24: Ureteric constriction is seen at all the following positions except

1. Ureteropelvic junction

2. Ureterovesical junction

3. Crossing of iliac artery

4. Ischial spine

5.

Correct Answer: 4.

Question 25: A patient of external piles has pain, which of the following nerves carry this pain sensation

1. Hypogastric nerves

Page 6: FMGE (MCI Screening test)  preparation material by Gankidi Raghavender Reddy

2. Parasympathetic plexus

3. Sympathetic nerve

4. Pudendal nerve

5.

Correct Answer: 4.

Question 26: All of the following development events are dependent on the production of maternal or fetal glucocorticoid,

except

1. Functional thyroid

2. Functional hypothalamopituitary axis

3. Production of surfactant by type II alveolar cells

4. Induction of thymic involution

5.

Correct Answer: 1.

Question 27: All of the following ocular muscle are supplied by occulomotor nerve except

1. Inferior oblique

2. Medial Rectus

3. Laevator palpabrae superioris

4. Lateral rectus

5.

Correct Answer: 4.

Question 28: Injury to radial nerve in lower part of spiral groove

1. Spares nerve supply to extensor carpiradialis longus

2. Results in paralysis of anconeus muscle

3. Leves extension at elbow joint intact

4. Weakens pronation movement

5.

Correct Answer: 3.

Question 29: The Ureter

1. Crosses superior to the uterine artery in the Broad ligament

2. Derives its sympathetic nerve supply from the 2nd and 3rd lumbar nerve roots

3. Is widest at the pelvic ureteric junction

4. Derives its blood supply entirely from the renal and superior vesical arteries

5.

Correct Answer: 2.

Question 30: The principal factor causing the rupture of the graffian follicle is

1. Increase in the intra follicular pressure

2. Necrobiosis of the overlying tissue

3. Both

4. None

5.

Page 7: FMGE (MCI Screening test)  preparation material by Gankidi Raghavender Reddy

Correct Answer: 2.

Question 31: All the following are true regarding blood supply to the kidney except

1. Arcuate artery is an end artery

2. It is a site for portal systemic anastomosis

3. The renal artery divides into five segmental arteries before entering the hilum

4. Left renal artery is a branch of internal iliac artery

5.

Correct Answer: 2.

Question 32: Which muscle of the anterior compartment also flexes the hip ?

Clinical correlations in the lower limb

1. Articularis genu

2. Vastus lateralis

3. Tensor fascia lata

4. Sartorius

5. Adductor longus

Correct Answer: 4.

Question 33: The skin overlying the region where a “cut-down” is made to access the Great saphenous vein is supplied by

1. Femoral Nerve

2. Sural Nerve

3. Tibial Nerve

4. Superficial Peroneal Nerve

5.

Correct Answer: 1.

Question 34: Which does not occur in ulnar nerve injury in the arm?

1. claw hand

2. atrophy of hypothenar eminence

3. loss of sensation of medial one third of hand

4. adduction of thumb

5.

Correct Answer: 4.

Question 35: A 48-year-old white female secretary presents with progressive difficulty typing over the past month. She

also notes that her hands begin to feel numb and weak after typing for long periods of time. Upon testing, which of the

following deficits would be predicted?

A. B. C. D. E. Loss of sensation over the lateral half of the palm F. Loss of sensation over the medial half of the dorsum of

the hand G.

1. Difficulty in abducting the fifth finger

2. Difficulty in adducting the thumb

3. Difficulty in flexing digits two and three at the metacarpophalangeal joints

4. Loss of sensation over the lateral half of the dorsum of the hand

5. Loss of sensation over the medial half of the palm

Page 8: FMGE (MCI Screening test)  preparation material by Gankidi Raghavender Reddy

Correct Answer: 3.

Question 36: All of the following are Somatic Efferent except

1. Facial nerve

2. Occulomotor nerve

3. Trigeminal nerve

4. Abducens nerve

5.

Correct Answer: 1.

Question 37: Caudate lobe of liver is situated between

1. Gall bladder & groove for ligamentum teres

2. IVC and ligamentum venosum

3. Posterior part of left lobe

4. Anterior superior surface of liver

5.

Correct Answer: 2.

Question 38: Afferent component of corneal reflex is mediated by

1. Vagus nerve

2. Facial nerve

3. Trigeminal nerve

4. Glossopharyngeal nerve

5.

Correct Answer: 3.

Question 39: Safety muscle of the tongue is

1. Hyoglossus

2. Genioglossus

3. Palatoglossus

4. Styloglossus

5.

Correct Answer: 2.

Question 40: Fecal fistula at umbilicus is due to persistence of

1. Vitellointestinal duct

2. Patent urachus

3. Remnant of urachus

4. Ligamentum arteriosum

5.

Correct Answer: 1.

Question 41: The following is true regarding spleen

1. Notch is on inferior border

Page 9: FMGE (MCI Screening test)  preparation material by Gankidi Raghavender Reddy

2. Long axis parallel to 12th Rib

3. Developed form ventral mesogastrium

4. Nerve supply from coeliac plexus

5.

Correct Answer: 4.

Question 42: All muscles of Pharynx are supplied by cranial accessory nerve except

1. Thyropharyngeus

2. Inferior constrictor

3. Stylopharyngeus

4. Superior constrictor

5.

Correct Answer: 3.

Question 43: The commonest variation in the arteries arising from the arch of aorta is

1. Absence of brachiocephalic trunk

2. Left vertebral artery arising from the arch

3. Left common carotid artery arising from brachiocephalic trunk

4. Presence of retroesophageal subelavian artery

5.

Correct Answer: 3.

Question 44: Blood vessel related to paraduodenal fossa is

1. Gonadal veins

2. Superior mesenteric artery

3. Portal vein

4. Inferior mesenteric vein

5.

Correct Answer: 4.

Question 45: A patient develops a swollen and tender lymph node in his popliteal fossa. An infected skin lesion in which of

the following sites would most likely induce lymphadenopathy in this region?

1. Lateral side of the dorsum of the foot

2. Lateral side of the thigh

3. Medial side of the leg below the knee

4. Medial side of the sole of the foot

5. Medial side of the thigh

Correct Answer: 1.

Question 46: Which of the following is true regarding gastrulation

1. Leads to formation of the three germ layers

2. Occurs at the caudal end prior to the cranial end

3. Inner cell mass gives rise to the yolk sac

4. Usually occurs at 4 weeks

5.

Page 10: FMGE (MCI Screening test)  preparation material by Gankidi Raghavender Reddy

Correct Answer: 1.

Question 47: Which implant is used in Nasopharyngeal carcinoma

1. Caesium

2. I-131

3. Gold

4. Iridium

5.

Correct Answer: 4.

Question 48: The superficial external pudendal artery is a branch of

1. Femoral artery

2. External iliac artery

3. Internal iliac artery

4. Aorta

5.

Correct Answer: 1.

Question 49: In dividing cells, spindle is formed by

1. Ubiquitin

2. Tubulin

3. Laminin

4. Keratin

5.

Correct Answer: 2.

Question 50: Buck’s fascia is related to

1. Ischiorectal fossa

2. Thigh

3. Neck

4. Penis

5.

Correct Answer: 4.

Question 51: Which of the following is not a boundary of the Koch's triangle

1. Tendon of Todaro

2. Limbus fossa ovalis

3. Coronay sinus

4. Tricuspid valve ring

5.

Correct Answer: 2.

Question 52: The least dilatable part of the urethra is

1. Prostatic

Page 11: FMGE (MCI Screening test)  preparation material by Gankidi Raghavender Reddy

2. Membranous

3. Spongy

4. All are equally dilatable

5.

Correct Answer: 2.

Question 53: Internal pudendal artery is a branch of

1. Anterior division of Internal iliac

2. Posterior division of Internal iliac

3. Obturator artery

4. Hypogastric

5.

Correct Answer: 1.

Question 54: Deep inguinal ring is a defect in the

1. External oblique

2. Internal oblique

3. Transversus abdominis

4. Transversus fascia

5.

Correct Answer: 4.

Question 55: All muscles of Tongue are supplied by hypoglossal nerve EXCEPT

1. Myoglossus

2. Palatoglossus

3. Genioglossus

4. Hyoglossus

5.

Correct Answer: 2.

Question 56: Fecal fistula at umbilicus is due to persistence of:

1. Vitellointestinal duct

2. Patent urachus

3. Remnant of urachus

4. Ligamentum arteriosum

5.

Correct Answer: 1.

Question 57: A 43 year old woman came with a large abscess in the middle of the right posterior triangle of the neck. The

physician incised and drained the abscess. Five days later the patient noticed that she could not extend her right hand

above her head to brush her hair. Which of the following are the signs and symptoms of additional harm?

1. Injury to suprascapular nerve

2. Damage to scalenus medius

3. Spread of infection to shoulder joint

4. Injury to spinal part of accessory nerve

5.

Page 12: FMGE (MCI Screening test)  preparation material by Gankidi Raghavender Reddy

Correct Answer: 4.

Question 58: The length of CBD is

1. 5 cm

2. 7.5 cm

3. 8.0 cm

4. 9 cm

5.

Correct Answer: 3.

Question 59: If a missile enters the body just above the pubic ramus through the anterior abdominal wall, it will most

likely pierce which of the following structures

1. Abdominal aorta

2. Left renal vein

3. Urinary bladder

4. Spinal cord

5.

Correct Answer: 3.

Question 60: After fracture of the penis (injury to the tunica albugenia) with intact Buck’s fascia, there occurs hematoma

at

1. The penis and scrotum

2. At the perineum in a butterfly shape

3. Penis, scrotum, perineum and lower part of anterior abdominal wall

4. Shaft of the penis only

5.

Correct Answer: 4.

Question 61: Which of the following muscles is NOT supplied by the median nerve or one of its branches:

1. Flexor pollicis brevis

2. Opponens pollicis

3. Flexor pollicis longus

4. 1st and 2nd lumbricals

5. Adductor pollicis

Correct Answer: 5.

Question 62: Taste sensation from the anterior 2/3rd of tongue is carried by

1. Hypoglossal nerve

2. Chorda tympani

3. Glossopharyngeal nerve

4. All of the above

5.

Correct Answer: 2.

Page 13: FMGE (MCI Screening test)  preparation material by Gankidi Raghavender Reddy

Question 63: A female come with complaints of chest pain. On examination she is found to have pericarditis with

pericardial effusion. The pain is mediated by

1. Deep cardiac plexus

2. Superficial cardiac plexus

3. Phrenic nerve

4. Subcostal nerve

5.

Correct Answer: 3.

Question 64: Elastic cartilage is found in

1. Auditory tube

2. Nasal septum

3. Articular cartilage

4. Costal cartilage

5.

Correct Answer: 1.

Question 65: All are blood supply of stomach except

1. Left gastric artery

2. Short gastric artery

3. Splenic artery proper

4. Left gastroepiploic artery

5.

Correct Answer: 3.

Question 66: Which is the Nucleus of Masseteric Reflex

1. Chief Sensory Nucleus

2. Nucleus of Spinal Tract of Trigeminal Nerve

3. Nucleus of Mesencephalic Tract of Spinal Nerve

4. Dorsal Vagal Nucleus

5.

Correct Answer: 3.

Question 67: Fascia extension of lacunar ligament along iliopectineal line is

1. Poupart ligament

2. Thomson’s ligament

3. Cooper’s ligament

4. Lacunar ligament

5.

Correct Answer: 3.

Question 68: Length of spermatozoa is

1. 50?m (micrometers)

2. 100?m

3. 120?m

Page 14: FMGE (MCI Screening test)  preparation material by Gankidi Raghavender Reddy

4. 500?m

5.

Correct Answer: 1.

Question 69: This organ is located in the Lower Digestive Tract. Its primary purpose is to break down the food in the Small

Intestine. It is divided into four parts which are the Superior, Descending, Inferior, and Ascending. What organ is it?

1. Duodenum.

2. Ileum

3. Jejenum

4. Cecum

5.

Correct Answer: 1.

Question 70: Lymphatic drainage of the anal canal is to

1. Inguinal nodes

2. External iliac nodes

3. Para - aortic nodes

4. Inferior mesenteric nodes

5.

Correct Answer: 1.

Question 71: Joint at Knee is

1. Synarthrosis

2. Symphysis

3. Amphiarthrosis

4. Diarthrosis

5.

Correct Answer: 3.

Question 72: The rectus sheath contains all of the following except

1. Pyramidalis muscle

2. Genitofemoral nerve

3. Inferior epigastric artery

4. Superior epigastric vessels

5.

Correct Answer: 2.

Question 73: The boundaries of morison's pouch are formed by all except

1. Inferior surface of liver

2. Anterior abdominal wall

3. Falciform ligament

4. Coronary ligament

5.

Correct Answer: 3.

Page 15: FMGE (MCI Screening test)  preparation material by Gankidi Raghavender Reddy

Question 74: Facial Nucleus / Facial Colliculus seen in

1. Midbrain

2. Pons

3. Interpeduncular fossa

4. Medulla

5.

Correct Answer: 2.

Question 75: Lateral boundary of femoral canal is formed by

1. Femoral artery

2. Femoral vien

3. Sartorius

4. Lacunar ligament

5.

Correct Answer: 3.

Question 76: The weight of the upper limb is transmitted to the axial skeleton by

1. Coracoclavicular ligament

2. Coracoacromial ligament

3. Costoclavicular ligament

4. Coracohumeral ligament

5.

Correct Answer: 1.

Question 77: Lower border of scapula corresponds to

1. D7

2. D9

3. D12

4. D5

5.

Correct Answer: 1.

Question 78: Spinal nerves are mixed because they contain

1. Sensory and motor nerves

2. Dorsal & ventral roots

3. Come through inter vertebral foramen

4. Constitute both craniosacral and the thoracolumbar out flow

5.

Correct Answer: 1.

Question 79: Main supply of small gut is by

1. Superior mesenteric Artery

2. Inferior mesenteric artery

3. Middle colic artery

Page 16: FMGE (MCI Screening test)  preparation material by Gankidi Raghavender Reddy

4. Celiac Axis

5.

Correct Answer: 1.

Question 80: In unexplained hypotension, which part of the vertebra should be examined

1. Upper cervical

2. Lumbar

3. Thoracic

4. Lower cervical

5.

Correct Answer: 3.

Question 81: 1. Which is the most superior structure at hilum of left lung?

1. pulmonary vein

2. pulmanary artery

3. bronchius

4. Bronchial artery

5.

Correct Answer: 2.

Question 82: All of the following are examples of traction epiphysis, except

1. Mastoid process

2. Tubercles of Humerus

3. Trochanters of femur

4. Condyles of tibia

5.

Correct Answer: 4.

Question 83: All the following are true about large intestine except

1. Large intestine secretes acidic mucus which helps in the formation of stools

2. It is a site of mucocutaneous junction

3. Its epithelium contains goblet cells in large numbers

4. Absorbs salt and water

5.

Correct Answer: 1.

Question 84: The human heart is divided into four parts. The Left and Right Ventricles and the Left and Right Atriums.

What is the purpose of the Right Ventricle?

1. To store blood temporarily

2. To manage the hearts rhythym

3. To pump blood to the lungs

4. To send blood to the Left Ventricle

5.

Correct Answer: 3.

Page 17: FMGE (MCI Screening test)  preparation material by Gankidi Raghavender Reddy

Question 85: Which activity will be difficult to perform for a patient with an anterior cruciate deficient joint

1. Walk downhill

2. Walk uphill

3. Sit Cross Leg

4. Getting up from sitting

5.

Correct Answer: 1.

Question 86: Structures related to lesser omentum are all except

1. Vagus nerve

2. Hepatic artery

3. Hepatic vein

4. Portal vein

5.

Correct Answer: 1.

Question 87: All are supports of uterus except

1. Uterosacral ligament

2. Round ligament of uterus

3. Mackenrodts ligament

4. Transverse cervical ligament

5.

Correct Answer: 2.

Question 88: Euchromatin is the region of DNA that is relatively

1. Uncondensed.

2. Condensed.

3. Overcondensed.

4. Partially condensed

5.

Correct Answer: 1.

Question 89: Serratus anterior is supplied by

1. Thoracodorsal nerve

2. Nerve to subclavius

3. Long thoracic nerve

4. Axillary nerve

5.

Correct Answer: 3.

Question 90: The sensory supply of the palate is through all the following except

1. Facial nerve

2. Hypoglossal nerve

3. Glossopharyngeal nerve

Page 18: FMGE (MCI Screening test)  preparation material by Gankidi Raghavender Reddy

4. Maxillary division of trigeminal

5.

Correct Answer: 2.

Question 91: All are true about urinary bladder in the male except

1. Posterior to the pubic symphysis

2. Anterior to the ampulla of the vas deferens

3. Superior to the prostate gland

4. Superior to the seminal vesicles

5.

Correct Answer: 4.

Question 92: The kidney has_____ segments

1. 11

2. 9

3. 7

4. 5

5.

Correct Answer: 4.

Question 93: In adults, the spinal cord normally ends at:

1. Lower border of L1

2. Lower border of L2

3. Lower border of L3

4. Lower border of L5

5.

Correct Answer: 1.

Question 94: Which of the following about meckel’s diverticulum is false

1. Present in 2% of population

2. Occurs at 2 feet form the ileocaecal junction

3. Posses all 3 coats of intestinal wall

4. Arises from the mesenteric border of ileum

5.

Correct Answer: 4.

Question 95: All of the following cell types contain the enzyme telomerase which protects the length of telomeres at the

end of chromosomes, except :

1. Somatic.

2. Germinal

3. Haemopoetic

4. Tumor

5.

Correct Answer: 1.

Page 19: FMGE (MCI Screening test)  preparation material by Gankidi Raghavender Reddy

Question 96: In a patient with a tumor in Superior medistinum compressing the superior vena cava, all the following venis

would serve as alternate pathways forthe blood to return to the right atrium, except

1. Lateral thoracic vein

2. Internal thoracic vein

3. Hemiazygos vein

4. Vertebral venous plexus

5.

Correct Answer: 3.

Question 97: Joint at Knee is

1. Synarthrosis

2. Amphiarthrosis

3. Diarthrosis

4. Symphysis

5.

Correct Answer: 2.

Question 98: The pancreatic bed does not include

1. Left kidney

2. Splenic artery

3. Left renal vein

4. Left crus of diaphragm

5.

Correct Answer: 2.

Question 99: The branches of internal iliac artery include all of the following except

1. Uterine artery

2. Middle rectal artery

3. Obturator artery

4. Inferior epigastric artery

5.

Correct Answer: 4.

Question 100: Urothelium lines all except:

1. minor calyces

2. ureter

3. urinary bladder

4. collecting duct

5.Correct Answer: 4.

Question 1: Mathemtical representation of Conserved region known as

1. Motif

2. Profile

Page 20: FMGE (MCI Screening test)  preparation material by Gankidi Raghavender Reddy

3. Pattern

4. Module

5.

Correct Answer: 1.

Question 2: The generation of messenger RNA expression profiles is referred as

1. transcription

2. trasncriptomics

3. Genomics

4. RNAsyn

5.

Correct Answer: 2.

Question 3: Which is the literature database of the following:

1. GEO

2. PubMed

3. EST

4. MMDB

5.

Correct Answer: 2.

Question 4: All of the following are carbohydrates EXCEPT:

1. starch

2. glycogen

3. chitin

4. cholestrol

5.

Correct Answer: 4.

Question 5: Books available in the following site

1. EMBL

2. NCBI

3. RCSB

4. PIR

5.

Correct Answer: 2.

Question 6: Lipinskis rule states that the drug has not more than ________ hydrogen bond donors.

1. 2

2. 3

3. 4

4. 5

5.

Correct Answer: 2.

Page 21: FMGE (MCI Screening test)  preparation material by Gankidi Raghavender Reddy

Question 7: Which type of BLOSUM matrix mostly used

1. BLOSUM62

2. BLOSUM72

3. BLOSUM82

4. BLOSUM92

5.

Correct Answer: 1.

Question 8: ________________ is 3D protein viewer.

1. Marvin space

2. Rasmol

3. Swiss PDB Viewer

4. All

5.

Correct Answer: 4.

Question 9: Phylogenetic analysis tools include

1. Phylip

2. PAUP

3. Both

4. none

5.

Correct Answer: 4.

Question 10: In how many centres clinical trials should be carried out

1. 1-2

2. 2-3

3. 3-4

4. 4-5

5.

Correct Answer: 3.

Question 11: How many SNPs present in Human Genome reported by HGP?

1. 2.2 million

2. 2.1 million

3. 3.3 billion

4. 4.2 billion

5.

Correct Answer: 2.

Question 12: tRNA genes are identified by using.

1. tRNAscan

2. Translate tool

3. Chime

Page 22: FMGE (MCI Screening test)  preparation material by Gankidi Raghavender Reddy

4. Hex

5.

Correct Answer: 1.

Question 13: The main enzyme responsible for activation of xenobiotics is

1. Cytochrome P-450

2. Glutathione S-transferase

3. NADPH cytochrome P-450-reductase

4. Glucuronyl transferase

5.

Correct Answer: 3.

Question 14: Name the protein sequence database

1. Uniprot

2. PIR

3. Swiss-prot

4. all of the above

5.

Correct Answer: 4.

Question 15: Which sequencing method efficiently helpful for Human Genome Project?

1. Sanger Method

2. Maxm-Gilbert method

3. Whole Genome Shotgun

4. None

5.

Correct Answer: 3.

Question 16: Translate tool at ExPASy provides, how many numbers of frames of frames of translation

1. 3

2. 4

3. 5

4. 6

5.

Correct Answer: 4.

Question 17: Pyrite oxidation thesis on ground water pollution mainly describes about____ poisoning of ground water

1. mercury

2. arsenic

3. aluminum

4. copper

5.

Correct Answer: 2.

Question 18: Leads means

Page 23: FMGE (MCI Screening test)  preparation material by Gankidi Raghavender Reddy

1. Chemical

2. drug

3. Possible drug

4. All

5.

Correct Answer: 4.

Question 19: Porphobilinogen in urine produces pink color with

1. Fouchet’s reagent

2. Benedict’s reagent

3. Sodium nitroprusside

4. Ehrlich’s aldehyde reagent

5.

Correct Answer: 4.

Question 20: What is the database used in ensembl

1. Oracle

2. MS-Access

3. Mysql

4. all of the above

5.

Correct Answer: 3.

Question 21: Which of the following aminoacids is associated with increased risk of Myocardial infarction

1. Methionine

2. Homocysteine

3. Ornithine

4. Valine

5.

Correct Answer: 2.

Question 22: Which of the following is used in PET scans?

1. uranium

2. ions

3. isotopes

4. steroids

5.

Correct Answer: 3.

Question 23: Macrophages in the walls of arteries have receptors for

1. Oxidised IDL

2. Oxidased LDL

3. LDL

4. IDL

5.

Page 24: FMGE (MCI Screening test)  preparation material by Gankidi Raghavender Reddy

Correct Answer: 2.

Question 24: A traditional chemist can synthesize _____________ compounds per year.

1. 100-200

2. 200-300

3. 300-400

4. 400-500

5.

Correct Answer: 1.

Question 25: Clustalw used for

1. Structure prediction

2. Motif analysis

3. Multiple sequence alignment

4. None

5.

Correct Answer: 3.

Question 26: When first completed report of Human Genome was released?

1. 2000

2. 2001

3. 2002

4. 2003

5.

Correct Answer: 1.

Question 27: Name the software for Natural Language Processing

1. openNLP

2. SPSS

3. SAS

4. None

5.

Correct Answer: 1.

Question 28: Which type of PAM matrix mostly used

1. PAM 100

2. PAM150

3. PAM200

4. PAM250

5.

Correct Answer: 3.

Question 29: Large genome annotated database is

1. Genbank

Page 25: FMGE (MCI Screening test)  preparation material by Gankidi Raghavender Reddy

2. Uniprot

3. Ensembl

4. MMDB

5.

Correct Answer: 3.

Question 30: ____________ tool used for Pattern construction.

1. Pratt

2. Patcons

3. Both

4. None

5.

Correct Answer: 3.

Question 31: What is the percentage of Human Genome sequenced properly?

1. 50%

2. 60%

3. 94%

4. 84%

5.

Correct Answer: 3.

Question 32: Secondary structure predicted by

1. GOR

2. COILS

3. Nnpred

4. All

5.

Correct Answer: 2.

Question 33: Server which predicts the presence and location of signal peptide cleavage sites is

1. Sigpred

2. SignalI

3. SignalP

4. Sigprepred

5.

Correct Answer: 3.

Question 34: Pharmacophore term defined by

1. Pauling

2. Ehrlich

3. Koshland

4. Watson

5.

Correct Answer: 3.

Page 26: FMGE (MCI Screening test)  preparation material by Gankidi Raghavender Reddy

Question 35: _______ - is an interconnected group of artificial neurons and processes information using a connectionist

approach to computation.

1. Neural network

2. Network

3. Internetwork

4. Intranetwork

5.

Correct Answer: 1.

Question 36: In the following acts as secondary pollutant.

1. Br2

2. Cl2

3. NO2

4. HNO3

5.

Correct Answer: 4.

Question 37: Which is the identifier in Genbank

1. Entry Name

2. Locus

3. both

4. None

5.

Correct Answer: 2.

Question 38: When Human Genome Project formally proposed?

1. 1975

2. 1985

3. 1995

4. 2005

5.

Correct Answer: 2.

Question 39: Characters are homologous with conserved function known as

1. Analogous

2. Orthologous

3. Paralogous

4. Xenologous

5.

Correct Answer: 1.

Question 40: ________________ is a database that provides comprehensive information on mammalian microRNAs.

1. Argonaute

2. RDP

Page 27: FMGE (MCI Screening test)  preparation material by Gankidi Raghavender Reddy

3. tRNADB

4. None

5.

Correct Answer: 1.

Question 41: ____is single largest source for carbon monoxide

1. Volcanoes eruptions

2. Dust storm

3. Cigarette smoke

4. Vehicular exhaust

5.

Correct Answer: 3.

Question 42: In CATH, T represents:

1. Torsion

2. Topology

3. Target

4. TRNA

5.

Correct Answer: 2.

Question 43: ___is the second layer of the atmosphere.

1. Stratosphere

2. thermosphere

3. Mesosphere

4. Troposphere

5.

Correct Answer: 4.

Question 44: When Human genome sequencing initiated?

1. 1996

2. 1997

3. 1998

4. 1999

5.

Correct Answer: 4.

Question 45: Phenylalanine is the precursor of all the following except

1. Tyrosine

2. Epinephrine

3. Thyroxine

4. Melatonin

5.

Correct Answer: 4.

Page 28: FMGE (MCI Screening test)  preparation material by Gankidi Raghavender Reddy

Question 46: The protein rich inbasic amino acids, which functions in the packaging of DNA in chromosomes, is

1. Histone

2. Collagen

3. Elastin

4. Hyaluronic acid binding protein

5.

Correct Answer: 1.

Question 47: The gas which produces systemic toxicity without causing local irritation is

1. Ammonium

2. Carbon monoxide

3. Hydrocyanic acid

4. Sulfur dioxide

5.

Correct Answer: 2.

Question 48: Competitive inhibition is characterized by

1. Increased Km and V Max is same

2. Increased Vmax and Km remains constant

3. Increased Km and increased Vmax

4. Decreased Km and increased Vmax

5.

Correct Answer: 1.

Question 49: The primary role of chaperones is to help in

1. Protein synthesis

2. Protein degradation

3. Protein denaturation

4. Protein folding

5.

Correct Answer: 4.

Question 50: BioEdit used for

1. Similarity studies

2. Structure Prediction

3. Profile search

4. Motif search

5.

Correct Answer: 2.

Question 51: When Human Genome Project work was officially started?

1. 1970

2. 1980

3. 1990

Page 29: FMGE (MCI Screening test)  preparation material by Gankidi Raghavender Reddy

4. 2000

5.

Correct Answer: 3.

Question 52: How many species support provided by Ensembl as by 47th release on 2007

1. 55

2. 45

3. 35

4. 25

5.

Correct Answer: 3.

Question 53: Entropy in a biological system does not increase because

1. It is an open system

2. It is a closed system

3. It is governed by vitalism

4. It is not related to thermodynamics

5.

Correct Answer: 1.

Question 54: Consider the following statements: 1. Caffeine, a constituent of tea and coffee is a diuretic. 2. Citric acid is

used in soft drinks. 3. Ascorbic acid is essential for the formation of bones and teeth. 4. Citric acid is a good substitution for

ascorbic acid in our nutrition. Which of the statements given above are correct?

1. 1 and 2, only

2. 1, 2 and 3, only

3. 3 and , only

4. 1, 2, 3 and 4

5.

Correct Answer: 1.

Question 55: The conversion of an optically pure isomer (enantiomer) into a mixture of equal amounts of both dextro and

levo forms is called as:

1. Polymerization

2. Stereoisomerization.

3. Racemization

4. Fractionation

5.

Correct Answer: 3.

Question 56: Gene prediction tool at NCBI is

1. Genscan

2. Genefinder

3. ORF finder

4. All

5.

Page 30: FMGE (MCI Screening test)  preparation material by Gankidi Raghavender Reddy

Correct Answer: 2.

Question 57: The cohesion of water is caused by:

1. ionic bonds

2. hydrophobic compounds

3. hydrogen bonds

4. covalent bonds

5.

Correct Answer: 3.

Question 58: Elasticity of the corneal layer of skin is due to the presence of

1. Histidine

2. Cysteine

3. Keratin

4. Lysine

5.

Correct Answer: 2.

Question 59: How many groups involved in HGP?

1. 10

2. 20

3. 30

4. 40

5.

Correct Answer: 2.

Question 60: Which of the following is not a post transcriptional modification of RNA

1. Splicing

2. 5' Capping

3. 3' polyadenylation

4. Glycosylation

5.

Correct Answer: 4.

Question 61: The part of enzyme bound to the protein part by a covalent bond is called

1. Holoenzyme

2. Cofactor

3. Prosthetic group

4. Apoenzyme

5.

Correct Answer: 3.

Question 62: How many ways Denovo designing might be used for drug designing?

1. 1

Page 31: FMGE (MCI Screening test)  preparation material by Gankidi Raghavender Reddy

2. 2

3. 3

4. 4

5.

Correct Answer: 1.

Question 63: Which pathway database based on reference pathway maps:

1. Metacyc

2. Ecocyc

3. KEGG pathway

4. UM-BBD

5.

Correct Answer: 1.

Question 64: Soil with equal mixture of clay, sand. Silt and humus is called

1. Gloom

2. Loams

3. Lombs

4. Roam

5.

Correct Answer: 2.

Question 65: Source of the nitrogen in urea cycle

1. aspartate and ammonia

2. glutamate and ammonia

3. argenase and ammonia

4.

5.

Correct Answer: 1.

Question 66: Before analyzing sequence, it can be converted into different formats using

1. Formcon

2. Readseq

3. Readcon

4. Confor

5.

Correct Answer: 4.

Question 67: 3DPSSM predicts tertiary structure through

1. Homology

2. Threading

3. Abinitio

4. All

5.

Correct Answer: 3.

Page 32: FMGE (MCI Screening test)  preparation material by Gankidi Raghavender Reddy

Question 68: Clinical trials for Vaccine contains _________ number of phases.

1. 1

2. 2

3. 3

4. 4

5.

Correct Answer: 3.

Question 69: The buffering capacity of a buffer is maximum at pH equal to

1. 0.5pKa

2. pKa

3. pKa+1

4. 2pKa

5.

Correct Answer: 2.

Question 70: Tool used for Local alignment is

1. BLAST

2. ClustalW

3. ORF Finder

4. Genscan

5.

Correct Answer: 3.

Question 71: Pair-wise alignment include

1. Local

2. Global

3. Both

4. None

5.

Correct Answer: 2.

Question 72: Which produces the abinitio prediction of protein function from sequence.

1. ProtFun

2. PFP Server

3. Pratt

4. none

5.

Correct Answer: 3.

Question 73: Which alignment might be useful for phylogenetic analysis

1. Pair-wise

2. Multiple

3. Both

Page 33: FMGE (MCI Screening test)  preparation material by Gankidi Raghavender Reddy

4. none

5.

Correct Answer: 1.

Question 74: Designing a drug molecule atom by atom against a receptor or target known as

1. Denovo

2. QSAR

3. Pharmacophore mapping

4. Docking

5.

Correct Answer: 1.

Question 75: It is a freely accessible database of protein and genetic interactions

1. GOD

2. MOTIF

3. BIOGRID

4. None

5.

Correct Answer: 3.

Question 76: An enzyme involved in the catabolism of fructose to pyruvate in the liver is

1. Glyceraldehyde-3-phosphate Dehydrogenase

2. Phosphoglucomutase

3. Lactate-dehydrogenase

4. Glucokinase

5.

Correct Answer: 1.

Question 77: Apo B 48 and Apo B 100 differ due to

1. RNA splicing

2. Apo B gene

3. Chromosomal loci

4. Gene rearrangement

5.

Correct Answer: 1.

Question 78: One of the following not comes under classification of databases on the basis of type of data:

1. Nucleotide sequence

2. Protein sequence

3. Primary db

4. Metabolic pathway

5.

Correct Answer: 3.

Question 79: In oxidative phosphorylation, the ATP production and respiratory chain are linked by

Page 34: FMGE (MCI Screening test)  preparation material by Gankidi Raghavender Reddy

1. Chemical methods

2. Physical methods

3. Chemiosmotic methods

4. Conformational changes

5.

Correct Answer: 3.

Question 80: The collagen triple helix structure is not found in

1. Cytoplasm

2. Golgi apparatus

3. Lumen of endoplasmic reticulum

4. Intracellular vesicles

5.

Correct Answer: 1.

Question 81: In Collagen synthesis, hydroxyproline is converted to

1. Proline

2. Lysine

3. Hydroxy lysine

4. None of the above

5.

Correct Answer: 1.

Question 82: The study of the effects of drugs on tissues and organs is called as

1. Pharmacodynamcis

2. Pharmacokinetics

3. Pharmacophore

4. All

5.

Correct Answer: 4.

Question 83: KEGG is an structural database

1. True

2. False

3. its a scheduling database

4. numerical database

5.

Correct Answer: 2.

Question 84: Homology Modelling server is

1. SWISS-MODEL

2. 3DPSSM

3. Phyre

4. Rosetta

5.

Page 35: FMGE (MCI Screening test)  preparation material by Gankidi Raghavender Reddy

Correct Answer: 4.

Question 85: B-oxidation of odd-chain fatty acids produces

1. Succinyl CoA

2. Propionyl CoA

3. Acetyl CoA

4. Malonyl CoA

5.

Correct Answer: 2.

Question 86: Which of the following elements is the LEAST abundant in living organisms?

1. oxygen

2. nitrogen

3. phosphorous

4. sodium

5.

Correct Answer: 4.

Question 87: Which of the following is true regarding hydroxyl ethyl starch

1. It is an anesthetic agent

2. It is a plasma expander

3. It is a crystalloid

4. Used as a nutritional agent

5.

Correct Answer: 2.

Question 88: What is the type of Recombination vector generally recommended for HGP?

1. Plasmid

2. Cosmid

3. BAC

4. YAC

5.

Correct Answer: 3.

Question 89: Fatty acids that are unsaturated have:

1. an amino group

2. a double bond

3. an excess of protons

4. a carboxyl group

5.

Correct Answer: 2.

Question 90: ExPASY used for

1. Nucleic acid analysis

Page 36: FMGE (MCI Screening test)  preparation material by Gankidi Raghavender Reddy

2. Protein sequence and structure analysis

3. chemical analysis

4. All

5.

Correct Answer: 2.

Question 91: Basic amino acids are

1. aspartate and glutamate

2. serine and glycine

3. lysine and arginie

4.

5.

Correct Answer: 3.

Question 92: Which components of Marvin Beans used for structure drawing

1. Marvin Sketch

2. Marvin View

3. Marvin space

4. All

5.

Correct Answer: 1.

Question 93: Artificial Intelligence term is coined by.

1. John McCarthy

2. Watson

3. Meselson

4. George Mitchel

5.

Correct Answer: 1.

Question 94: Transition temperature of lipid bilayers of cell membranes is increased by

1. Cholesterol

2. Saturated fatty acids

3. Unsaturated fatty acids

4. Hydrocarbons

5.

Correct Answer: 2.

Question 95: How the fasta format file starts:

1. begin

2. >

3. start

4. entry

5.

Correct Answer: 2.

Page 37: FMGE (MCI Screening test)  preparation material by Gankidi Raghavender Reddy

Question 96: The primary defect which leads to sickle cell anemia is

1. An abnormality in porphyrin part of hemoglobin

2. Replacement of glutamate by valine in B-chain of HbA

3. A nonsense mutation in the B-chain of HbA

4. Substitution of valine by glutamate in the a-chain of HbA

5.

Correct Answer: 2.

Question 97: Pathway prediction system of UM-BBD predicts pathway from

1. SMILES

2. structures

3. Both

4. None

5.

Correct Answer: 1.

Question 98: Substrate level phosphorylation is seen in conversion of

1. Acetoacetate to alpha keto glutarate

2. Succinyl CoA to Succinate

3. Fumarate to malate

4. Succinate to fumarate

5.

Correct Answer: 2.

Question 99: PDB stands for

1. Public DNA Bank

2. Protein Data Bank

3. Protein DNA Bank

4. All of the above

5.

Correct Answer: 2.

Question 100: Handerson Hasselbach equation is for

1. Electrolytes

2. Acid base balance

3. Fluid

4. Enzymatic reaction

5.

Question 1: An upper limit of noise which people can tolerate without sub damage to their hearing is

1. 45 db

2. 65 db

Page 38: FMGE (MCI Screening test)  preparation material by Gankidi Raghavender Reddy

3. 85 db

4. 105 db

5.

Correct Answer: 3.

Question 2: A child aged 24 months was brought to the Primary Health Centre with complaints of cough and fever for the

past 2 days. On examination, the child weighed 11 Kg. respiratory rate was 38 per minute, chest indrawing was present.

The most appropriate line of management for this patient is?

1. Classify as pneumonia and refer urgently to secondary level hospital

2. Classify as pneumonia, start antibiotic and advise to report after 2 days

3. Classify as severe pneumonia, start antibiotics and refer urgently

4. Classify as severe pneumonia and refer urgently

5.

Correct Answer: 3.

Question 3: In leprosy nerve involved is

1. Ulnar Nerve

2. Median nerve

3. Radial Nerve

4. Axillary nerve

5.

Correct Answer: 1.

Question 4: Point of control in tuberculosis is when Prevalence is

1. < 1 in 0-14 group of children

2. > 1% is all children

3. <1% in 0-5yrs of age group

4. <2% in 0-14 group

5.

Correct Answer: 1.

Question 5: A measure of location which divides the distribution in the ratio of 3:1 is

1. Median

2. First quartile

3. Third quartile

4. Mode

5.

Correct Answer: 3.

Question 6: WHO defines blindness if visual acuity is

1. Less than 3/60 vision

2. Less than 5/60

3. Less than 6/6

4. Less than 2/60

5.

Page 39: FMGE (MCI Screening test)  preparation material by Gankidi Raghavender Reddy

Correct Answer: 1.

Question 7: Which of the following gives the best idea regarding the morbidity in a community

1. Sentinal surveillance

2. Active surveillance

3. Passive surveillance

4. None of the above

5.

Correct Answer: 2.

Question 8: The most important function of sentinel surveillance is

1. To find total amount of disease in a population

2. To plan effective control measures

3. To determine the trend of disease in a population

4. To find out reqiured control measures to control disease

5.

Correct Answer: 1.

Question 9: Leprosy is considered a public health problem if the prevalence of leprosy is more than

1. 1 per 10,000

2. 2 per 10,000

3. 5 per 10,000

4. 10 per 10,000

5.

Correct Answer: 1.

Question 10: Which one of the following is not a contraindication for giving a live attenuated viral vaccine

1. Current febrile illness

2. Recent administration of immunoglobulin

3. Immunosuppressive disorder

4. Administration of another live vaccine

5.

Correct Answer: 1.

Question 11: For the calculation of positive predictive value of a screening test, the denominator is comprised of

1. True positives +False negatives

2. False positives + True negatives

3. True positives + False positives

4. True positives + True negatives

5.

Correct Answer: 3.

Question 12: Which one of the following diseases has the shortest incubation period

1. Salmonella food poisoning

Page 40: FMGE (MCI Screening test)  preparation material by Gankidi Raghavender Reddy

2. Staphylococcus food poisoning

3. Cholera

4. Diphtheria

5.

Correct Answer: 1.

Question 13: Zero population growth rate is seen in

1. Sweden and GDR

2. USA and Russia

3. France and Japan

4. No country in the world so far

5.

Correct Answer: 1.

Question 14: Which of the following statements is true regarding pertusis

1. Neurological complication rate of DPT is 1 in 50000

2. Vaccine efficacy is more than 95%

3. Erythromycin prevents spread of disease between children

4. Leukocytosis correlates with the severity of cough

5.

Correct Answer: 3.

Question 15: Median weight of 100 children was 12kgs. The Standard Deviation was 3. Calculate the percent coefficient of

variance

1. 25%

2. 35%

3. 45%

4. 55%

5.

Correct Answer: 1.

Question 16: Which one among the following edible oils yields highest quantity of polyunsaturated fatty acids (PUFA)

1. Coconut oil

2. Corn oil

3. Groundnut oil

4. Sunflower oil

5.

Correct Answer: 4.

Question 17: Maximum level of Chloride in Drinking water is

1. 600 mg/L

2. 200 mg/L

3. 300 mg/L

4. 400 mg/L

5.

Page 41: FMGE (MCI Screening test)  preparation material by Gankidi Raghavender Reddy

Correct Answer: 2.

Question 18: Path of longest duration is

1. Limiting path

2. Critical path

3. Cumulative path

4. None of the above

5.

Correct Answer: 2.

Question 19: In the context of epidemiology, the following are important criteria for making causal inferences, except

1. Strength of association

2. Consistency of association

3. Coherence of association

4. Predictive value

5.

Correct Answer: 4.

Question 20: In a study, variation in cholesterol was seen before and after giving a drug. The test which would give its

significance is

1. Unpaired t test

2. Paired t test

3. Chi square test

4. Fisher’s test

5.

Correct Answer: 2.

Question 21: Elemental iron and folic acid contents of pediatric iron-folic acid tablets supplied under Rural child Health

(RCH) Program are

1. 20 mg iron & 100 micrograms folic acid

2. 40 mg iron & 100 micrograms folic acid

3. 40 mg iron & 50 micrograms folic acid

4. 60 mg iron & 100 micrograms folic acid

5.

Correct Answer: 1.

Question 22: All of the following are random sampling methods except

1. Simple random

2. Cluster sampling

3. Stratified random

4. Quota Sampling

5.

Correct Answer: 4.

Question 23: Consider the following statement regarding drinking water samples : 1. No sample should be containts

Page 42: FMGE (MCI Screening test)  preparation material by Gankidi Raghavender Reddy

coliform organisms 2. No sample should containts E.coli 3. No two successive samples should contain coliform organisms 4.

No two successive samples should contain E.coli Which of the above criterion /criteria is /are included in the standards

prescribed by WHO for drinking water?

1. 1 only

2. 1 and 2

3. 2 and 3

4. 1 and 4

5.

Correct Answer: 3.

Question 24: Savlon contains

1. Chlorhexidine and chlorxylenol

2. Cetavlon and chlorhexidine

3. Cetavlon and habitané

4. Hibitane and chlorxylenol

5.

Correct Answer: 2.

Question 25: The best indicator for measuring the replacement of females in a population is

1. Net reproduction rate

2. Gross reproduction rate

3. Total fertility rate

4. Age specific fertility rate

5.

Correct Answer: 1.

Question 26: Following latrines are suitable for camps and temporary use except

1. Shollow trench latrine

2. Borehole latrine

3. Pit latrine

4. Septic tank

5.

Correct Answer: 4.

Question 27: Under EMCP (Enhanced Malaria Control Project) launched in 1997, the criteria for selection of PHCs (Primary

Health Centers) included the following except

1. API more than 2 for the last 3 years

2. Plasmodium falciparum more than 30 % of total malaria cases

3. The area has been reporting deaths

4. The area has been reporting epidemics

5.

Correct Answer: 4.

Question 28: According to WHO, adolescence generally means the age group of

1. 13 to 19 years

Page 43: FMGE (MCI Screening test)  preparation material by Gankidi Raghavender Reddy

2. 12 to 18 years

3. 11 to 15 years

4. 10 to 19 years

5.

Correct Answer: 1.

Question 29: Class II exposure in animal bites includes the following

1. Scratches without oozing of blood

2. Licks on a fresh wound

3. Scratch with oozing of blood on palm

4. Bites from wild animals

5.

Correct Answer: 2.

Question 30: About direct standardization all are true except

1. Age specific death rate is not needed

2. A standard population is needed

3. Population should be comparable

4. Two populations are compared

5.

Correct Answer: 1.

Question 31: According to the World Health Report 2000, India’s health expenditure is

1. 4.8% of G.D.P.

2. 5.2% of G.D.P.

3. 6.8% of G.D.P.

4. 7.2% of G.D.P.

5.

Correct Answer: 2.

Question 32: The amount of oral Vitamin A solution administered to children under Vitamin A prophylaxis programs is

1. One lakh units every 6 months

2. One lakh units every 1 year

3. Two lakh units every 6 months

4. Two lakh units every 1 years

5.

Correct Answer: 3.

Question 33: Which one of the followings statement about leprosy is true

1. Group surveys for case detection are carried out when prevalence of leprosy is less than 1 per1000

2. For determining bacteriologieal index,++ in a smear indicates 2 bacilli in every field

3. Minimum duration of treatment for paucibacillary cases is for 9 months

4. Minimum duration of treatment for multibacillary cases is for 12 months

5.

Correct Answer: 1.

Page 44: FMGE (MCI Screening test)  preparation material by Gankidi Raghavender Reddy

Question 34: Kata thermometer measures

1. Air temperature only

2. Air temperature and humidity

3. Air temperature humidity & air movement

4. None of the above

5.

Correct Answer: 3.

Question 35: PQLI index includes all except

1. Life expectancy at 1 yr of age

2. Per capita income

3. Literacy

4. Infant Mortality rate

5.

Correct Answer: 2.

Question 36: According to International Health Regulations (IHR) Act, a pregnant woman, with the following duration of

pregnancy (in weeks), cannot travel by air to other country

1. 20

2. 28

3. 32

4. 36

5.

Correct Answer: 4.

Question 37: Elements of primary health care include all of the following except

1. Adequate supply of safe water and basic sanitation

2. Providing essential drugs

3. Sound referral system

4. Health Education

5.

Correct Answer: 3.

Question 38: What is the correct recommended schedule [on days ] for post exposure treatment of persons who have

been vaccinated for rabies previously with HDC?

1. 0,3 and 7

2. 0 ,3,7 and 14

3. 0, 3, 7, 14 and 28

4. 0 and 3

5.

Correct Answer: 1.

Question 39: ‘Cut-off’ point to define obesity in “Body Mass Index” for females is

1. 30

Page 45: FMGE (MCI Screening test)  preparation material by Gankidi Raghavender Reddy

2. 28.6

3. 26

4. 25

5.

Correct Answer: 1.

Question 40: Consider the following: 1. Disease control phase 2. Health promotional phase 3. social engineering phase 4.

Health for all phase The correct sequence of distinct phases demarcated in the history of public health is

1. 1,2,3,4

2. 2,3,4,1

3. 2,1,3,4

4. 1,4,2,3

5.

Correct Answer: 1.

Question 41: In high risk areas the radial treatment for plasmodium vivax infection sfter microscopic confermation is

administration of tablets primaquine in the daily dosage of

1. 0.25 mg/Kg body weight

2. 0.5 mg/Kg body weight

3. 0.75 mg/Kg body weight

4. 1.0 mg/Kg body weight

5.

Correct Answer: 1.

Question 42: The biological oxygen demand indicates

1. Organic matter

2. Bacterial content

3. Anaerobic bacteria

4. Chemicals

5.

Correct Answer: 1.

Question 43: Active and passive immunity should be given together in all except

1. Rabies

2. Measles

3. Tetanus

4. Hepatitis B

5.

Correct Answer: 2.

Question 44: Scope of family planning services include all of the following except

1. Screening for cervical cancer

2. Providing services for unmarried mothers

3. Screening for HIV infection

4. Providing adoption services

5.

Page 46: FMGE (MCI Screening test)  preparation material by Gankidi Raghavender Reddy

Correct Answer: 3.

Question 45: The response which is graded by an observer on an agree or disagree continuum is based on

1. Visual analog scale

2. Guttman Scale

3. Likert Scale

4. Adjectival scale

5.

Correct Answer: 3.

Question 46: Ability of the test to detect absence of the disease in those who actually do not have it is called

1. Sensitivity

2. Specificity

3. Positive predictive value

4. Negative predictive value

5.

Correct Answer: 2.

Question 47: Ability of the test to detect absence of the disease in those who actually do not have it is called

1. Sensitivity

2. Specificity

3. Positive predictive value

4. Negative predictive value

5.

Correct Answer: 2.

Question 48: In one single visit, a 9 month-old, unimmunized children should begiven the following vaccination

1. Only BCG

2. BCG, DPT-1, OPV-1

3. DPT-1, OPV-1, Measles

4. BCG,DPT-1, OpV-1, Measles

5.

Correct Answer: 4.

Question 49: The extra energy allowances needed per day during pregnancy is

1. 150 KCals

2. 300 KCals

3. 400 KCals

4. 550 KCals

5.

Correct Answer: 2.

Question 50: Reservoir for Yellow fever is

1. Man

Page 47: FMGE (MCI Screening test)  preparation material by Gankidi Raghavender Reddy

2. Aedes

3. Monkey

4. Rat

5.

Correct Answer: 3.

Question 51: Prudent diet is

1. Diet for dietary goal achievement

2. Diet, which contains variety of foods to safe guard from deficiencies

3. Diet on which a person or group lives

4. Diet, which fulfills recommended daily allowances

5.

Correct Answer: 1.

Question 52: The correlation between variables A and B in a study was found to be 1.1. This indicates

1. Very strong correlation

2. Moderately strong correlation

3. Weak correlation

4. Computational mistake in calculating correlation

5.

Correct Answer: 4.

Question 53: By international agreement , low birth weight has been defined as a birth weight when measured within the

first hour of life is

1. Less than 2000 grams

2. Less than 2500 grams

3. Less than 2800 grams

4. Less than 3000 grams

5.

Correct Answer: 2.

Question 54: The drug of choice for chemoprophylaxis of cholera in adults needs the adminstration of

1. Doxycycline [300 mg] once/ single dose

2. Tetracycline in a dose of 500 mg twice daily for 3 days

3. Metronidazole [400 mg] thrice daily for 7 days

4. Chloramphenicol [500 mg] thrice daily for 7 days

5.

Correct Answer: 2.

Question 55: The best indicator for monitoring the impact of Iodine Deficiency Disorders Control programme is:

1. Prevalence of Goiter among school children

2. Urinary iodine levels among pregnant women

3. Neonatal Hypothyroidism

4. Iodine level in soil

5.

Page 48: FMGE (MCI Screening test)  preparation material by Gankidi Raghavender Reddy

Correct Answer: 3.

Question 56: Incubation period of mumps is

1. 18 days

2. 10 days

3. 7 days

4. 12 days

5.

Correct Answer: 1.

Question 57: Under the National Polio Eradication Program, a case of Acute Flaccid Paralysis is confirmed as Polio, under

the following circumstances except

1. If a case is lost to follow up

2. If a case could not be confirmed because the patient died before that

3. If a wild strain of polio virus is isolated from stool

4. If a patient develops paralysis 30 days after diagnosis of AFP

5.

Correct Answer: 4.

Question 58: Chronic carriers in salmonella are

1. Faecal Carrier

2. Urinary Carriers

3. Blood Carriers

4. None of the above

5.

Correct Answer: 1.

Question 59: According to the Central Birth and Death registration act of 1969, birth is to be reported within

1. 10 days

2. 7 days

3. 14 days

4. 30 days

5.

Correct Answer: 3.

Question 60: Which one among the following edible oils yields highest quantity of polyunsaturated fatty acids (PUFA)

1. Coconut oil

2. Corn oil

3. Groundnut oil

4. Sunflower oil

5.

Correct Answer: 4.

Question 61: The purpose of a double-blind study is to

Page 49: FMGE (MCI Screening test)  preparation material by Gankidi Raghavender Reddy

1. Reduce the effects of sampling variation

2. Avoid observer and subject bias

3. Avoid observer bias and sampling variation

4. Avoid subject bias and sampling variation

5.

Correct Answer: 2.

Question 62: Which one of the following is a conjugated vaccine

1. Hepatitis B

2. Rubella

3. Pertussis

4. Hemophilus influenzae B

5.

Correct Answer: 1.

Question 63: The following statements about meningococcal meningitis are true, except

1. The source of infection is mainly clinical cases

2. The disease is more common in dry and cold months of the year

3. Chemoprophylaxis of close contacts of cases is recommended

4. The vaccine is not effective in children below 2 years of age

5.

Correct Answer: 1.

Question 64: Amount of Iodine in salt should be

1. 15 & 30ppm

2. 15 & 40 ppm

3. 20 & 40 ppm

4. 10 & 20 ppm

5.

Correct Answer: 1.

Question 65: A population study showed a mean glucose of 86 mg/ dL. In a sample of 100 showing normal curve

distribution, what percentage of people have glucose above 86%?

1. 65

2. 50

3. 75

4. 60

5.

Correct Answer: 2.

Question 66: For an adult Indian male the daily requirement of protein is expressed as

1. 0.5 gms/ Kg body weight

2. 0.75 gms/ Kg body weight

3. 1.0 gms/ Kg body weight

Page 50: FMGE (MCI Screening test)  preparation material by Gankidi Raghavender Reddy

4. 1.5 gms/ Kg body weight

5.

Correct Answer: 3.

Question 67: For calculation of sample size for a prevalence study all of the following are necessary except

1. Prevalence of the disease in population

2. Power of the study

3. Significance level

4. Desired precision

5.

Correct Answer: 3.

Question 68: In the management of leprosy, Lepromin test is most useful for

1. Herd Immunity

2. Prognosis

3. Treatment

4. Epidemiological investigations

5.

Correct Answer: 2.

Question 69: Physical quality of life index is measured by all except

1. Infant mortality rate

2. Life expectancy at age one

3. Per capita income

4. Literacy

5.

Correct Answer: 3.

Question 70: Screening of women above 35 years for cancer cervix, using the Pap smear, is a method of

1. Primordial prevention

2. Health promotion

3. Specific protection

4. Secondary prevention

5.

Correct Answer: 3.

Question 71: API stands for

1. Average parasitic index

2. Animal parasite interval

3. Annual parasitic index

4. None of the above

5.

Correct Answer: 3.

Question 72: Human development Index (HDI) does not consider

Page 51: FMGE (MCI Screening test)  preparation material by Gankidi Raghavender Reddy

1. Life expectancy

2. Literacy

3. Income

4. Infant mortality

5.

Correct Answer: 4.

Question 73: Level of proteins in human milk is

1. 0.8ng/dl

2. 0.9ng/dl

3. 1ng/dl

4. 2ng/dl

5.

Correct Answer: 1.

Question 74: The association between coronary artery disease and smoking was found to be as follows. Coronary Art Dis

No Coronary Art Dis Smokers 30 20 Nonsmokers 20 30 The Odds ratio can be estimated as

1. 0.65

2. 0.8

3. 1.3

4. 2.25

5.

Correct Answer: 4.

Question 75: Sullivan index indicates

1. Life free of disability

2. Hook worm eggs/gm of stool

3. Standard of living

4. Pregnancy rate per HWY

5.

Correct Answer: 1.

Question 76: Homes where children are placed under the care of doctors and psychiatrists are called

1. Foster homes

2. Borstals

3. Remand homes

4. Child guidance clinics

5.

Correct Answer: 2.

Question 77: Retrospective cohort studies are not characterized by

1. The study groups are exposed and non-expose

2. Incidence rates may be computed

3. The required sample size is smaller than that needed for a concurrent cohort study

Page 52: FMGE (MCI Screening test)  preparation material by Gankidi Raghavender Reddy

4. The required sample size is similar to that needed for concurrent cohort study

5.

Correct Answer: 4.

Question 78: Fluoride content in drinking H2O normally safe is

1. 0.5-0.8 mg/dl

2. 0.8-0.1 ng/dl

3. 0.2-0.8ng/dl

4. 0.2-0.5mg/dl

5.

Correct Answer: 1.

Question 79: Which one of the following 'outcome measures' is not indicative of the benefit of a disease screening

program

1. Reduction of case-fatality rate in screened individuals

2. Improvement in the quality of life in screened individuals

3. Reduction of incidence in the population screened

4. Reduction of complications

5.

Correct Answer: 3.

Question 80: Which one of the following has highest “Glycemic Index”?

1. Corn flakes

2. Ice cream

3. Brown rice

4. Whole wheat bread

5.

Correct Answer: 1.

Question 81: Essential obstetric care includes the following, except

1. Early registration of pregnancy (Within 12-16 weeks)

2. Provision of safe delivery

3. 24- hour delivery services at primary health centers (PHCs)

4. Provision of minimum three antenatal check-ups

5.

Correct Answer: 3.

Question 82: The Vitamin A supplement administered in Prevention of nutritional blindness in children programme"

contain:

1. 25,000 i.u./ml

2. 1 lakh i.u./m.l.

3. 3 lakh i.u./m.l.

4. 5 lakh i.u./m.l.

5.

Correct Answer: 2.

Page 53: FMGE (MCI Screening test)  preparation material by Gankidi Raghavender Reddy

Question 83: Serial interval is

1. Time gap between primary and secondary case

2. Time gap between index and primary case

3. Time taken for a person from infection to develop maximum infectivity

4. The time taken from infection till a person infects another person

5.

Correct Answer: 1.

Question 84: Most important epidemiological tool used for assessing disability in children is

1. Activities of Daily living (ADL) scale

2. Wing's Handicaps, Behavior and Skills (HBS) Schedule

3. Binet and Simon IQ tests

4. Physical Quality of Life Index (PQLI)

5.

Correct Answer: 1.

Question 85: In a prospective study comprising 10000 subjects, 6000 subjects were put on beta carotene and 4000 were

not. 3 out of the first 6000 developed lung cancer and 2 out of the second 4000 developed lung cancer. What is the

interpretation of the above?

1. Beta carotene is protective in lung cancer

2. Beta carotene is not protective in lung cancer

3. The study design is not sufficient to draw any meaningful conclusions

4. Beta carotene is carcinogenic

5.

Correct Answer: 2.

Question 86: Denominator while calculating the secondary attack rate includes

1. All the people living in next fifty houses

2. All the close contacts

3. All susceptibles amongst close contact

4. All susceptibles in the whole village

5.

Correct Answer: 3.

Question 87: The parameters of sensitivity and specificity are used for assessing

1. Criterion validity

2. Construct validity

3. Discriminant validity

4. Content validity

5.

Correct Answer: 1.

Question 88: Acute flaccid paralysis is reported in a child aged

1. 0-3 years

Page 54: FMGE (MCI Screening test)  preparation material by Gankidi Raghavender Reddy

2. 0-5 years

3. 0-15 years

4. 0-25 years

5.

Correct Answer: 3.

Question 89: Net protein utilization of fish is

1. 57%

2. 67%

3. 77%

4. 87%

5.

Correct Answer: 3.

Question 90: In North India most common congenital lesion seen is

1. Neural Tube defects

2. Cleft Palate

3. Congenital heart disease

4. Club foot

5.

Correct Answer: 1.

Question 91: Which cardiac defect is more common in congenital rubella syndrome

1. PDA

2. Aortic stenosis

3. Aortic regurgitation

4. MVP

5.

Correct Answer: 1.

Question 92: National Tree

National Tree of India

1. Peepal

2. Mango

3. coconut

4. Pine

5.

Correct Answer: 1.

Question 93: All the following are true in a randomized control trial (RCT) except

1. Baseline characteristics of intervention and control groups should be similar

2. Investigator’s bias is minimized by double blinding

3. The sample size required depends on the hypothesis

4. The drop-outs from the trial should be excluded from the analysis.

5.

Page 55: FMGE (MCI Screening test)  preparation material by Gankidi Raghavender Reddy

Correct Answer: 3.

Question 94: For a 60 kg Indian male, the minimum daily protein requirement has been calculated to be 40 g (mean) ± 10

(Standard deviation). The recommended daily allowance of protein would be

1. 60 g/ day

2. 70 g/ day

3. 30 g/ day

4. 90 g/ day

5.

Correct Answer: 1.

Question 95: The permissible dose of man made radiation should not exceed

1. 3 rads per year

2. 5 rads per year

3. 8 rads per year

4. 12 rads per year

5.

Correct Answer: 2.

Question 96: Consider the following contraceptives: 1. Copper T 2. Contraceptive Pill 3. Condom 4. Spermicides Which of

these are conventional Contraceptives

1. 1 and 2

2. 1,2 and 3

3. 3 and 4

4. 2,3 and 4

5.

Correct Answer: 3.

Question 97: Positive Schick test indicates

1. Immunity to diphtheria

2. Susceptibility to diphtheria

3. Hypersensitivity to diphtheria

4. Infection with diphtheria

5.

Correct Answer: 2.

Question 98: World health day is on

1. 7th April

2. 10th October

3. 21st May

4. 23rd June

5.

Correct Answer: 1.

Question 99: Acquisition of skills is known as

Page 56: FMGE (MCI Screening test)  preparation material by Gankidi Raghavender Reddy

1. Cognitive learning

2. Affective learning

3. Psychomotor learning

4. Learning by conditioned reflex

5.

Correct Answer: 1.

Question 100: The number of years of healthy life lost due to all causes whether from premature mortality or from

disability is called

1. Quality adjusted life lost years(QALYs)

2. Disability adusted life years (DALYs)

3. Sullivan's index

4. Standardised mortality ratio (SMR)

5.Correct Answer: 3.

Question 1: A male aged 60 years has foul breath, he regurgitates food that is eaten 3 days ago. Likely diagnosis is

1. Zenker's diverticulum

2. Meckel's diverticulum

3. Scleroderma

4. Achalasia cardia

5.

Correct Answer: 1.

Question 2: Prime modality of treatment of Naso Pharyngeal Carcinoma is

1. Radiotherapy

2. Surgery

3. Chemotherapy

4. None of the above

5.

Correct Answer: 1.

Question 3: A middle aged male comes to the outer patient department (OPD) with the only complaint of hoarseness of

voice for the past 2 years. he has been a chronic smoker for 30 years. On examination, a reddish area of mucosal

irregularity overlying a portion both cords was seen. Management would include all except

1. Cessation of smoking

2. Bilateral cordectomy

3. Microlaryngeal surgery for biopsy

4. Regular follow-up

5.

Correct Answer: 2.

Question 4: The most common site of carcinoma of tongue is

1. Posterior one -third

2. Dorsum of anterior two-third

Page 57: FMGE (MCI Screening test)  preparation material by Gankidi Raghavender Reddy

3. Ventral surface of anterior two-third

4. Lateral border of anterior two-third

5.

Correct Answer: 4.

Question 5: Peritonsillar abscess can extend posteriorly into

1. Anterior triangle of neck

2. Parapharyngeal space

3. Posterior triangle on neck

4. Submaxillary space

5.

Correct Answer: 2.

Question 6: A 5 year old boy has been diagnosed to have posterior superior retraction pocket cholesteatoma. All would

constitute part of the management, except

1. Audiometry

2. Mastoid exploration

3. Tympanoplasty

4. Myringoplasty

5.

Correct Answer: 4.

Question 7: Carharts notch in audiogram is deepest frequency of

1. 0.5 KHz

2. 2 KHz

3. 4 KHz

4. 8 KHz

5.

Correct Answer: 2.

Question 8: The most common diagnosis in a young boy with history of profuse nasal bleeding is

1. Juvenile Naso pharyngeal angiofibroma

2. Nasal polyp

3. Deviated nasal septum

4. Cirrhosis

5.

Correct Answer: 1.

Question 9: All of the following cause a grey-white membrane in the throat EXCEPT

1. Streptococcal tonsillitis

2. Diphtheria

3. Ludwig's angina

4. Adenoviral pharyngitis

5.

Correct Answer: 3.

Page 58: FMGE (MCI Screening test)  preparation material by Gankidi Raghavender Reddy

Question 10: The most common cause of ASOM in children is

1. Streptococcus pneumonia

2. Hemophilus influenza

3. E. coli

4. Moraxella

5.

Correct Answer: 1.

Question 11: Causes of posterior soft tissue nasopharyngeal mass include all EXCEPT

1. Plasmacytoma

2. Choanal atresia

3. Thornwaldt’s Cyst

4. Aneurysm of the carotid artery

5.

Correct Answer: 2.

Question 12: Thornwaldt disease in inflammation of

1. Pharyngeal bursa

2. Subhyoid bursa

3. Crypta Magna

4. Piriform fossa

5.

Correct Answer: 1.

Question 13: Submandibular space infection is known as

1. Ludwig's angina

2. Vincent's angina

3. Parapharyngeal abscess

4. None

5.

Correct Answer: 1.

Question 14: Adenocarcinoma of esophagus is commonly found in

1. Achalasia acardia

2. Barrett's oesophagus

3. Plummer Vinson syndrome

4. Chronic smoking

5.

Correct Answer: 2.

Question 15: Conductive deafness is caused by all except

1. Otitis media with effusion

2. Tumours of the external canal

3. Perforated ear drum

Page 59: FMGE (MCI Screening test)  preparation material by Gankidi Raghavender Reddy

4. Viral infection like mumps

5.

Correct Answer: 4.

Question 16: Indications for tracheostomy are all EXCEPT

1. Acute epiglottitis

2. Maxillofacial trauma

3. Laryngeal malignancy

4. Extensive consolidation of lung

5.

Correct Answer: 4.

Question 17: Post cricoid malignancy is due to

1. Alcohol

2. Smoking

3. Plummer vinson syndrome

4. None of the above

5.

Correct Answer: 3.

Question 18: Which of the following is not the site for PARAGANGLIOMA

1. Carotid bifurcation

2. Jugular foramen

3. Promontory in middle ear

4. Geniculate ganglion

5.

Correct Answer: 4.

Question 19: After a long-standing tracheostomy patient developed almost complete stenosis of trachea, treatment is

1. Tracheal dilation

2. Laser with stent

3. Surgery

4. Removal of stenosed part with anastomosis

5.

Correct Answer: 4.

Question 20: Chronic pharyngeal abcess is due to

1. Caries in spine

2. Tuberculosis of lungs

3. Cancer of larynx

4. Chronic streptococcal infection

5.

Correct Answer: 1.

Question 21: Tensor of the Vocal cord is

Page 60: FMGE (MCI Screening test)  preparation material by Gankidi Raghavender Reddy

1. Cricothyroid

2. Posterior Crico arytenoids

3. Lateral crico arytenoids

4. Thyro arytenoids

5.

Correct Answer: 1.

Question 22: Which one of the following statements is not true about epiglottitis

1. The most frequent causative agent is pneumococcus

2. Presents with high fever and stridor

3. Stridor diminishes with fatigue

4. Examination of throat may cause sudden death

5.

Correct Answer: 1.

Question 23: Left sided vocal cord palsy is commonly due to

1. Left hilar bronchial carcinoma

2. Mitral Stenosis

3. Thyroid Malignancy

4. Thyroid Surgey

5.

Correct Answer: 1.

Question 24: Bano Begum presented with bleeding from ear, pain, tinnitus and increasing deafness. Examination revealed

red swelling/mass behind the intact tympanic membrane that blanches on pressure with pneumatic speculum. Treatments

for her include all EXCEPT:

1. Preoperative embolisation

2. Radiotherapy

3. Surgery

4. Interferons

5.

Correct Answer: 4.

Question 25: False positive fistula test is seen in

1. Perilymph fistula

2. Malignant sclerosis

3. Congenital syphilis

4. Cholesteatoma

5.

Correct Answer: 3.

Question 26: Access to airway is gained during emergency by

1. Endotracheal tube

2. Emergency Tracheostomy

3. Crico thyroidotomy

Page 61: FMGE (MCI Screening test)  preparation material by Gankidi Raghavender Reddy

4. All of the above

5.

Correct Answer: 4.

Question 27: Investigation of choice for Nasopharyngeal angiofibroma is

1. Helical CT

2. Angiography

3. MRI

4. CECT

5.

Correct Answer: 2.

Question 28: The most common tumor of the salivary gland is

1. Mucoepidermoid tumor

2. Pleomorphic adenoma

3. Acinic cell tumor

4. Warthin’s tumor

5.

Correct Answer: 2.

Question 29: Dangerous area of Nose is

1. Little’s area

2. Woodruff’s area

3. Olfactory area

4. Posterior wall

5.

Correct Answer: 1.

Question 30: Third molar caries with extension of the lesion towards tonsillar fossa and shift of tonsil reveals which of the

following complication

1. Parapharyngeal abscess

2. Retropharyngeal abscess

3. Tonsillar abscess

4. Dental abscess

5.

Correct Answer: 1.

Question 31: The opening in case of Dacrocysto rhinostomy is made in

1. Superior Meatus

2. Middle Meatus

3. Inferior Meatus

4. Sphenoidal sinus

5.

Correct Answer: 2.

Page 62: FMGE (MCI Screening test)  preparation material by Gankidi Raghavender Reddy

Question 32: A tracheostomised patient, with portex tracheostomy tube, in the ward, developed sudden complete

blockage of the tube. Which of the following is best next step in the management?

1. Immediate removal of the tracheostomy tube

2. Suction of tube with sodium bicarbonate

3. Suction of tube with saline

4. Jet ventilation

5.

Correct Answer: 1.

Question 33: Palatal myoclonus is seen in

1. Epilepsy

2. Multiple sclerosis

3. Cerebellar infarction

4. Guillain Barre syndrome

5.

Correct Answer: 2.

Question 34: Hyperkeratosis of palm and sole is seen in

1. Carcinoma colon

2. Hepatoma

3. Adenocarcinoma lung

4. CA oesophagus

5.

Correct Answer: 4.

Question 35: Odynophagia is

1. Pain during swallowing

2. Difficulty in swallowing

3. Bad odour from mouth

4. Psychiatric disease

5.

Correct Answer: 1.

Question 36: Organ of corti is situated in

1. Basilar membrane

2. Utricle

3. Saccule

4. None of the above

5.

Correct Answer: 1.

Question 37: The antibiotic of choice in acute epiglottitis pending culture sensitivity report is

1. Erythromycin

2. Rolitetracycline

3. Doxycycline

Page 63: FMGE (MCI Screening test)  preparation material by Gankidi Raghavender Reddy

4. Ampicillin

5.

Correct Answer: 1.

Question 38: Vocal cord paralysis is due to involvement of which of the following nerves

1. External laryngeal

2. Internal laryngeal

3. Recurrent laryngeal

4. Superior laryngeal

5.

Correct Answer: 3.

Question 39: A chronic smoker with history of hoarseness was found on examination to have keratosis of the larynx. All

the following are possible treatment modalities except

1. Laser

2. Radiotherapy

3. Stripping of the vocal cord

4. Partial laryngectomy

5.

Correct Answer: 4.

Question 40: Killiance dehiscence is seen in

1. Oropharynx

2. Nasopharynx

3. Cricopharynx

4. Vocal cords

5.

Correct Answer: 3.

Question 41: An 18 year old boy presents with epistaxis and unilateral nasal mass. The most likely diagonosis is

1. Rhinoscleroma

2. Angiofibroma

3. Rhinosporidiosis

4. Antrochoanal polyp

5.

Correct Answer: 2.

Question 42: An eight month old infant had stridor with respiratory difficulty which worsened on crying. On examination, it

was found to have a subglottic reddish mass. All the following can be used in the management except

1. Tracheostomy

2. Laser vaporization

3. Corticosteroid

4. Radiotherapy

5.

Correct Answer: 4.

Page 64: FMGE (MCI Screening test)  preparation material by Gankidi Raghavender Reddy

Question 43: Main problem associated with carotid body tumor operation is

1. The tumor blends with bifurcation of carotid artery

2. The tumor blends with jugular vein

3. Recurrence

4. Vaso vagal Shock

5.

Correct Answer: 1.

Question 44: Lalloo, a 55 year old chronic smoker, presents with complaints of hoarseness of voice. On examination there

is a single enlarged painless lymphnode in the left supraclavicular area. What is the next step to be done

1. Excision biopsy of the node

2. Laryngoscopy and Chest X ray

3. CT scan

4. Sputum for AFB

5.

Correct Answer: 2.

Question 45: Myosis is

1. Maggots seen is nose

2. Maggots in the anus

3. Inflammatory disease of nose

4. Necrotic inflammation with maggots in ear

5.

Correct Answer: 1.

Question 46: Swelling between tonsillar area and superior constrictor muscle is know as

1. Quinsy

2. Dental abscess

3. Parapharyngeal abscess

4. Retropharyngeal abscess

5.

Correct Answer: 1.

Question 47: The treatment of choice of a glottic cancer with stage T1N0M0 is

1. Brachytherapy

2. External beam radiotherapy

3. Laryngectomy

4. Chemotherapy

5.

Correct Answer: 2.

Question 48: The presenting symptoms in majority cases of Acoustic Neuroma is

1. Hearing Loss

2. Vertigo

Page 65: FMGE (MCI Screening test)  preparation material by Gankidi Raghavender Reddy

3. Signs of Space occupying lesions

4. Pain

5.

Correct Answer: 1.

Question 49: The main differential diagnosis is second branchial cleft cyst include all EXCEPT

1. Thyroglossal duct cyst

2. Cystic hygroma

3. Laryngocele

4. Ranula

5.

Correct Answer: 4.

Question 50: Not true regarding pterygopalatine fossa

1. Inferomedial to the foramen rotundum lies the Vidian (Pterygoid) canal, which connects the foramen lacerum to the

pterygopalatine fossa

2. Medially, the pterygopalatine fossa communicates through the pterygomaxillary fissure with the infratemporal

fossa

3. It communicates with the middle cranial cavity through the foramen rotundum and pterygoid canal.

4. It contains the sphenopalatine ganglion and the third segment of the maxillary artery.

5.

Correct Answer: 2.

Question 51: Peritonsillar abscess is also known as

1. Retropharyngeal abscess

2. Tonsillar abscess

3. Quinsy

4. Thornwaldt's abscess

5.

Correct Answer: 3.

Question 52: Tullio phenomenon is

1. Better hearing in Noisy surroundings

2. Increased sensitivity

3. Impedance

4. Vertigo in noisy surroundings

5.

Correct Answer: 1.

Question 53: Collar stud abscess is seen in

1. Pyogenic cervical abscess

2. Peritonsillar abscess

3. Retropharyngeal abscess

4. TB lymphadenitis

5.

Page 66: FMGE (MCI Screening test)  preparation material by Gankidi Raghavender Reddy

Correct Answer: 4.

Question 54: Which of the following is the most beneficial technique of using chemotherapy with a course of radiotherapy

in head and neck malignancies

1. Neo adjuvant chemotherapy

2. Adjuvant chemotherapy

3. Concurrent chemotherapy

4. Alternating chemotherapy and radiotherapy

5.

Correct Answer: 1.

Question 55: Corkscrew esophagus is seen in which of the following conditions

1. Carcinoma esophagus

2. Scleroderma

3. Achalasia cardia

4. Diffuse esophageal spasm

5.

Correct Answer: 4.

Question 56: Reaction time for tymphanic reflex is

1. 20 to 30 ms

2. 30 to 40 ms

3. 40 to 160 ms

4. 600 to 800 ms

5.

Correct Answer: 3.

Question 57: 'Cork-screw' esophagus is seen in

1. Vigorous achalasia cardia

2. Scleroderma

3. Diffuse esophageal spasm

4. Carcinoma esophagus

5.

Correct Answer: 3.

Question 58: Type I Thyroplasty is for

1. Vocal cord medialization

2. Vocal cord laterlization

3. Vocal Cord shortening

4. Vocal cord lengthening

5.

Correct Answer: 1.

Question 59: Normal ear drums are seen in ----- % cases of otoscelerosis

Page 67: FMGE (MCI Screening test)  preparation material by Gankidi Raghavender Reddy

1. 90

2. 75

3. 70

4. 78

5.

Correct Answer: 1.

Question 60: White oral lesions are seen in EXCEPT

1. Leukoplakia

2. Keratosis

3. Addison's disease

4. Candidiasis

5.

Correct Answer: 3.

Question 61: Why vocal cord appears pale ?

1. Vocal cord is muscle, lack of blood vessels network

2. Absence of mucosa, no blood vessels

3. Absence of sub mucosa, no blood vessels

4. Absence of mucosa with blood vessels

5.

Correct Answer: 3.

Question 62: Noise level which can be tolerated by ears is

1. 85 dB

2. 70 dB

3. 120 dB

4. 100 dB

5.

Correct Answer: 1.

Question 63: What is the correct sequence of the following while resuscitating an infant with Foreign Body Airway

Obstruction? 1. Chest thrust 2. Tongue-jaw lift 3. Back blows Select the correct sequence form the codes given below:

1. 1,3,2

2. 3,2,1

3. 3,1,2

4. 2,1,3

5.

Correct Answer: 3.

Question 64: Columella effect is seen in

1. Tympanoplasty

2. Septoplasty

3. Tracheostomy

Page 68: FMGE (MCI Screening test)  preparation material by Gankidi Raghavender Reddy

4. None of the above

5.

Correct Answer: 1.

Question 65: Most common complication of Tracheostomy is

1. Stenosis

2. Infection

3. Pneumonia

4. Respiratory failure

5.

Correct Answer: 2.

Question 66: Weber test is done by

1. Placing the tuning fork on the vertex and examined after partial occlusion of the auditory canal

2. Placing the tuning fork on the forehead and by assessing the side of better hearing

3. Placing the tuning fork on the mastoid and assessing the side of better hearing

4. Placing the tuning fork on the mastoid and examined after partial occlusion of the auditory canal

5.

Correct Answer: 2.

Question 67: A 70 year old man presents with complaints of tinnitus. Which of the following is the most probable

diagnosis

1. Labrynthitis

2. Acoustic trauma

3. ASOM

4. Acoustic neuroma

5.

Correct Answer: 4.

Question 68: Openings of the tube of bronchoscope are known as

1. Holes

2. Apertures

3. Vents

4. Any of the above

5.

Correct Answer: 3.

Question 69: The most common malignancy of the oropharynx

1. Tonsil

2. Soft palate

3. Tongue base

4. Valleculae

5.

Correct Answer: 1.

Page 69: FMGE (MCI Screening test)  preparation material by Gankidi Raghavender Reddy

Question 70: Which of the following structure is seen in Oro Pharynx

1. Pharyngotympanic tube

2. Fossa of Rosenmuller

3. Palatine Tonsil

4. Piriform Fossa

5.

Correct Answer: 3.

Question 71: A 31 year old female patient complaints of bilateral impairment of hearing for the past 5 years. On

examination, tympanic membrance is normal and aduiogram shows a bilateral conductive deafness. Impedance

audiometry. Shows as type of curve and acoustic reflexes are absent. All constitute part of treatment, except

1. Hearing aid

2. Stapedectomy

3. Sodium Fluoride

4. Gentamicin

5.

Correct Answer: 4.

Question 72: Acoustic dip in audiogram in noise induced hearing loss is at

1. 4 KHz

2. 3 KHz

3. 2 KHz

4. 1 KHz

5.

Correct Answer: 1.

Question 73: Presbyacusis is

1. Age associated vision loss

2. Age associated hearing loss

3. Both

4. None

5.

Correct Answer: 2.

Question 74: Scanty, foul smelling, painless discharge from the ear is a characteristic feature of which of the following

lesions

1. Acute otitis media

2. Otitis externa

3. Central perforation

4. Cholesteatoma

5.

Correct Answer: 4.

Question 75: Intermaxillary suture are sometimes raised forming a longitudinal midline ridge known as

Page 70: FMGE (MCI Screening test)  preparation material by Gankidi Raghavender Reddy

1. Torus auditory

2. Torus mandibularis

3. Torus palatine

4. Torus maxillaris

5.

Correct Answer: 3.

Question 76: Maxillary Sinus opens in

1. Superior Meatus

2. Inferior meatus

3. Infundibulum

4. None of the above

5.

Correct Answer: 3.

Question 77: Fibrous layer is absent in

1. Pars tensa

2. Pars flaccida

3. Umbo

4. Annulus

5.

Correct Answer: 2.

Question 78: Rhinitis sicca is due to drying of

1. Anterior wall of Nose

2. Postetior Wall of nose

3. Septum

4. Roof of Nose

5.

Correct Answer: 3.

Question 79: To do myringotomy, the incision is put in the posterior inferior region. This is the preferred region for all the

following reasons except

1. It is easily accessible

2. Damage to chorda tympani is avoided

3. Damage to ossicular chain does not occur

4. It is the least vascular region

5.

Correct Answer: 4.

Question 80: A false positive fistula test is seen in all the following except

1. Hypermobile ossicular chain

2. Labyrinthine fistula

3. Dead ear

Page 71: FMGE (MCI Screening test)  preparation material by Gankidi Raghavender Reddy

4. Post fenestration

5.

Correct Answer: 3.

Question 81: Congenital stridor is caused by

1. Laryngomalacia

2. Laryngeal stenosis

3. Ca Larynx

4. Foreign body

5.Correct Answer: 1.

Question 1: Blood stain of humun origin is tested by

1. Preciptin test

2. Heamin crystal test

3. Haemochromogen test

4. Benzidine test

5.

Correct Answer: 3.

Question 2: Widmark's formula is used for measurement of blood levels of

1. Alcohol

2. Barbiturates

3. Arsenic

4. Benzodiazepines

5.

Correct Answer: 1.

Question 3: Immediate stiffness post mortem occurs in

1. Cadaveric rigidity

2. Cadaveric spasm

3. Rigor mortis

4. Algor mortis

5.

Correct Answer: 2.

Question 4: The temperature of the body rises up for the first two hours after death. The probable condition include

following except

1. Sun stroke

2. Frost bite

3. Septicemia

4. Strychnine poisoning

5.

Correct Answer: 2.

Question 5: Foamy liver is seen in

Page 72: FMGE (MCI Screening test)  preparation material by Gankidi Raghavender Reddy

1. Putrefaction

2. Drowning

3. Mummification

4. Adipocere

5.

Correct Answer: 1.

Question 6: Post mortem staining of lower parts of hands and feet indicates

1. Prolonged suspension after death

2. Antemortem hanging

3. Death due to asphyxia

4. Drowning

5.

Correct Answer: 1.

Question 7: Which one of the following is NOT a principle followed in the management of missile injuries

1. Excision of all dead muscles

2. Removal of foreign bodies

3. Removal of fragments of bone

4. Leaving the wound open

5.

Correct Answer: 3.

Question 8: Rigor mortis is simulated by

1. Cadaveric spasm

2. Mummification

3. Algor mortis

4. All of the above

5.

Correct Answer: 1.

Question 9: If a body is left undisturbed for how long does post-mortem staining persist

1. Few hours

2. Few days

3. Few months

4. Persists till it demerges with discoloration of putrefaction

5.

Correct Answer: 4.

Question 10: A patient with myocardial infarction, after death, underwent autopsy. Microscopic examination of the cardiac

tissue as part of autopsy revealed granulation tissue. The time from infarction till death is most likely to be

1. < 24 hours

2. < 2 hours

3. < 10 days

Page 73: FMGE (MCI Screening test)  preparation material by Gankidi Raghavender Reddy

4. < 1 month

5.

Correct Answer: 3.

Question 11: In rigor mortis muscle

1. Stiffens

2. Shortens

3. Stiffens and shortens

4. Stiffens and lengthens

5.

Correct Answer: 3.

Question 12: Entire absence of fat throughout the body in post mortem is suggestive of

1. Starvation

2. Tuberculosis

3. Diabetes

4. All of the above

5.

Correct Answer: 1.

Question 13: Presence of maggots at post-mortem indicates

1. Duration since death

2. Cause of death

3. Mode of Death

4. Posture of body at death

5.

Correct Answer: 1.

Question 14: Which of the following is not a diagnostic feature for allergic fungal sinusitis?

1. CT shows hyperdense / hyperechoic mass

2. Orbital extension / orbital penetration

3. Allergic reaction / type 1 hypersensitivity

4. Steroids are used in management

5.

Correct Answer: 2.

Question 15: Rigor mortis first starts in

1. Upper eyelids

2. Lower eyelids

3. Lower limbs

4. Fingers

5.

Correct Answer: 1.

Question 16: A drug addict gives history of tactile sensations all over his body. He is likely ot have consumed

Page 74: FMGE (MCI Screening test)  preparation material by Gankidi Raghavender Reddy

1. Opium

2. Heroin

3. Cannabis

4. Cocaine

5.

Correct Answer: 4.

Question 17: What disappears first in Post mortem rigidity

1. Eyelids

2. Neck

3. Lower limbs

4. Upper limbs

5.

Correct Answer: 1.

Question 18: Post mortem staining can occur in some cases before death in

1. Children

2. Old persons

3. Cholera

4. All of the above

5.

Correct Answer: 3.

Question 19: Rigor mortis is simulated by

1. Cadaveric spasm

2. Mummification

3. Algor mortis

4. All of the above

5.

Correct Answer: 1.

Question 20: Rigor mortis develops__ after death

1. 1/2 - 1 hour

2. 1-2 hours

3. 3-6 hours

4. 6-8 hours

5.

Correct Answer: 3.

Question 21: Post mortem staining gets fixed after

1. 2-3 hours

2. 3-4 hours

3. 5-6 hours

4. 6-7 hours

5.

Page 75: FMGE (MCI Screening test)  preparation material by Gankidi Raghavender Reddy

Correct Answer: 4.

Question 22: A skull was found which had an oval nasal opening, horseshoe shaped palate, round orbits and a cephalic

index greater than 80. The race to which it belongs is probably belongs is most probably

1. Negroes

2. Mongols

3. Caucasians

4. Aryans

5.

Correct Answer: 2.

Question 23: Paultaf's hemorrhage is seen in which injury?

1. Strangulation

2. Hanging

3. Drowning

4. Burns

5.

Correct Answer: 3.

Question 24: Post mortem wound best differs from the antemortem wound by

1. Gaps on incising

2. No clots

3. Absence of erythema and cellular changes

4. Removable stain

5.

Correct Answer: 3.

Question 25: True about Cadaveric spasm

1. Instant in onset

2. Confined to small group of muscles

3. Occurs only in voluntary muscles

4. Indicates that the person was alive at that time

5. All of the above

Correct Answer: 5.

Question 26: Among the following, the drugs that are of use in a case of Ectopic pregnancy include

1. Adriamycin

2. Actinomycin D

3. Potassium fluoride

4. Methotrexate

5.

Correct Answer: 4.

Question 27: Which of the following sign will indicate that the death was antemortem

Page 76: FMGE (MCI Screening test)  preparation material by Gankidi Raghavender Reddy

1. Cutis anserina

2. Washerwoman’s skin

3. Weed & grass in hands

4. Cyanosis

5.

Correct Answer: 3.

Question 28: In a decomposed body the first sign seen is

1. Greenish discolouration over skin of right iliac fossa

2. Greenish discolouration over skin over left iliac fossa

3. Purplish black discolouration on face

4. Purplish black discolouration over lower extremities

5.

Correct Answer: 1.

Question 29: Elevated levels of cyanide is seen in death due to which of the following

1. Cold

2. Scald

3. Starvation

4. Thermal burns

5.

Correct Answer: 4.

Question 30: Which one of the tissues putrefies late

1. Brain

2. Prostate

3. Liver

4. Stomach

5.

Correct Answer: 2.

Question 31: On firearm injuries, blackish discoloration around the entry wound is due to

1. Flame

2. Smoke

3. Deposition of un burnt powder

4. Hot gases

5.

Correct Answer: 3.

Question 32: Blood clots after__ of death

1. 1/2 hr

2. 45 min

3. 1/4hr

4. 1 1/2 hrs

5.

Page 77: FMGE (MCI Screening test)  preparation material by Gankidi Raghavender Reddy

Correct Answer: 1.

Question 33: Dying deposition is recorded by

1. Doctor

2. Magistrate

3. Police officer

4. Lawyer

5.

Correct Answer: 2.

Question 34: A patient is brought with opium poisoning. The antagonists to opium include

1. Buprenorphine

2. Meptazinol

3. Nalorphine

4. Neostigmine

5.

Correct Answer: 3.

Question 35: The postmortem finding seen in smothering includes

1. Bruising in inner aspect of upper lip

2. Fracture body of hyoid

3. Fracture of cricoid

4. Curved marks on neck

5.

Correct Answer: 1.

Question 36: One of the following is a definite sign of death due to drowning

1. Sand and mud in the mud and nostrils

2. Fine lathery froth in mouth and nostrils

3. Washerwoman’s hand and feet

4. Diatoms in bone marrow

5.

Correct Answer: 4.

Question 37: Sentence of death is passed in which judicial court

1. High court

2. Supreme court

3. Session court

4. Assistant session court

5.

Correct Answer: 1.

Question 38: Hypostasis lasts for

1. Hours

Page 78: FMGE (MCI Screening test)  preparation material by Gankidi Raghavender Reddy

2. Days

3. Weeks

4. Months

5.

Correct Answer: 2.

Question 39: Spinal cord is opened from

1. Frontal approach

2. Back side

3. Lateral side

4. Any of the above

5.

Correct Answer: 2.

Question 40: True statement about adipocere

1. It is due to dehydration of the subcutaneous tissue

2. It is due to hydrogenation of subcutaneous fat

3. It is formed in the epidermal tissues

4. It is retarded by heat and accelerated by cold

5.

Correct Answer: 2.

Question 41: Aseptic autolysis is found in

1. Adipocere

2. Putrefaction

3. Mummification

4. Maceration

5.

Correct Answer: 4.

Question 42: Reddish brown colour in Post mortem lividity

1. CO poisoning

2. Cyanide poisoning

3. Phosphorus poisoning

4. Aniline poisoning

5.

Correct Answer: 4.

Question 43: Postmortem in a newborn baby is done by opening first

1. The skull

2. The chest cavity

3. The abdominal cavity

4. As per convenience

5.

Correct Answer: 3.

Page 79: FMGE (MCI Screening test)  preparation material by Gankidi Raghavender Reddy

Question 44: Correct order of putrefaction

1. Brain, Heart, Uterus

2. Brain, Uterus, Heart

3. Uterus, Heart, Brain

4. Brain, Uterus, Heart

5.

Correct Answer: 1.

Question 45: In suspected case of death due to poisoning where cadaveric rigidity is lasting longer than usual, it may be a

case of poisoning due to

1. Lead.

2. Arsenic

3. Mercury

4. Copper

5.

Correct Answer: 2.

Question 46: Scab or Crust of abrasion appears brown

1. Between 2-3 days

2. Between 5-6 days

3. Between 3-5 days

4. Between 12-24 hours

5.

Correct Answer: 1.

Question 47: Post-mortem autopsy is done in

1. Whole body

2. Parts which are injured

3. Parts under inquest

4. Parts under consent from relatives

5.

Correct Answer: 1.

Question 48: Activated charcoal is used in poisoning with

1. Alcohol

2. Barbiturates

3. Lead

4. Mercury

5.

Correct Answer: 2.

Question 49: Which of the following is true regarding cadaveric spasm

1. No primary relaxation

2. No secondary relaxation

Page 80: FMGE (MCI Screening test)  preparation material by Gankidi Raghavender Reddy

3. No primary or secondary relaxation

4. No change

5.

Correct Answer: 1.

Question 50: What is suspended animation

1. The animal is suspended in air

2. Hibernation

3. The subject is alive but shows no signs of life

4. Deep sleep

5.

Correct Answer: 3.

Question 51: The most reliable criteria in Gustafson's method of identification is

1. Cementum apposition

2. Transparency of root

3. Attrition

4. Root resorption

5.

Correct Answer: 2.

Question 52: Tests for heavy metals are all except

1. Harrison and Gilfroy’s test

2. Neutron Emission test

3. Atomic spectroscopy

4. Paraffin test

5.

Correct Answer: 4.

Question 53: Post mortem cherry red discolouration is due to

1. Asphyxia

2. Carbon monoxide

3. Drowning

4. OPC poisoning

5.

Correct Answer: 2.

Question 54: Cardiac Polyp is a term used for Postmortem

1. Fibrinous clots in heart

2. Aneurysm

3. Infarcts in heart

4. Pale patches in heart

5.

Correct Answer: 1.

Page 81: FMGE (MCI Screening test)  preparation material by Gankidi Raghavender Reddy

Question 55: The following situations are associated with rise of temperature after death EXCEPT

1. Burns

2. Heat Strokes

3. Pontine Hemorrhage

4. Septicemia

5.

Correct Answer: 1.

Question 56: Blue line on the gums is seen in chronic poisoning with

1. Mercury

2. Lead

3. Arsenic

4. Antimony

5.

Correct Answer: 2.

Question 57: Tentative cuts are seen in a case of

1. Homicide

2. Suicide

3. Accidents

4. Fall from height

5.

Correct Answer: 2.

Question 58: Basophilic stippling is seen in

1. RBC

2. WBC

3. Neutrophilis

4. Basophil

5.

Correct Answer: 1.

Question 59: Maggots appear in body in rainy season within

1. 24 hours

2. 1-2 days

3. 2-3 days

4. 3-4 days

5.

Correct Answer: 2.

Question 60: Paltauf’s haemorrhage may be seen in

1. Hanging

2. Drowning

3. Strangulation

Page 82: FMGE (MCI Screening test)  preparation material by Gankidi Raghavender Reddy

4. Carbon monoxide poisoning

5.

Correct Answer: 2.

Question 61: Reasons for marked variation in the shape of exit wounds of rifled weapons

1. Deformation of bullet during its passage through body and thereby presenting an irregular wound

2. Absence of tumbling of the bullet in the body & hence it may not be able to exit with nose end first

3. Intact bullet in the body after striking bone

4. At times , both entry and exit wounds are of the same as in the assassination of kennedy , President of U

5.

Correct Answer: 1.

Question 62: In a completely charred body at autopsy medical officers can say that the burns are ante-mortem if he can

find

1. Soot in respiratory passages

2. Cyanosis of finger nails

3. Congestion of kidney

4. Pale internal organs

5.

Correct Answer: 1.

Question 63: Patient of head injury, has no relatives, requires urgent cranial decompression; Doctor should

1. operate without formal consent

2. take police consent

3. wait for relatives

4. take

5.

Correct Answer: 1.

Question 64: Blackening and tattooing of skin and clothing can be best demonstrated by

1. Luminol spray

2. Infra red photography

3. UV light

4. Magnifying lens

5.

Correct Answer: 2.

Question 65: Which of the following sign will indicate that menstrual blood was antemortem

1. Alkaline

2. Doesn’t clot unless mixed with semen

3. Light pink

4. Shows endometrial & vaginal cells

5.

Correct Answer: 2.

Page 83: FMGE (MCI Screening test)  preparation material by Gankidi Raghavender Reddy

Question 66: Perjury means giving willfull false evidence by a witness while under oath, the witness is liable to be

prosecuted for perjury and the imprisonment may extend to seven years. This falls under which section of IPC

1. 190 of Indian Penal Code

2. 191 of Indian Penal Code

3. 192 of Indian Penal Code

4. 193 of Indian Penal Code

5.

Correct Answer: 4.

Question 67: Which of the following is not correct regarding diatoms

1. Diatoms are aquatic unicellular plant

2. Diatoms has an extracellular coat composed of magnesium

3. Acid digestion technique is used to extract diatoms

4. Presence of diatoms in the Bone marrow is an indication of antemortem

5.

Correct Answer: 2.

Question 68: Which organ putrefies last in females

1. Brain

2. Breast

3. Uterus

4. Ovary

5.

Correct Answer: 3.

Question 69: Rate of cooling helps in determining

1. Time of death

2. Manner of death

3. Place of death

4. Cause of death

5.

Correct Answer: 1.

Question 70: After post-mortem autopsy body is handed over to

1. The authority who had conducted inquest in that case

2. Police Station near by

3. Coroner

4. Magistrate

5.

Correct Answer: 1.

Question 71: Post mortem lividity is unlikely to develop in a case of

1. Drowning in well

2. Drowning in fast flowing river

3. Postmortem submersion

Page 84: FMGE (MCI Screening test)  preparation material by Gankidi Raghavender Reddy

4. Drowning in swimming pool

5.

Correct Answer: 2.

Question 72: Immediate rigidity of a group of muscle without passing into stage of primary relaxation

1. Cadaveric spasm

2. Cadaveric rigidity

3. Both

4. None

5.

Correct Answer: 1.

Question 73: The rate of cooling down of dead bodies in tropical climate is

1. 0.20 C / hr

2. 0.50C / hr

3. 1.0 C / hr

4. 1.50 C / hr

5.

Correct Answer: 2.

Question 74: Barberio,s test is done for

1. Blood

2. Urine

3. Saliva

4. Semen

5.

Correct Answer: 4.

Question 75: Mummification is due to

1. Putrefaction

2. Desication

3. Saponification

4. Necrosis

5.

Correct Answer: 2.

Question 76: One of the following is true of antemortem abrasion

1. Bright red in colour

2. Exudation of serum is more

3. Intravital reaction is seen

4. All of the above

5.

Correct Answer: 1.

Question 77: Post mortem lividity is unlikely to develop in a case of

Page 85: FMGE (MCI Screening test)  preparation material by Gankidi Raghavender Reddy

1. Drowning in well

2. Drowning in fast flowing river

3. Post mortem submersion

4. Drowning in clorinated swimming pool

5.

Correct Answer: 2.

Question 78: The hydrostatic test need not be performed in case of

1. Infanticide

2. Spongy or crepitant

3. Lungs or Liver like in consistency

4. None

5.

Correct Answer: 2.

Question 79: All the following are related to legal responsibility of an insane person except

1. Mc Naughten's rule

2. Durham's rule

3. Rule of nine

4. Curren's rule

5.

Correct Answer: 3.

Question 80: At autopsy, a body was found to have fine froth from the nose and mouth which increased on compression

of the chest. Which of the following is the most likely cause of death

1. Opioid poisoning

2. Hanging

3. Drowning

4. Cyanide poisoning

5.

Correct Answer: 3.

Question 81: Maggots are seen on the dead body after

1. One day

2. 2 to 3 days

3. 3-4 days

4. One week

5.

Correct Answer: 2.

Question 82: Tentative cuts are seen in a case of

1. Homicide

2. Suicide

3. Accident

Page 86: FMGE (MCI Screening test)  preparation material by Gankidi Raghavender Reddy

4. Drowning

5.

Correct Answer: 2.

Question 83: Antemortem blister differs from postmortem blister by

1. Presence of Albumin & Chloride in blister fluid

2. Gas in blister

3. Dry hard surface of the floor of blister

4. Absence of hyperemia around the blister

5.

Correct Answer: 1.

Question 84: Which of the following is not a feature of brain death

1. Complete apnea

2. Absent pupillary reflex

3. Absent deep tendon reflex

4. Heart rate unresponsive to atropine

5.

Correct Answer: 3.

Question 85: More than 5% carboxyhaemoglobin is indicative of

1. Ante mortem burns

2. Drowning

3. HCN poisoning

4. Suffocation

5.

Correct Answer: 1.

Question 86: Mummification refers to

1. Hardening of muscles after death

2. Colliquative putrifaction

3. Saponification ofsubcutaneous fat

4. Dessication of a dead body

5.

Correct Answer: 4.

Question 87: Endotracheal intubations prior to gastric lavage is done in cases of poisoning

1. To prevent aspiration

2. Due to fear of cardiac arrest

3. For ventilatory support

4. For easy passage of gastric tube

5.

Correct Answer: 3.

Question 88: Mummification occurs when

Page 87: FMGE (MCI Screening test)  preparation material by Gankidi Raghavender Reddy

1. High atmospheric temperature is present

2. Dry Air condition

3. Wind is present

4. All of the above

5.

Correct Answer: 4.

Question 89: Post mortem caloricity is seen with

1. Strychnine poisoning

2. Lead poisoning

3. Mercury poisoning

4. Phosphorus poisoning

5.

Correct Answer: 1.

Question 90: . Number of hours since death is calculated by multiplying the fall in rectal temperature with

1. 0.33

2. 0.67

3. 1.0

4. 1.33

5.

Correct Answer: 2.

Question 91: Which of the following combination is correct

1. Deferroxamine - lead

2. Penicillamine - Copper

3. Dimercaprol - Iron

4. EDTA - Arsenic

5.

Correct Answer: 2.

Question 92: A patient has been allegedly bitten by cobra snake. The venom in such a bite would be

1. Musculotoxic

2. Vasculotoxic

3. Cardiotoxic

4. Neurotoxic

5.

Correct Answer: 4.

Question 93: In starvation, the post mortem appearance of gall bladder is

1. Atrophied

2. Distended

3. Show stones

4. Normal

5.

Page 88: FMGE (MCI Screening test)  preparation material by Gankidi Raghavender Reddy

Correct Answer: 2.

Question 94: In burnt bones, the following can be detected

1. Arsenic

2. Lead

3. Organophosphorus

4. Mercury

5.

Correct Answer: 1.

Question 95: Cafe Coronary is due to

1. Myocardial infarction

2. Asphyxia

3. Strangulation

4. Drowning

5.

Correct Answer: 2.

Question 96: Honey combing is seen in liver in cases of

1. Cirrhosis

2. Rupture

3. Putrefaction

4. Hydatid disease

5.

Correct Answer: 3.

Question 97: Vibices are

1. Arborescent markings

2. Lichenberg flowers

3. Marbling

4. Hypostasis in subcutaneous area

5.

Correct Answer: 1.

Question 98: Changes in retinal vessels soon after death include

1. Fragmentation of Blood column

2. Haemorrhage into vitreous

3. Collapse of vessels

4. None of the above

5.

Correct Answer: 1.

Question 99: Suspended animation occurs after

1. Electrocution

Page 89: FMGE (MCI Screening test)  preparation material by Gankidi Raghavender Reddy

2. Cyanide poisoning

3. Burns

4. Drowning

5.

Correct Answer: 1.

Question 100: Putrefaction starts in

1. Liver

2. Lung

3. Heart

4. Brain

5.Correct Answer: 1.

Question 1: The most common cause of osteomyelitis is

1. Staphylococcus aureus

2. Salmonella typhi

3. Streptococcus pyogens

4. Mycobacterium tuberculosis

5.

Correct Answer: 1.

Question 2: .Staphylococcus aureus differs from Staphylococcus epidermidis by

1. Is coagulase positive

2. Forms white colonies

3. A common cause of UTI

4. Causes endocarditis in drug addicts

5.

Correct Answer: 1.

Question 3: The membrane attack complex consists of

1. Colicins

2. C3b, 2a

3. C5b,6,7,8,9

4. Properdin

5.

Correct Answer: 3.

Question 4: Long lasting immunity in hepatitis infection is due to

1. IgA

2. IgM

3. IgE

4. IgG

5.

Page 90: FMGE (MCI Screening test)  preparation material by Gankidi Raghavender Reddy

Correct Answer: 1.

Question 5: Erysepelas is caused by

1. Staphylococcus aureus

2. Pseudomonas

3. Streptococcus pyogens

4. Pneumococcus

5.

Correct Answer: 3.

Question 6: In Hepatitis B infection, window period is characterized by

1. Anti HBS antibody

2. HBS antigen

3. Anti HBE

4. HBE

5.

Correct Answer: 1.

Question 7: All of the following statements about NK cells are true. except

1. They are derived from large granular cells

2. They comprise about 5% of human peripheral lymphoid cells

3. They are MHC restricted cytotoxic cells

4. They express lgG Fc receptors

5.

Correct Answer: 3.

Question 8: Satellitism is seen in culture of

1. Hemophilus

2. Streptococcus

3. Klebsiella

4. Proteus

5.

Correct Answer: 1.

Question 9: Following is true about C-reactive protein

1. Detected by precipitation with carbohydrate

2. Raised in acute pneumococcal infection

3. It is an antibody

4. Detected by agglutination test

5.

Correct Answer: 2.

Question 10: Antigenic drift in influenza virus infections is due to

1. Small mutation in neuraminidase and hemaglutimin

Page 91: FMGE (MCI Screening test)  preparation material by Gankidi Raghavender Reddy

2. Large mutations in hemaglutimin only

3. Step mutations in viral genome

4. None of the above

5.

Correct Answer: 1.

Question 11: The mechanism of genetic transfer where a phage serves as a

1. Transduction

2. Translation

3. Lysogeny

4. Conjugation

5.

Correct Answer: 3.

Question 12: Epstein Barr virus causes all the following except

1. Infectious mononucleosis

2. Measles

3. Nasopharyngeal carcinoma

4. Non Hodgkins lymphoma

5.

Correct Answer: 2.

Question 13: All of the following infections are often associated with acute intravascular hemolysis except

1. Clostridium tetani.

2. Bartonella bacilliformis

3. Plasmodium falciparum

4. Babesia microti

5.

Correct Answer: 1.

Question 14: A beta hemolytic bacteria resistant to vancomycin, gentamycin and ampicillin. It shows growth in 6.5% NaCl

and bile is likely to be

1. strep agalactiae

2. strep pneumococcus

3. enterococcus

4. strep bovis

5.

Correct Answer: 3.

Question 15: Fimbriae are demonstrated by

1. Culture

2. Gram stain

3. Biochemical reaction

4. Haemagglutination test

5.

Page 92: FMGE (MCI Screening test)  preparation material by Gankidi Raghavender Reddy

Correct Answer: 4.

Question 16: Which of the following is common to NK cells and Cytotoxic T cells

1. Antibody production

2. Antiviral action

3. Antibody is required for cytotoxicity

4. HLA class II is involved

5.

Correct Answer: 2.

Question 17: Visceral larva migrans is caused by

1. Toxocara canis

2. Toxoplasma

3. Toxocora catis

4. Ascariasis

5.

Correct Answer: 1.

Question 18: Fungi affecting reticuloendothelial cells are

1. Cryptcoccus

2. Histoplasmosis

3. Sporothrix

4. Cryptosporidium

5.

Correct Answer: 2.

Question 19: Fibroblast in tissue culture form interferon of which type

1. Alpha

2. Beta

3. Gamma

4. All of the above

5.

Correct Answer: 2.

Question 20: In a patient, corneal scraping reveals narrow angled septate hyphae. Which of the following is the likely

etiologic agent

1. Mucor

2. Aspergillus

3. Histoplasma

4. Candida

5.

Correct Answer: 2.

Question 21: Retroviruses contain

Page 93: FMGE (MCI Screening test)  preparation material by Gankidi Raghavender Reddy

1. DNA polymerase

2. Reverse transcriptase

3. Segmented DNA

4. ds DNA

5.

Correct Answer: 2.

Question 22: Which of the following is true about Lymphokines

1. Alpha-interferon activates macrophages

2. Colony stimulating factors (CSF) stimulate bacterial growth

3. All CD4+ T-cells do not produce the same Lymphokines

4. The action of lymphokines is not antigen specific

5.

Correct Answer: 2.

Question 23: The following are true for Bordetella perussis except

1. It is a strict human pathogen

2. It can be cultured from the patient during catarrhal stage

3. It leads to invasion of the respiratory mucosa

4. Infection can be prevented by a acellular vaccine

5.

Correct Answer: 3.

Question 24: The pathogenesis of which one of the following infections does NOT occur through an exotoxin

1. Staphylococcus aureus

2. Escherichia coli

3. Clostridum botulinum

4. Streptococcus pyogenes

5.

Correct Answer: 4.

Question 25: Gasgangrene is due to the infection of

1. E.coli

2. Clostridium perfringens

3. Shigella

4. salmonella

5. all of the above

Correct Answer: 2.

Question 26: % VDRL POSTIVITY IN SECONDARY SYPHILIS

1. 100%

2. 60%

3. 70%

4. 80%

5.

Page 94: FMGE (MCI Screening test)  preparation material by Gankidi Raghavender Reddy

Correct Answer: 1.

Question 27: Memory T cells can be identified by using the following marker

1. CD45 RA

2. CD45RO

3. CD45RC

4. CD45 RB

5.

Correct Answer: 2.

Question 28: In a patient with AIDS chorioretinitis is typically caused by

1. Cytomegalovirus

2. Toxoplasma gondii

3. Cryptococcus neoformans

4. Histoplasma capsulatum

5.

Correct Answer: 1.

Question 29: C-reactive protein (CRP) is

1. Produced by Pneumococcus

2. Marker of septicemia

3. Raised in acute inflammation

4. Low in rheumatoid arthritis

5.

Correct Answer: 3.

Question 30: Germ tube test is diagnosis for

1. Candida albicans

2. Cryptococcus neoformans

3. Histoplasma capsulatum

4. Coccidiodomycosis

5.

Correct Answer: 1.

Question 31: HIV virus is a

1. Single stranded DNA

2. Single stranded RNA

3. Double stranded RNA

4. Double stranded DNA

5.

Correct Answer: 2.

Question 32: The earliest immunoglobulin to be synthesized by the fetus is

1. IgA

Page 95: FMGE (MCI Screening test)  preparation material by Gankidi Raghavender Reddy

2. IgM

3. IgD

4. IgG

5.

Correct Answer: 2.

Question 33: All the following are true about acute infection except

1. Specific IgM antibody occurs in acute infection

2. Specific IgM indicates rubella immune status

3. Immunofluorescence indicates influenza infection

4. ELISA is useful in Hepatitis B infection

5.

Correct Answer: 2.

Question 34: Which of the following statements is true about Bacteroides

1. It is a gram positive bacilli

2. It is strictly aerobic

3. It can cause peritonitis

4. Presence in stools culture indicates need for treatment

5.

Correct Answer: 3.

Question 35: Which of the following is transmitted by Rat urine

1. Leptospira

2. Listeria

3. Legionella

4. Mycoplasma

5.

Correct Answer: 1.

Question 36: Recommended vaccines for rabies

1. DPT

2. MMR

3. BCG

4. HCDV

5.

Correct Answer: 4.

Question 37: Plastic Syringes are sterilized by

1. Gamma rays

2. Hot air oven

3. Autoclave

4. Washing with phenol

5.

Correct Answer: 1.

Page 96: FMGE (MCI Screening test)  preparation material by Gankidi Raghavender Reddy

Question 38: In immunocompromised person, fungal viral, protozoal infection are due to deficiency of

1. T lymphocytes

2. B lymphocytes

3. Macrophages

4. Neutrophils

5.

Correct Answer: 1.

Question 39: A patient, Gopi, had lepromatous leprosy. Which of the following is true regarding globi in leprosy

1. Consists of lipid laden macrophages

2. Consists of macrophages filled with bacteria

3. Consists of neutrophils filled with bacteria

4. Degenerated neural tissue

5.

Correct Answer: 2.

Question 40: Dark ground microscopy is used for detection of

1. Spirochete

2. Myco-Tuberculosis

3. Myco-Leprae

4. Mycoplasma

5.

Correct Answer: 1.

Question 41: Which on of the following solutions is used to disinfect flexible endoscopes

1. 10 % chlorhexidine

2. Absolute alcohol

3. 2 % Glutaraldehyde

4. 5 % Paracetic acid

5.

Correct Answer: 3.

Question 42: Obligate anaerobes cannot withstand oxygen because of absence of

1. Superoxide dismutase

2. Catalase

3. Peroxidase

4. Cytochrome oxidase

5.

Correct Answer: 4.

Question 43: Which of the following are not a feature of the bacterial cell wall

1. Peptidoglycan

2. Lipopolysaccharide

3. Pilus protein

Page 97: FMGE (MCI Screening test)  preparation material by Gankidi Raghavender Reddy

4. Teichoic acid

5.

Correct Answer: 3.

Question 44: Flurescent microscopy is used to diagnose

1. Mycobacterium tubeculosis

2. Staphylococcus aureus

3. Streptococcus capsulatum

4. Klebsilla aerogenes

5.

Correct Answer: 1.

Question 45: Woolsorter disease is

1. Pneumonic form of anthrax

2. Pneumonic plague

3. Hydatid disease of lungs

4. A form of psittacoccis

5.

Correct Answer: 1.

Question 46: CAUSATIVE ORGANISM OF SOFT CHANCRE

1. H.DUCREYI

2. SYPHILIS

3. NEISSERIA

4. DONOVANOSIS

5.

Correct Answer: 1.

Question 47: pH of sabouraud’s dextrose agar is adjusted to

1. 1-2

2. 4-6

3. 6-8

4. 8-10

5.

Correct Answer: 2.

Question 48: Heat stable enterotoxin causing food poisoning is caused by all the following except

1. Bacillus cereus

2. Yersinia enterocolitica

3. Staphylococcus aureus

4. Clostridium perfringens

5.

Correct Answer: 4.

Question 49: Type IV hypersensitivity to Mycobacterium tuberculosis antigen may manifest as:

Page 98: FMGE (MCI Screening test)  preparation material by Gankidi Raghavender Reddy

1. Iridocylitis.

2. Polyarteritis nodosa

3. Phlyctenular conjunctivitis

4. Giant cell arteritis

5.

Correct Answer: 3.

Question 50: Which of the following is true about autoclaving

1. It kills by denaturation of proteins

2. Presence of air is good for the killing of organisms

3. It is best for oils and jellys

4. Kills all organisms except HIV

5.

Correct Answer: 1.

Question 51: A labourer involved with repair-work of sewers was admitted with fever, jaundice and renal failure. The most

appropriate test to diagnose the infection in this patient is

1. Weil Felix test

2. Paul Bunnel test

3. Microscopic agglutination test

4. Micro immunofluorescence test

5.

Correct Answer: 3.

Question 52: wchich is wrong about Shigella

1. nonmotile

2. capsulated

3. nonsporing

4. Fimbriae may present

5. all of the above

Correct Answer: 2.

Question 53: Virus mediated transfer of host DNA from one cell to another is known as

1. Transduction

2. Transformation

3. Transcription

4. Integration

5.

Correct Answer: 1.

Question 54: PAINLESS GENITAL ULCERS ARE SEEN IN

1. HERPES GENITALIS

2. LYMPHOGRANULOMA VEVNEREUM

3. GONORRHOEA

Page 99: FMGE (MCI Screening test)  preparation material by Gankidi Raghavender Reddy

4. SYPHILIS

5. CHANCRE

Correct Answer: 4.

Question 55: Most of the drug resistance occurs due to

1. Transduction

2. Translation

3. Mutation

4. Conjugation

5.

Correct Answer: 4.

Question 56: The discovery of gene transformation came from study of one of the following bacteria

1. Bacillus subtilis

2. Streptococcus pyogenes

3. Streptococcus pneumoniae

4. E. coli

5.

Correct Answer: 3.

Question 57: Cutting instruments like knives and scissors should be sterilized by

1. Boiling in water

2. Heating

3. Detergents

4. Carbolic

5.

Correct Answer: 1.

Question 58: Which of the following is not a Staphylococcus species

1. Aureus

2. Albicans

3. Fragilis

4. Epidermidis

5.

Correct Answer: 3.

Question 59: Which of the following is caused by E.coli infection

1. Urinary tract infection

2. Diarrhea

3. Pyogenic infection

4. Septicemia

5. all of the above

Correct Answer: 5.

Page 100: FMGE (MCI Screening test)  preparation material by Gankidi Raghavender Reddy

Question 60: Ring worm may be detected by

Ring worm may be detected by

1. infrared rays

2. X-rays

3. Ultraviolet rays

4. Cathode rays

5.

Correct Answer: 3.

Question 61: The most primitive mode of gene transfer occurs by

1. Transduction

2. Translation

3. Cell fusion

4. Conjugation

5.

Correct Answer: 4.

Question 62: Which of the following statements is true about rabies virus

1. It is a double stranded RNA virus

2. Contains a DNA dependant RNA polymerase

3. RNA has a negative polarity

4. Affects motor neurons

5.

Correct Answer: 3.

Question 63: Pigment producing colonies are seen in

1. Pseudomonas

2. Atypical mycobacteria

3. Serratia marcescens

4. All of the above

5.

Correct Answer: 4.

Question 64: Skin test for ecchinococcus is

1. Casoni test

2. Frei's test

3. Disk test

4. Kveim test

5.

Correct Answer: 1.

Question 65: Which of the following statements is true about endemic typhus

1. Is caused by R. rickettsii

2. Is transmitted by the bite of fleas

3. Has no mammalian reservoir

Page 101: FMGE (MCI Screening test)  preparation material by Gankidi Raghavender Reddy

4. Can be cultured in chemical defined culture medium

5.

Correct Answer: 2.

Question 66: Bacteria acquire characteristics by all of the following except

1. Through plasmids

2. Incorporating part of host DNA

3. Through bacteriophages

4. Through conjugation

5.

Correct Answer: 2.

Question 67: Drug used to treat ascariasis are all except.

1. Albendazole

2. Piperazine

3. Pyrantel pamoate

4. Ivermectin

5.

Correct Answer: 4.

Question 68: Mossy foot means

1. Zygomycoccis

2. Rhinosporidiasis

3. Chromoblastomycosis

4. Sporotrichosis

5.

Correct Answer: 3.

Question 69: Most common cause of infective endocarditis

1. Staphylococcus aureus

2. Streptococcus viridians

3. Streptococcus pyogens

4. Staphylococcus albus

5.

Correct Answer: 2.

Question 70: All the following are true about Listeria monocytogens except

1. Transmitted by contaminated cheese

2. Gram negative bacteria

3. Causes abortion in pregnancy

4. Causes meningitis in neonates

5.

Correct Answer: 2.

Question 71: Which virus is most notorious for causing a chronic hepatitis evolving cirrhosis

Page 102: FMGE (MCI Screening test)  preparation material by Gankidi Raghavender Reddy

1. Hepatitis C virus

2. Hepatitis B virus

3. Hepatitis E virus

4. Cytomegalovirus

5.

Correct Answer: 1.

Question 72: Appendicitis like syndrome is caused by

1. Brucella abortis

2. Yersinia tuberculosis

3. Pasturella multicoda

4. Yersenia pestis

5.

Correct Answer: 3.

Question 73: All the following are transmitted by blood transfusion except

1. Hepatitis G

2. Cytomegalovirus

3. Dengue

4. Parvovirus B 19

5.

Correct Answer: 3.

Question 74: A chest physician performs bronchoscopy in the procedure room of the out patient department. To make the

instrument safe for use in the next patient waiting outside. The most appropriate method to disinfect the endoscope is by

1. 70% alcohol for 5 min

2. 2% gluteraldelyde for 20 min

3. 2% formaldehyde for 10 min

4. 1% sodium hypochlorite for 15 min

5.

Correct Answer: 2.

Question 75: Sulfur granules in actinomycosis are composed of

1. Organisms

2. Neutrophils and lymphocytes

3. Monocytes

4. Eosinophils

5.Correct Answer: 1.

Question 1: At the end of the normal puerperium the uterine weight is

1. 40 gm

2. 60 gm

3. 80 gm

Page 103: FMGE (MCI Screening test)  preparation material by Gankidi Raghavender Reddy

4. 100 gm

5.

Correct Answer: 2.

Question 2: The cause of 'Postpartum blue' is

1. Decreased estrogen

2. Decreased progesterone

3. Increased prolactin

4. Decreased estrogen and progesterone

5.

Correct Answer: 4.

Question 3: Which one of the following is not an indication for IVF-ET

1. Tubal disease

2. Severe endometriosis

3. Impotence in the male

4. Unexplained infertility

5.

Correct Answer: 2.

Question 4: All occurs in abruptio placentae except

1. Coagulation failure

2. Renal failure

3. Uterine inversion

4. Intrauterine death

5.

Correct Answer: 3.

Question 5: The severity of pelvic pain in endometriosis correlates best with

1. Number of implants

2. Depth of invasion

3. Stage of disease

4. Ca 125 levels

5.

Correct Answer: 2.

Question 6: Elderly woman with recurrent swelling and pain in vulve is due to

1. Hydradenitis

2. Bartholin cyst

3. Hematoma

4. Sebaceous cyst

5.

Correct Answer: 2.

Question 7: Caesarean section has to be done in which of the following conditions

Page 104: FMGE (MCI Screening test)  preparation material by Gankidi Raghavender Reddy

1. Previous LSCS for transverse lie

2. Previous LSCS for ante partum haemorrhage

3. Previous LSCS for myomectomy scar 2 years back

4. Previous vesicovaginal repair

5.

Correct Answer: 4.

Question 8: All are true regarding episiotomy except

1. Increases the size of the birth canal

2. Midline episiotomy causes less bleeding and heals early

3. Involvement of rectum is classified as grade 3/ 4

4. Mediolateral episiotomy is most preferred one

5.

Correct Answer: 2.

Question 9: The most common site of ureteric injury during Wertheim's hysterectomy is

1. At the level of infundibulopelvic ligament

2. Where the uterine artery crosses the urter form above

3. Where ureter traverses through the smuculature of the bladder

4. In the ovarin fossa

5.

Correct Answer: 1.

Question 10: In a patient with peivic inflammatory disease due to tuberculosis, all of the following statements are true

except

1. Mycobacterium can be grown from menstrual blood

2. Associated with infertility

3. Ectopic pregnancy is common

4. Dysmenorrhea is a common presentation

5.

Correct Answer: 4.

Question 11: Which is not ture rearding oral contraceptive pills

1. Interact with Rifampicin

2. Prevent benign breast disease

3. Prevent PID

4. Increase the severity of rheumatoid arthritis

5.

Correct Answer: 4.

Question 12: Ligation of anterior division of internal iliac artery stops intractable pelvic haemorrhage because of

1. Lack of collateral circulation

2. Shuting off of blood flow

3. Decrease in arterial pulse pressure

Page 105: FMGE (MCI Screening test)  preparation material by Gankidi Raghavender Reddy

4. Increased capillary clotting

5.

Correct Answer: 3.

Question 13: A 47-year-old woman has achieved a pregnancy via in vitro fertilization (IVF) using donor eggs from a 21-

year-old donor and sperm from her 46-year-old husband. She has a sonogram performed at 7 to 12 weeks gestational age

that shows a quintuplet pregnancy. A 5-mm nuchal translucency is discovered in one of the embryos. Implications of this

include which of the following?

a. b. c. d. e.

1. The embryo has a high risk of neural tube defect

2. The embryo has a high risk of cardiac malformation

3. The nuchal translucency will enlarge by 20 weeks

4. If the nuchal translucency resolves, the risk of a chromosome abnormality is comparable to that of other embryos

5. If the embryo is aneuploid, the most likely diagnosis is Turner syndrome

Correct Answer: 2.

Question 14: All are absolute indications for cesarean section except

1. Placenta previa grade IV

2. Abruptio placenta

3. Carcinoma cervix Stage IB

4. Active herpetic lesion

5.

Correct Answer: 2.

Question 15: Ectopic pregnancy is differentiated from abortion by the fact that in ectopic pregnancy

1. There is slight amount of bleeding

2. Pain appears after vaginal bleeding

3. No enlargement of uterus

4. Histological examination of products of expulsion shows villi

5.

Correct Answer: 1.

Question 16: B Lynch suture is applied on

1. Cervix

2. Fallopian tubes

3. Ovaries

4. Uterus

5.

Correct Answer: 4.

Question 17: A patient with choriocarcinoma, is not responding to the usual doses of methotrexate. She is having

jaundice. Which drug will you prefer for further treatment

1. Actinomycin D

2. Cyclophosphamide

3. Cisplatin

Page 106: FMGE (MCI Screening test)  preparation material by Gankidi Raghavender Reddy

4. Chlorambucil

5.

Correct Answer: 1.

Question 18: Therapeutic level of magnesium in serum for treatment of eclampsia is

1. 2-7 mEq/L

2. 9-12 mEq/L

3. 15-17 mEq/L

4. 30-35 mEq/L

5.

Correct Answer: 2.

Question 19: Laproscopic sterlization is contraindicated in all except

1. Elderly primi

2. Major disproportion

3. Minor disproportion

4. Severe PET

5.

Correct Answer: 3.

Question 20: A G2P1+0+0, Diabetic mother presents at 32 weeks of pregnancy. She gives a history of full term fetal

demise during her last pregnancy. Her sugar is now controlled and the child is stable. What is the best plan of action

1. Await spontaneous delivery

2. Induce at 38 weeks

3. Induce at 40 weeks

4. Cesarian section at 38 weeks

5.

Correct Answer: 1.

Question 21: Which one of the following statements is not correct regarding RU 486 used for MTP

1. It is a 19 -nortestosterone derivative

2. It is effective up to 12 weeks of pregnancy

3. It acts as an inhibitor of progesterone-receptor in the endometrium

4. A single dose of 200 mg is given orally

5.

Correct Answer: 4.

Question 22: According to MTP Act, 2 doctor's opinion is required when pregnancy is

1. >12 weeks

2. >10 weeks

3. >20 weeks

4. >8 weeks

5.

Correct Answer: 1.

Page 107: FMGE (MCI Screening test)  preparation material by Gankidi Raghavender Reddy

Question 23: Controlled ovarian hyperstimulation as a treatment for infertility is most useful in which patient

1. Couples undergoing artificial insemination

2. Women undergoing cancer therapy

3. Women with premature ovarian failure

4. Women with fibroid uterus

5.

Correct Answer: 1.

Question 24: A patient presents with menorrhagia. All the following are possible treatment modalities except

1. NSAIDs

2. Tranexamic acid

3. Progesterone

4. Clomiphene

5.

Correct Answer: 4.

Question 25: For a menopausal patient having hot flushes, which of the following can be given as treatment

1. Ethinyl estradiol

2. Progesterone

3. Gonadotropin

4. Danazol

5.

Correct Answer: 1.

Question 26: A 24-year-old woman has had three first-trimester spontaneous abortions. Which of the following statements

concerning chromosomal aberrations in abortions is true?

1. 45,X is more prevalent in chromosomally abnormal term babies than in abortus products

2. Approximately 20% of first-trimester spontaneous abortions have chromosomal abnormalities

3. Trisomy 21 is the most common trisomy in abortuses

4. Despite the relatively high frequency of Down syndrome at term, most Down fetuses abort spontaneously

5. Stillbirths have twice the incidence of chromosomal abnormalities as live births

Correct Answer: 4.

Question 27: Treatment for 32 ys. old multipara with dysfunctional uterine bleeding (DUB) is

1. Danazol

2. Prostaglandins

3. Endometrial ablation

4. Progesterones

5.

Correct Answer: 4.

Question 28: Following a vaginal delivery, a woman develops a fever, lower abdominal pain and ulterine tenderness. She

is alert, and her blood pressure and urine output are good. Large gram positive rods suggestive of clostridia are seen in a

smear of cervix. management should include all except

Page 108: FMGE (MCI Screening test)  preparation material by Gankidi Raghavender Reddy

1. Immediate radiographic examination for gas in uterus

2. High dose antibiotic therapy

3. Hysterectomy

4. Close observation for renal failure or hemolysis

5.

Correct Answer: 3.

Question 29: Which one of the following lymph nodes is not involved in carcinoma cervix

1. Inguinal

2. Paramertrial

3. Obturator

4. Hypogastric

5.

Correct Answer: 1.

Question 30: Treatment of choice for carcinoma cervix IIb in a 35-year old woman is

1. Radical surgery

2. Radical radiotherapy

3. Radical radiotherapy followed by simple hysterectomy

4. Chemotherapy followed by radical surgery

5.

Correct Answer: 3.

Question 31: Current modes of investigation for infertility to check functioning of tubes are all of the following execpt

1. Air insufflation

2. Sonosalpingography

3. Hysterrosalpingography

4. Laparoscopic chromotubation

5.

Correct Answer: 1.

Question 32: A hypertensive pregnant woman at 34 weeks comes with history of pain in abdomen, bleeding per vaginum

and loss of fetal movements. On examinationthe uterus is contracted with increased uterine tone. Fetal heart sounds are

absent. The most likely diagosis is

1. Placenta previa

2. Hydramnios

3. Premature labour

4. Abruptio placenta

5.

Correct Answer: 4.

Question 33: A patient, Shalu, presents with vaginal discharge. Examination of the discharge reveals the presence of

Chlamydial infection. The treatment of choice is

1. Azithromycin + contact tracing

2. Doxycycline + Metronidazole

Page 109: FMGE (MCI Screening test)  preparation material by Gankidi Raghavender Reddy

3. Fluconazole + Doxycycline

4. Metronidazole

5.

Correct Answer: 1.

Question 34: A 35 year old woman with dysmenorrhea and menorrhagia of 6 months duration showed an enlarged uterus

of 20 weeks which was tender, the possible diagnosis is :

1. Adenomyosis

2. Fibroid

3. Carcinoma endometrium

4. PID

5.

Correct Answer: 1.

Question 35: Which one is not true regarding Centchroman

1. It is anti estrogenic

2. It acts on the endometrium

3. It is a synthetic hormone

4. It is developed in India

5.

Correct Answer: 3.

Question 36: In prenatal diagnostic technique Act 1994 which one of the following is not a ground for carrying out

prenatal test

1. Pregnant women above 35 years of age

2. History of two or more spontaneous abortion of fetal loss.

3. When fetal heart rate is 160 per min at fifth and 120 per min at ninth month

4. History of exposure to potentially teratogenic drugs

5.

Correct Answer: 3.

Question 37: A patient with infertility and PID was investigated and was found in hysterosalpingogram to have beaded

fallopian tubes with clubbing of the ampullary end. The diagnosis is most likely to be

1. Gonococcus

2. Chlamydia

3. Tuberculosis

4. Ureaplasma

5.

Correct Answer: 3.

Question 38: Clinical features of uterine fibroid include all except

1. Menorrhagia

2. Sub fertility

3. Recurrent spontaneous abortion

4. Constipation

5.

Page 110: FMGE (MCI Screening test)  preparation material by Gankidi Raghavender Reddy

Correct Answer: 4.

Question 39: Treatment of single large fibroid in a 45 years old female

1. Hysterectomy

2. Myomectomy

3. Observe till menopause

4. Oral pills

5.

Correct Answer: 1.

Question 40: In a patient suspected to have polycystic ovarian disease, LH and FSH should be examined in which part of

the menstrual cycle

1. 1-3 days

2. 8-10 days

3. 13-16 days

4. 24-28 days

5.

Correct Answer: 3.

Question 41: Fetal heart activity can be detected by sonography at about

1. 5 weeks

2. 6 weeks

3. 7 weeks

4. 8 weeks

5.

Correct Answer: 3.

Question 42: A 25-year-old G3P0 presents for preconception counseling. She has had three first-trimester pregnancy

losses. As part of her evaluation for recurrent abortion, she had karyotyping done on herself and her husband. Her husband

is 46,XY. She was found to carry a balanced 13;13 translocation. What is the likelihood that her next baby will have an

abnormal karyotype?

a. <5% b. 10% c. 25% d. 50% e. 100%

1. <5%

2. 10%

3. 25%

4. 50%

5. 100%

Correct Answer: 3.

Question 43: Regarding cholestasis in pregnancy, all the following statements are true except

1. Bilirubin more than 5 mg/dL

2. SGPT/SGOT ratio less than 60 units

3. Presence of severe itching

4. Alkaline phosphatase is increased maximally

5.

Page 111: FMGE (MCI Screening test)  preparation material by Gankidi Raghavender Reddy

Correct Answer: 1.

Question 44: Cause of decubitus ulcer in uterine proplase is

1. Friction

2. Venous congestion

3. Sexual act

4. Trauma

5.

Correct Answer: 2.

Question 45: Commonest mode of contraception in INDIA

1. Intra uterine device

2. Condoms

3. Sterilization

4. Oral contraceptive pills(OCP)

5.

Correct Answer: 3.

Question 46: Ectopic pregnancy is seen maximum with

1. IUCD

2. OC pills

3. Barrier method

4. Tubal insertion

5.

Correct Answer: 1.

Question 47: Treatment of choice for carcinoma cervix IIb in a 35-year old woman is

1. Radical surgery

2. Radical radiotherapy

3. Radical radiotherapy followed by simple hysterectomy

4. Chemotherapy followed by radical surgery

5.

Correct Answer: 3.

Question 48: Which one of the following is NOT used in the treatment of poor postcoital test in infertility

1. Condom therapy

2. Estrogen in mid-cycle

3. Intrauterine insemination

4. Antioxidant therapy to male

5.

Correct Answer: 2.

Question 49: The commonest site of ectopic gestation

1. Ovary

Page 112: FMGE (MCI Screening test)  preparation material by Gankidi Raghavender Reddy

2. Broad ligament

3. Fallopian tube

4. Cervix

5.

Correct Answer: 3.

Question 50: The commonest type of myoma in the body of uterus is

1. Intramural

2. Submucous

3. Subserous

4. Cervical

5.

Correct Answer: 1.

Question 51: Endometrial carcinoma is predisposed to by all the following except

1. Ethinyl estradiol

2. Tamoxifen

3. Oral contraceptives

4. Nulliparity

5.

Correct Answer: 3.

Question 52: Medical treatment for endometriosis includes the following, except

1. Gn RH analogues

2. Progestins

3. Corticosteroids

4. Danazol

5.

Correct Answer: 3.

Question 53: The most common causes for female pseudohermaphroditism is

1. Ovarian dysgenesis

2. Congenital adrenal hyperplasia

3. Virilising ovarian tumors

4. Exogenous androgen

5.

Correct Answer: 2.

Question 54: Degeneration of the myomata is more likely to start from

1. Centre

2. Periphery

3. From any portion

4. None of the above

5.

Correct Answer: 1.

Page 113: FMGE (MCI Screening test)  preparation material by Gankidi Raghavender Reddy

Question 55: All of the following are known risk factors for development of endometrial carcinoma except

1. Obesity

2. Early Menopause

3. Use of Hormone Replacement Therapy

4. Family History

5.

Correct Answer: 2.

Question 56: Red degeneration of fibroid is associated with all except

1. Pregnancy

2. Aseptic infraction

3. Thrombosis

4. Leukocytosis

5.

Correct Answer: 4.

Question 57: Type of pelvis in which deep transverse arrest occurs is

1. Android

2. Anthropoid

3. Gynecoid

4. Platy pelloid

5.

Correct Answer: 1.

Question 58: A primigravida at 37 week of gestation reported to labour room with central placenta praevia with heavy

bleeding per vaginum. The fetal heart rate was normal at the time of examination. The best management option for her is

1. Expectant management

2. Caesarean section

3. Induction and vaginal delivery

4. Induction and forceps delivery

5.

Correct Answer: 2.

Question 59: All of the following drugs are used for management of post partum haemorrhage except

1. Mifepristone (RU-486)

2. Misoprostol

3. Prostaglandin.

4. Oxytocin

5.

Correct Answer: 1.

Question 60: A young lady with spontaneous abortions has a history of joint pains and fever. She currently presents with

thrombosis of her leg vein. Her APTT is prolonged. The diagnosis is most likely to be

1. Inherited protein C and S deficit

Page 114: FMGE (MCI Screening test)  preparation material by Gankidi Raghavender Reddy

2. Factor XII deficiency

3. Antiphospholipid antibody syndrome

4. Increased antithrombin III levels

5.

Correct Answer: 3.

Question 61: Pyometra is a complication associated with all of the following conditions except

1. Carcinoma of the vulva

2. Carcinoma of the cervix.

3. Carcinoma of endometrium

4. Pelvic radiotherapy

5.

Correct Answer: 1.

Question 62: All are true about cephalhematoma except

1. Occurs due to subcutaneous edema

2. Seen mostly over occipital bone

3. Treatment is by aspiration

4. Varies in size when the child cries

5.

Correct Answer: 4.

Question 63: The most common major complication with Laparoscopic hysterectomy is

1. Pulmonary embolus

2. Uncontrolled bleeding

3. Bowel injury

4. Urinary tract injury

5.

Correct Answer: 4.

Question 64: A multiparous woman aged 40 years, presents with menorahagia and progressively increasing

dysmenorrhoea. Most probable diagnosis is

1. Ca Cervix

2. Ca Endometrium

3. Adenomyosis

4. DUB

5.

Correct Answer: 3.

Question 65: Medical treatment for endometriosis includes the following, except

1. Progestins

2. Corticosteroids

3. Danazol

4. GnRH

5.

Page 115: FMGE (MCI Screening test)  preparation material by Gankidi Raghavender Reddy

Correct Answer: 3.

Question 66: Red degeneration of fibroid is associated with

1. Postpartum

2. 3rd trimester

3. 2nd trimester

4. 1st trimester

5.

Correct Answer: 2.

Question 67: While performing a fractional curettage for perimenopausal bleeding which procedure is performed first

1. Endocervical curettage

2. Uterine sounding

3. Exocervical punch biopsy

4. Dilatation of the cervix

5.

Correct Answer: 1.

Question 68: Which one of the following is true regarding median episiotomy

1. Blood loss is more

2. More likely to extend to the anal sphincter

3. Anatomical apposition is poor

4. There is increased incidence of dyspareunia

5.

Correct Answer: 2.

Question 69: The data are most convincing for which theory as the pathogenesis of endometriosis in the peritoneal cavity

1. Embryonic rests

2. Lymphatic & vascular metastasis

3. Induction

4. Coelomic metaplasia

5.

Correct Answer: 1.

Question 70: Drug used in last stage of endometrial carcinoma is

1. Danazol

2. O.C. Pills

3. Cisplatin

4. Levonorgestrol

5.

Correct Answer: 3.

Question 71: Which one of the following is the most common problem associated with the use of condom

1. Increased monilial infection of vagina

Page 116: FMGE (MCI Screening test)  preparation material by Gankidi Raghavender Reddy

2. Premature ejaculation

3. Contact dermatitis

4. Retention of urine

5.

Correct Answer: 3.

Question 72: In a case of ectopic pregnancy, medical treatment is contraindicated if

1. Sac size is 3.0 cm

2. Serum HCG levels > 1500 miu / ml

3. Significant hemoperitoneum is present

4. Absent fetal activity

5.

Correct Answer: 3.

Question 73: Amoniotic fluid at 38 weeks in normal pregnancy is

1. 800 cc

2. 1100 cc

3. 1400 cc

4. 1800 cc

5.

Correct Answer: 1.

Question 74: Most common cause of ectopic pregnancy is

1. IUCD

2. Previous tubal disease

3. Endometriosis

4. Mini pills

5.

Correct Answer: 2.

Question 75: During the first stage of labour the membranes ruptured and the umbilical cord prolapsed. Foetal heart rate

is 120-130 per minute; cervix is about 5 cm dilated .Which one of the following will be the appropriate treatment

1. Repose cord and raise foot end of bed

2. Repose cord and apply ventouse

3. Lower segment caesarean section

4. Forceps delivery under general anesthesia

5.

Correct Answer: 3.

Question 76: Medical treatment of ectopic gestation involves use of all the following drugs except

1. Prostaglandins

2. RU-486

3. Methotrexate

4. Dexamethasone

5.

Page 117: FMGE (MCI Screening test)  preparation material by Gankidi Raghavender Reddy

Correct Answer: 4.

Question 77: The most common side effect of injectable hormonal contaceptive is

1. Bone density loss

2. Disruption of menstrual pattern

3. Risk of breast cancer

4. Dysmennorrhoea

5.

Correct Answer: 2.

Question 78: All are interceptive contraception except

1. O.C. pills

2. Centchromen

3. Levonorgestrol

4. Mifepristone

5.

Correct Answer: 2.

Question 79: A lady with IUCD becomes pregnant with tail of IUCD being seen next course of action is

1. MTP

2. Remove the IUCD

3. Continue the pregnancy

4. Remove IUCD and terminate pregnancy

5.

Correct Answer: 2.

Question 80: Which of the following is seen in infant of a diabetic mother

1. Hypercalcemia

2. Hyperglycemia

3. Increased blood viscosity

4. Hyperkalemia

5.

Correct Answer: 3.

Question 81: All of the following are the indications for Myomectomy in a case of fibroid uterus, EXCEPT

1. Associated infertility

2. Recurrent pregnancy loss

3. Pressure symptoms

4. Red degeneration

5.

Correct Answer: 4.

Question 82: A 50 year old woman, nulliparous, diabetic and obese presenting with post-menopausal bleeding can be any

of the following except

Page 118: FMGE (MCI Screening test)  preparation material by Gankidi Raghavender Reddy

1. Carcinoma in situ of cervix

2. Carcinoma endometrium

3. DUB

4. None of the above

5.

Correct Answer: 1.

Question 83: Hormone Replacement Therapy is contraindicated in

1. Atherosclerosis

2. Osteoporosis

3. Urogenital atrophy

4. Venous thromboembolism

5.

Correct Answer: 4.

Question 84: A 30 year old female was brought to the OPD on the 4th day after deliver. She was showing mood changes,

irritability, fearfullness and anxiety about her baby, had fight with relatives and remained tearful. She is suffering most

probably from :

1. Postpartum depression

2. Postnatal blues

3. Puerperal psychosis

4. None of the above

5.

Correct Answer: 2.

Question 85: Which one is not true regarding Centchroman

1. It is ant estrogenic

2. It acts on the endometrium

3. It is a synthetic hormone

4. It is developed in India

5.

Correct Answer: 3.

Question 86: Ectopic pregnancy is managed conservatively by all except

1. Methotrexate

2. Salphingostomy

3. Salphingectomy

4. Segmental resection

5.

Correct Answer: 3.

Question 87: The following parameters are included in 'Foetal Biophysical Profile ',except

1. Placental calcification

2. Amoniotic fluid volume

3. Non -stress test

Page 119: FMGE (MCI Screening test)  preparation material by Gankidi Raghavender Reddy

4. Foetal breathing movement

5.

Correct Answer: 1.

Question 88: A patient on OC pills intake develops psychiatric symptoms, and abdominal pain. Diagnosis is

1. Acute intermittent porphyria

2. Systemic lupus

3. Thromobosis

4. Anemia

5.

Correct Answer: 1.

Question 89: Which of the following regarding Ectopic pregnancy is true

1. Occurs in about 10% of pregnancies

2. The risk is increased in those with a history of PID

3. Usually presents between 2 and 4 months of gestation

4. Patients usually have a negative pregnancy test

5.

Correct Answer: 2.

Question 90: The patient with pre-eclampsia diagnosed remote from term, which of the following is NOT necessarily an

indication for expeditious delivery

1. Blood Pressure 170/115 on medication

2. Proteinuria 5g per 24 hours

3. Platelet count 80,000/ul

4. Serum transaminase levels three times normal

5.

Correct Answer: 2.

Question 91: Preterm labour is not treated by

1. Nifedipine

2. MgSO4

3. Nitrous oxide patch

4. Misoprostol

5.

Correct Answer: 4.

Question 92: Fothergill's repair for prolapsed uterus will not lead to

1. First trimester abortions

2. Cervical distocia

3. Premature labor

4. Premature rupture of membrane

5.

Correct Answer: 1.

Page 120: FMGE (MCI Screening test)  preparation material by Gankidi Raghavender Reddy

Question 93: Laporatomy performed in a case of ovarian tumor revealed unilateral ovarian tumor with ascities positive for

malignatn cells and positive pelvic lyjph nodes. All other structures were free of disease. what is stage of the disease

1. Stage IIc.

2. Stage IIIa

3. Stage IIIb

4. Stage IIIc

5.

Correct Answer: 4.

Question 94: Which of the follwing is responsible for inability to rotate anteriorly in the occipitoposterior position

1. Moderate size fetus

2. Gynecoid pelvis

3. Weak uterine contractions

4. Good levator ani muscle contractions

5.

Correct Answer: 3.

Question 95: A Para II poorly compensated cardiac patient has delivered 2 days back. You will advise her to

1. Undergo sterilization (tubectomy) after 1 week

2. Undergo sterilization after 6 weeks

3. Suggest her husband to undergo vasectomy

4. Take oral contraceptive pills after 6 months

5.

Correct Answer: 3.

Question 96: Decubitus ulcer is produced due to

1. Congestion

2. Inflammation

3. Malignancy

4. Dilatation

5.

Correct Answer: 1.

Question 97: Most common cause of first trimester abortion is

1. Chromosomal abnormalities

2. Syphilis

3. Cervical incompetence

4. Rhesus isoimmunisation

5.

Correct Answer: 1.

Question 98: The following drugs can beused to suppres lactation, except

1. Carbergoline

2. High dose estrogens

3. Metoclopramide

Page 121: FMGE (MCI Screening test)  preparation material by Gankidi Raghavender Reddy

4. Pyridoxine

5.

Correct Answer: 3.

Question 99: The severity of pelvic pain in endometriosis correlates best with

1. Number of implants

2. Depth of invasion

3. Stage of disease

4. Ca 125 levels

5.

Correct Answer: 2.

Question 100: A 30 yrs. old multigravida presented with transverse lie with hand prolapse in IInd stage of labour with

dead fetus. The treatment is

1. Chemical caesarean section

2. Craniotomy

3. Cleidotomy

4. Decapitation

5.Correct Answer: 4.

Question 1: Which drug is used in the management of acute angle closure glaucoma

1. IV mannitol

2. Phenylephrine

3. Atenolol

4. Nifedipine

5.

Correct Answer: 1.

Question 2: Varicella infection in pregnancy can lead to following eye problems

1. Microphthalmia,

2. Cataracts

3. Chorioretinitis

4. Optic atrophy

5. All of the above

Correct Answer: 5.

Question 3: All of the following are symptoms of early primary open angle glaucoma except

1. Coloured halos

2. Headache occasional

3. Decreased fields of vision

4. Defective vision

5.

Page 122: FMGE (MCI Screening test)  preparation material by Gankidi Raghavender Reddy

Correct Answer: 4.

Question 4: Exudative retinal detachment is seen in

1. Coat's disease

2. Vogt koyanagi Harada disease

3. Choroidal hemangioma

4. All of the above

5.

Correct Answer: 4.

Question 5: Following are features of retinitis pigmentosa except

1. Bony spicules in fundus

2. Consecutive optic atrophy

3. Nyctalopia

4. Early Childhood blindness

5.

Correct Answer: 4.

Question 6: Which is the most common cause of infectious uveitis in India

1. TB

2. Sarcoidosis

3. Idiopathic

4. Drug induced

5.

Correct Answer: 1.

Question 7: Fuch's cyclitis is characterised by all except

1. Heterochromia

2. Small white keratic precipitates

3. Relatively white eye with absence of posterior synechiae

4. Red eye with severe pain

5.

Correct Answer: 4.

Question 8: All of the following are causes of complicated cataract except

1. Myopia

2. Uveitis

3. RP

4. Hyperopia

5.

Correct Answer: 4.

Question 9: Which of the following surgery is not done as a part of cataract management?

1. Phaco

Page 123: FMGE (MCI Screening test)  preparation material by Gankidi Raghavender Reddy

2. ECCE

3. SICS

4. Trabeculectomy

5.

Correct Answer: 4.

Question 10: Prominent corneal nerves are seen in all except

1. Keratoconus

2. Leprosy

3. Von Recklinghausen's disease

4. Hyperopia

5.

Correct Answer: 4.

Question 11: To maintain the transperancy of cornea, the endothelial cell density should be at least

1. 500 cells/sq.mm

2. 1500 cells/sq.mm

3. 2500 cells/sq.mm

4. 3500 cells/sq.mm

5.

Correct Answer: 1.

Question 12: Presbyopia is

1. Insufficiency of accomodation,

2. Age related decrease in the power of ciliary muscles

3. Decrease in elasticity and plasticity of the crystalline lens

4. All of the above

5.

Correct Answer: 4.

Question 13: Following are the human diseases related to cat

1. Toxoplasmosis

2. Cat scratch fever

3. Leptospirosis

4. 1 & 2 are correct

5.

Correct Answer: 4.

Question 14: Following drugs are used in the treatment of glaucoma except

1. Diamox

2. Oral glycerol

3. Timolol

4. Cisapride

5.

Correct Answer: 4.

Page 124: FMGE (MCI Screening test)  preparation material by Gankidi Raghavender Reddy

Question 15: Automated perimetry is a test used for knowing

1. Visual fields

2. Corneal thickness

3. Corneal diameter

4. AC depth

5.

Correct Answer: 1.

Question 16: An elderly male with heart disease has sudden loss of vision in one eye, examination reveals a cherry red

spot; diagnosis

1. Central retinal vein occlusion

2. Central retinal artery occlusion

3. Amaurosis fugax

4. Acute ischemic optic neuritis

5.

Correct Answer: 2.

Question 17: Herpes zoster ophthalmicus is predictor of all except

1. Leukemia

2. Lymphoma

3. HIV

4. Metastasis

5.

Correct Answer: 4.

Question 18: Central serous retinopathy is treated with all except

1. Antianxiety drugs

2. Reassurance & relaxation methods

3. Focal laser

4. Oral steroids

5.

Correct Answer: 4.

Question 19: Central serous retinopathy is related to all of the following except

1. Stress

2. Steroid intake

3. Diabetes

4.

5.

Correct Answer: 3.

Question 20: Following are indications for use of topical beta blocker in Ophthalmology

1. Primary open angle glaucoma

2. Primary narrow angle glaucoma with acute attack

Page 125: FMGE (MCI Screening test)  preparation material by Gankidi Raghavender Reddy

3. Postoperative intraocular pressure rise

4. All are correct

5.

Correct Answer: 4.

Question 21: All of the following are topical antiglaucoma drugs except

1. Timolol

2. Latanoprost

3. Alphagan

4. Acetaminophen

5.

Correct Answer: 4.

Question 22: The following is a component of visual perception

1. Color Vision

2. Texture Vision

3. Eye Movement

4. All of the above

5.

Correct Answer: 4.

Question 23: Glaucoma affects

1. Lens

2. Optic nerve

3. Conjunctiva

4. Sclera

5.

Correct Answer: 2.

Question 24: Common complications of IOL lens are except

1. Corneal dystrophy

2. Glaucoma

3. Anisocoria

4. Mascular edema

5.

Correct Answer: 3.

Question 25: A child has got a congenital catract involving the visual axis which was detected by the parents right at

birth. This child should be operated

1. Immediately

2. At 2 months of age

3. At 1 year of age when the globe becomes normal sized

4. After 4 year when entire ocular and orbital growth become normal

5.

Correct Answer: 4.

Page 126: FMGE (MCI Screening test)  preparation material by Gankidi Raghavender Reddy

Question 26: Intermediate uveitis is characterised by

1. Relatively white eye

2. Pars plana exudates

3. Only 2 is correct

4. Both are correct

5.

Correct Answer: 4.

Question 27: Image in Direct opthalmoscopy is

1. Virtual, Inverted magnified

2. Real, Inverted Magnified

3. Real, Erect and Magnified

4. Virtual, Erect and Magnified

5.

Correct Answer: 4.

Question 28: Congenital rubella syndrome is characterised by

1. Congenital cataract

2. Deafness

3. Cardiac defects

4. All of the above

5.

Correct Answer: 4.

Question 29: Toxoplasmosis affects all except

1. Retina

2. Choroid

3. Sclera

4. None of the above

5.

Correct Answer: 3.

Question 30: What are the ocular side effects of steroids

1. Cataract

2. Glaucoma

3. Increased chances of infection due to decrease in immunity

4. Al of the above

5.

Correct Answer: 4.

Question 31: All of the following are used to dilate pupil except

1. Atropine

2. Homatropine

3. Phenylephrine

Page 127: FMGE (MCI Screening test)  preparation material by Gankidi Raghavender Reddy

4. Pilocarpine

5.

Correct Answer: 4.

Question 32: Following are the causes of scleritis except

1. TB

2. Rheumatoid arthritis

3. Collagen vascular disease

4. Gonorrhoea

5.

Correct Answer: 4.

Question 33: Enucleation is done in all except

1. Total anterior staphyloma

2. Absolute glaucoma

3. Panopthalmitis

4. Pthisis bulb

5.

Correct Answer: 1.

Question 34: Vitreous hemorrhage occurs commonly in

1. Proliferative diabetic retinopathy

2. Blunt Trauma

3. Eale's disease

4. All of the above

5.

Correct Answer: 4.

Question 35: Uhthoff's phenomenon is seen in which of the following condition

1. Myopia

2. Glaucoma

3. Macular scar

4. Multiple sclerosis

5.

Correct Answer: 4.

Question 36: Causes of tunnel vision

1. Advanced glaucoma

2. Advanced retinitis pigmentosa

3. 1 & 2 are correct

4. Only 2nd is correct

5.

Correct Answer: 3.

Question 37: What is meant by "oculus dexter"

Page 128: FMGE (MCI Screening test)  preparation material by Gankidi Raghavender Reddy

1. Right eye

2. Left eye

3. Both eyes

4. All of the above

5.

Correct Answer: 1.

Question 38: Reiter's syndrome is characterised by all except

1. Conjunctivitis

2. Uveitis

3. Arthritis

4. Glaucoma

5.

Correct Answer: 4.

Question 39: All are features of Horner’s syndrome except

1. Anhydrosis

2. Ptosis

3. Proptosis

4. Miosis

5.

Correct Answer: 3.

Question 40: All of the following are treatment options for central retinal occlusion except

1. Paracentesis

2. Ocular massage

3. Intravenous acetazolamide

4. IV ranitidine

5.

Correct Answer: 4.

Question 41: Neovascular glaucoma occurs commonly after

1. Central retinal vein occlusion

2. Central retinal artery occlusion

3. Trauma

4. Branch retinal artery occlusion

5.

Correct Answer: 1.

Question 42: Marfan's syndrome is characterised by following Ophthalmic feature

1. Subluxated lens

2. Myopia

3. Retinal degenerative changes with increased risk for RD

4. All of the above

5.

Page 129: FMGE (MCI Screening test)  preparation material by Gankidi Raghavender Reddy

Correct Answer: 4.

Question 43: Waxy pallor of optic disc is seen in

1. Retinitis pigmentosa

2. Gyrate atrophy

3. Fundus albipunctatus

4. High myopia

5.

Correct Answer: 1.

Question 44: Which of the following is not a part of glaucoma management

1. Phaco

2. Laser suturolysis

3. IV mannitol

4. Trabeculectomy

5.

Correct Answer: 1.

Question 45: Giant cell arteritis is also known as

1. Temporal arteritis

2. Temporal lobe syndrome

3. Takayasu arteritis

4. VHL disease

5.

Correct Answer: 1.

Question 46: Complicated cataract is seen in

1. Anterior Uveitis

2. Fuch's uveitis

3. Intermediate uveitis

4. All of the above

5.

Correct Answer: 4.

Question 47: Visual evoked potential in retrobulbar neuritis is

1. P100

2. P300

3. Wave I

4. P450

5.

Correct Answer: 4.

Question 48: Following are the protective factors for diabetic retinopathy progression except

1. High myopia

Page 130: FMGE (MCI Screening test)  preparation material by Gankidi Raghavender Reddy

2. Glaucoma

3. Retinitis pigmentosa

4. Cataract surgery

5.

Correct Answer: 4.

Question 49: Which of the following is not an Ophthalmic emergency

1. Open angle glaucoma

2. Acute attack of angle closure glaucoma

3. Central retinal artery occlusion

4. Alkali burn

5.

Correct Answer: 1.

Question 50: Retinal vasculitis is seen in

1. Ocular TB

2. Syphilis

3. Collagen vascular disease

4. Toxoplasmosis

5. Al of the above

Correct Answer: 5.

Question 51: Hereditary retinoblastomas develop the following choromosomal deletion

1. 13q 14

2. 13p14

3. 14p13

4. 14q13

5.

Correct Answer: 1.

Question 52: True about Acute congestive glaucoma are all except

1. Deep anterior chamber

2. Oval dilated pupil

3. Edematous cornea

4. Edematous optic disc

5.

Correct Answer: 1.

Question 53: Panretinal photocoagulation is indicated in all except

1. Branch retinal vein occlusion

2. Ischemic central retinal vein occlusion with iris neovascularisation

3. Ischemic central retinal vein occlusion with angle neovascularisation

4. Proliferative diabetic retinopathy

5.

Correct Answer: 1.

Page 131: FMGE (MCI Screening test)  preparation material by Gankidi Raghavender Reddy

Question 54: In all of the following conditions,steroids are used except

1. Uveitis

2. Central serous retinopathy

3. Postoperative inflammation

4. VKH syndrome

5.

Correct Answer: 2.

Question 55: All are commonly seen in a case of high myopia except

1. RD

2. Chorioretinal degeneration

3. Retinal break

4. Corneal dystrophy

5.

Correct Answer: 4.

Question 56: Which is the sutureless cataract surgery

1. ECCE

2. ICCE

3. Phaco

4.

5.

Correct Answer: 3.

Question 57: Which of the following is true regarding cortical blindness

1. Direct and consensual reflex absent in both the eyes

2. Direct and consensual reflexes are present in both the eyes

3. Direct reflex present and consensual reflex absent on the affected side

4. Direct reflex present and consensual absent on the normal side

5.

Correct Answer: 2.

Question 58: What is meant by "oculus sinister"

1. Right eye

2. Left eye

3. Both eyes

4. All of the above

5.

Correct Answer: 2.

Question 59: In myopia glasses used are

1. Convex

2. Concave

3. Cylindrical

Page 132: FMGE (MCI Screening test)  preparation material by Gankidi Raghavender Reddy

4. Elliptical

5.

Correct Answer: 2.

Question 60: Following are features of acute central retinal occlusion except

1. Retinal pallor with cloudy edema

2. Cherry red spot

3. Arterial attenuation

4. Neovascularisation in fundus

5.

Correct Answer: 4.

Question 61: In preoperative assessment of cataract patient following is to be seen

1. Axial length of cornea

2. Corneal thickness

3. Keratometry

4. Corneal curvature

5.

Correct Answer: 3.

Question 62: Rubeosis iridis is seen in

1. Advanced diabetic retinopathy

2. Ischemic Central retinal vein occlusion

3. Some cases of Eale's disease

4. All are correct

5. Only 1 & 2 are correct

Correct Answer: 4.

Question 63: True about Epiphora is

1. An epiphenomenon of ophthalmic allergy

2. Excess flow of tears due to blockade of nasolacrimal duct

3. Ectropion is the most common cause

4. Due to irritation of cornea by eyelashes

5.

Correct Answer: 2.

Question 64: Which is highly vascular structure in eye

1. Choroid

2. Cornea

3. Lens

4. Aqueous

5.

Correct Answer: 1.

Question 65: Crocodile tears are seen in

Page 133: FMGE (MCI Screening test)  preparation material by Gankidi Raghavender Reddy

1. Frey's syndrome

2. Conjunctivitis

3. Lacrimal tumour

4. Abnormal VII nerve regeneration

5.

Correct Answer: 4.

Question 66: Hemeralopia means

1. Night blindness

2. Day blindness

3. Constricted fields

4. Total blindness

5.

Correct Answer: 2.

Question 67: Kayser-Fleischer's ring is seen in

1. Wilson's disease

2. Keratoconus

3. Trauma

4. 1 & 2 are correct

5.

Correct Answer: 1.

Question 68: In Marfan's syndrome,lens is subluxated

1. Upwards & outwards

2. Inwards & downwards

3. Inwards & upwards

4. Downwards

5.

Correct Answer: 1.

Question 69: Following are the risk factors for progression of diabetic retinopathy except

1. Nephropathy

2. Hypertension

3. Cataract surgery

4. Glaucoma

5.

Correct Answer: 4.

Question 70: You have been referred a case of open angle glaucoma. Which of the following would be important point in

diagnosing the case?

1. Shallow anterior chamber

2. Optic disc cupping

3. Narrow angle

Page 134: FMGE (MCI Screening test)  preparation material by Gankidi Raghavender Reddy

4. Visual acuity and refractive error

5.

Correct Answer: 2.

Question 71: All of the following are used in treatment of primary angle closure glaucoma except

1. Intravenous mannitol

2. Acetazolamide

3. Timolol

4. Phenylephrine

5.

Correct Answer: 4.

Question 72: Proptosis is measured with

1. Hertel's exophthalmometer

2. Direct ophthalmoscope

3. Tonometer

4. Indirect ophthalmoscope

5.

Correct Answer: 1.

Question 73: Following are the features of Fuch's cyclitis

1. Complicated cataract

2. Absence of posterior synechiae

3. Keratic precipitates scattered over whole cornea

4. All of the above

5.

Correct Answer: 4.

Question 74: What happens to anterior chamber angle in some cases of trauma

1. Angle recession

2. Optic nerve comes in angle

3. Cornea comes in angle

4.

5.

Correct Answer: 1.

Question 75: Munson's sign is seen in

1. Keratoconus

2. Cataract

3. Glaucoma

4. RD

5.

Correct Answer: 1.

Question 76: Sturge Weber syndrome is characterised by all except

Page 135: FMGE (MCI Screening test)  preparation material by Gankidi Raghavender Reddy

1. Unilateral glaucoma

2. Choroidal hemangioma

3. Port wine stain

4. Complicated cataract

5.

Correct Answer: 4.

Question 77: A recurrent bilateral conjunctivitis occurring with the onset of hot weather in young boys with symptoms of

burning, itching, and lacrimation with polygonal raised areas in the palpebral conjunctiva is

1. Vernal kerato conjunctivitis

2. Mucopurulent conjunctivitis

3. Phlyctenular conjunctivitis

4. Trachoma

5.

Correct Answer: 1.

Question 78: All of the following are used in the treatment of fungal corneal ulcer except

1. Natamycin

2. Fluconazole

3. Ketoconazole

4. Acivir

5.

Correct Answer: 4.

Question 79: A patient using contact lens develops corneal infection. Laboratory diagnosis of acanthamoeba keratitis was

established. The following is the best drug for treatment

1. Propamidine

2. Neosporine

3. Ketocanazole

4. Polyhexamethylene biguanide

5.

Correct Answer: 1.

Question 80: Indications for intravenous methyl prednisolone in Ophthalmology are all except

1. Bilateral exudative retinal detachment

2. Optic neuritis

3. Anterior ischemic optic neuropathy in a case of giant cell arteritis

4. Post cataract surgery eye inflammation

5.

Correct Answer: 4.

Question 81: In which condition lens subluxation occurs

1. Marfan's syndrome

2. Weil Marchesani's syndrome

3. Blunt Trauma

Page 136: FMGE (MCI Screening test)  preparation material by Gankidi Raghavender Reddy

4. All of the above

5.

Correct Answer: 4.

Question 82: Munson's sign means

1. A bulge seen on lower lid in keratoconus when patient looks down

2. Oil droplet reflex

3. Lines seen on cornea

4. Pigment deposits in cornea

5.

Correct Answer: 1.

Question 83: Which of the following is not a feature of glaucoma

1. Visual field loss

2. Raised IOP

3. Optic nerve damage

4. Early Vision loss

5.

Correct Answer: 4.

Question 84: Subluxation (dislocation) of the crystalline lens in one or both eyes (ectopia lentis) in Marfan syndrome is

seen in

1. 80% of patients

2. 70 % of patients

3. 50 % of patients

4. 40 % of patients

5.

Correct Answer: 1.

Question 85: Exudative RD is seen in all of the following except

1. Vogt Koyanagi Harada syndrome

2. Coat's disease

3. Cataract

4. VHL disease

5.

Correct Answer: 3.

Question 86: Spring catarrh is characterized by

1. White ropy discharge

2. Thin purulent discharge

3. Thick purulent discharge

4. Serous discharge

5.

Correct Answer: 1.

Page 137: FMGE (MCI Screening test)  preparation material by Gankidi Raghavender Reddy

Question 87: A lady wants LASIK surgery for her daughter. She asks for your opinion. All the following things are suitable

for performing LASIK except

1. Myopia of 4 Diopters

2. Age of 15 years

3. Stable refraction for 1 year

4. Corneal thickness of 600 microns

5.

Correct Answer: 2.

Question 88: Ocular side effects of steroids are all of the following except

1. Glaucoma

2. Cataract

3. Central serous retinopathy

4. Inflammation

5.

Correct Answer: 4.

Question 89: Keratoderma blenorrhegica is seen in

1. Behcet's disease

2. Reiter's syndrome

3. Ankylosing spondylosis

4. None of the above

5.

Correct Answer: 1.

Question 90: Complicated cataract is not seen in

1. Diabetes mellitus

2. Retinitis pigmentosa

3. Malignant myopia

4. Iridocyclitis

5.

Correct Answer: 2.

Question 91: All of the following are methods to correct myopia except

1. Spectacles

2. LASIK

3. Contact lens

4. Tablets

5.

Correct Answer: 4.

Question 92: Hypertension affects eye in following ways

1. Retinopathy

2. Choroidopathy

3. Optic neuropathy

Page 138: FMGE (MCI Screening test)  preparation material by Gankidi Raghavender Reddy

4. All of the above

5.

Correct Answer: 4.

Question 93: Which antibiotics are used commonly as intravitreal agent in endophthalmitis

1. Vancomycin

2. Ceftazidime

3. Both are not used

4. Both are used

5.

Correct Answer: 4.

Question 94: Right homomymous hemianopia is due to defect at

1. Optic chiasma

2. Left optic tract

3. Right optic radiation

4. Right optic nerve

5.

Correct Answer: 3.

Question 95: All of the following are refractive surgeries except

1. LASIK

2. LASEK

3. PRK

4. Phaco

5.

Correct Answer: 4.

Question 96: In angle-closure glaucoma, treatment given to fellow eye is

1. Pilocarpine eyedrops

2. Atropine

3. Laser iridectomy

4. Physostigmine eyedrops

5.

Correct Answer: 3.

Question 97: Omar has hypermetropia needs + 1.5 D glasses for distant vision. What would happen to his near vision

when his glasses slip down his nose

1. Distorted

2. Remains unchanged

3. Becomes better

4. Becomes worse

5.

Correct Answer: 3.

Page 139: FMGE (MCI Screening test)  preparation material by Gankidi Raghavender Reddy

Question 98: Cobble stone appearance is seen in

1. Vernal conjunctivitis

2. Epidemic keratoconjunctivitis

3. Phlyctenular conjunctivitis

4. Angular conjunctivitis

5.

Correct Answer: 1.

Question 99: Following are components of Sjogren's syndrome except

1. Dry eye

2. Rheumatoid arthritis

3. Dry mouth

4. Retinal detachment

5.

Correct Answer: 4.

Question 100: All of the following are treatment methods for acute uveitis except

1. Topical steroids

2. Oral steroids

3. Posterior subtenon steroid injection

4. Surgery

5.Correct Answer: 4.

Question 1: A 2 year old child with Down syndrome was investigated. Her father was found to have a balanced 21-21 translocation. What is the chance of the next offspring to have Down syndrome

1. 0%

2. 4%

3. 50%

4. 100%

5.

Correct Answer: 3.

Question 2: A young boy, Ram Lal developed respiratory distress. On examination, the Chest Xray showed hyperinflation

of one lung. Which of the following is the most possible diagnosis

1. Congenital lobar emphysema

2. Foreign body aspiration

3. Bronchiectasis

4. Atelectasis

5.

Correct Answer: 2.

Page 140: FMGE (MCI Screening test)  preparation material by Gankidi Raghavender Reddy

Question 3: Child care virus is

1. Para influenza virus

2. Herpes virus

3. Chicken pox virus

4. Adenovirus

5.

Correct Answer: 4.

Question 4: A newborn was brought with persistent crying and was found to have rashes. On laboratory examination, it

was found to have lactic acidosis. The diagnosis is likely to be

1. Organic aciduria

2. Urea cycle defect

3. MELAS with lactic acidosis

4. Aminoacidopathy

5.

Correct Answer: 1.

Question 5: Which one of the following statements is false with regard to pyuria in children

1. Presence of more than 5 WBC/hpf (high power field) for girls and more than 3 WBC/hpf for boys

2. Infection can occur without pyuria

3. Pyuria may be present without Urinary tract infection

4. Isolated pyuria is neither confirmatory nor diagnostic for Urinary tract infection

5.

Correct Answer: 2.

Question 6: Upper respiratory tract infection is a common problem in children. All the following anesthetic complications

can occur in children with respiratory infections except

1. Bacteremia

2. Halothane granuloma

3. Increased mucosal bleeding

4. Laryngospasm

5.

Correct Answer: 2.

Question 7: A 7 year old girl from Bihar presented with three episodes of massive hematemasis and melena. There is no

history of jaundice. On examination, she had a large spleen, non-palpable liver and mild ascites. Portal vein was not

visualized on ultrasonography. Liver function tests were normal and endoscopy reveled esophageal varices. The most likely

diagnosis is

1. Kala azar with portal hypertension

2. Portal hypertension of unknown etiology

3. Chronic liver disease with portal hypertension

4. Portal hypertension due to extrahepatic obstruction

5.

Correct Answer: 4.

Page 141: FMGE (MCI Screening test)  preparation material by Gankidi Raghavender Reddy

Question 8: A child was brought 6 hours after a snake bite. On examination a localized edema of less than two inches

edema was present at the site of the bite. There were no systemic signs and symptoms. The most correct management

would be

1. Incision and suction

2. Inject antivenom locally

3. Polyvalent antivenom should be started i.v

4. Observe for progression of symptoms

5.

Correct Answer: 3.

Question 9: A child with a small head, minor anomalies of the face including a thin upper lip, growth delay, and

developmental disability can have all of the following, except:

1. A chromosomal syndrome

2. A Mendelian syndrome

3. A polygenic syndrome

4. A teratogenic syndrome

5.

Correct Answer: 3.

Question 10: Common cause for epiglottitis in children is due to

1. Hemophilus influenzae

2. Pneumococci

3. Staphylococci

4. Streptococci

5.

Correct Answer: 1.

Question 11: A 12 week old child should be further evaluated for developmental delay if found to have which of the

following defects

1. Failure to raise head to 90 degrees

2. Fails to vocalize

3. Unable to make babbling noises

4. Does not transfer a red ring even when given in hand

5.

Correct Answer: 1.

Question 12: Which one of the following is the most complication of measles infection in children

1. Otitis media

2. Bronchopneumonia

3. Encephalitis

4. Diarrhea

5.

Correct Answer: 1.

Question 13: Hypoglycemia in newborn is defined as the blood sugar level below (in mg %)

Page 142: FMGE (MCI Screening test)  preparation material by Gankidi Raghavender Reddy

1. 30

2. 40

3. 50

4. 55

5.

Correct Answer: 2.

Question 14: While bathing a 6 month old child, his mother noted that there was a swelling in the abdomen towards the

left side. On examination there was an abdominal mass, which showed calcification near the left kidney. What is the most

probable diagnosis

1. Lymphoma

2. Leukemia

3. Neuroblastoma

4. Renal cell carcinoma

5.

Correct Answer: 3.

Question 15: Transient myeloproliferative disorder of the new born is seen in association with

1. Turner syndrome

2. Down syndrome

3. Neurofibromatosis

4. Ataxia telangiectasia

5.

Correct Answer: 2.

Question 16: Which one of the following is not a component of APGAR scoring in the newborn

1. Heart rate

2. Muscle tone

3. Respiratory effort

4. Body temperature

5.

Correct Answer: 4.

Question 17: The cause for anoixic spells in children include all except

1. Transposition of great vessels

2. Tricuspid artresia

3. Total anomalous of muscular venous return

4. Coarctation of aorta

5.

Correct Answer: 4.

Question 18: The 9 month old child of a diabetic mother presents with tachypnea and hepatomegaly. Echo cardiography

of the heart showed normal cardiac morphology with asymmetric septal hypertrophy. Which of the following you will give to

treat this child

1. Digoxin

Page 143: FMGE (MCI Screening test)  preparation material by Gankidi Raghavender Reddy

2. Frusemide

3. Propranolol

4. Isoptin

5.

Correct Answer: 3.

Question 19: Which of the following hepatitis viruses have significant perinatal transmission

1. Hepatitis E virus

2. Hepatitis C virus

3. Hepatitis B virus

4. Hepatitis A virus

5.

Correct Answer: 3.

Question 20: A 6-year old boy is brought at emergency with purpuric and petechial rash all over the body. He appears

semiconscious and febrile. Total leukocyte count is 20000 per mm3 with 88% polymorphs. platelet count is 1-2 lac/mm3.

The most likely diagnosis is

1. Idiopathic thrombocytopenic purpura

2. Meningococcemia

3. Staphylococcal toxic shock syndrome

4. Dengue hemorrhagic fever

5.

Correct Answer: 2.

Question 21: Osteomatacia is associated with

1. Decrease in osteoid volume

2. Derease in osteoid surface

3. Increase in osteoid maturiaton time

4. Increase in mineral apposition rate

5.

Correct Answer: 3.

Question 22: A 2 year old boy presented with fever. On examination, he had a respiratory rate of 50 per minute with

bilateral crepitations and no chest indrawing. Cyanosis was present. Which of the following is the most likely diagnosis

1. Pneumonia

2. No pneumonia

3. Severe pneumonia

4. Upper respiratory tract infection

5.

Correct Answer: 1.

Question 23: Which positive test does not necessarily indicate HIV infection in a newborn?

1. ELISA for HIV IgG antibody

2. p24 antigen

3. Virus culture

Page 144: FMGE (MCI Screening test)  preparation material by Gankidi Raghavender Reddy

4. ELISA for HIV IgA antibody

5.

Correct Answer: 1.

Question 24: All the following are seen in congenital rubella syndrome except

1. Mental retardation

2. Aortic stenosis

3. Dearness

4. Patent ductus arteriosus

5.

Correct Answer: 2.

Question 25: The most common genetic disorder causing liver disease in children is

1. Hemochromatosis

2. Alpha 1 antitrypsin deficiency

3. Glycogen storage diseases

4. Cystic fibrosis

5.

Correct Answer: 2.

Question 26: A 2 year old child develops acute respiratory distress and was brought to the casualty. On examination,

there were decreased breath sounds with wheeze on the right side. The Chest Xray showed diffuse opacity in the right

hemithorax. Which of the following is the most likely diagnosis?

1. Pneumothorax

2. Foreign body aspiration

3. Pleural effusion

4. Unilateral emphysema

5.

Correct Answer: 2.

Question 27: A 2-month baby presents with history of jaundice, turmeric colored urine and pale stools since birth.

Examination reveals liver span of 10 cms. Firm in consistency and spleen of 3 cms. The most specific investigation for

establishing the diagnosis would be

1. Liver function tests

2. Ultrasound abdomen

3. Peroperative Cholangiogram

4. Liver biopsy

5.

Correct Answer: 4.

Question 28: An 8 year old patient presented with fever and cervical lymphadenopathy with prior history of sore throat.

There was no hepatomegaly. The peripheral blood smear shows more than 20 lymphoplasmacytoid cells. The diagnosis is

most likely to be

1. Tuberculosis

2. Infectious mononucleosis

3. Acute leukemia

Page 145: FMGE (MCI Screening test)  preparation material by Gankidi Raghavender Reddy

4. Influenza

5.

Correct Answer: 2.

Question 29: A 5 year old boy passed 18 loose stools in last 24 hour and vomited twice in last 4 hour. he is irritable but

drinking fluids. The optimal therapy for this child is

1. Intravenous fluids

2. Oral rehydration therapy

3. Intravenous fluid initially for 4 hours followed by oral fluids

4. Plain water ad libitum

5.

Correct Answer: 2.

Question 30: Child of Vasanthi, was weaned from breast milk on the fifth day and was given sugar cane juice. The child

developed hypoglycemia and hepatomegaly. Biochemical examination showed hypophosphatemia and reducing

substances in urine. The child is probably suffering from which of the following enzyme deficiencies

1. Aldolase B

2. Fructokinase

3. Glucose 6 Phosphatase

4. Beta Galactosidase

5.

Correct Answer: 1.

Question 31: A 2 month old baby with accute icteric viral hepatitis like illness slips into encephalopathy after 48 hours.

The mother is a known hepatitis B carrier. Mother's hepatitis B virus serological profile is most likely to be

1. HBsAg positive only

2. HbsAg and HBeAg positve

3. HBsAg and HBe antibody positive

4. HBV DNA positive

5.

Correct Answer: 2.

Question 32: In a newborn with respiratory distress syndrome, all the following are true except

1. Increased incidence in < 34 weeks

2. More common in infants of diabetic mothers

3. 100% oxygen is given in treatment

4. Cyanosis is present

5.

Correct Answer: 3.

Question 33: A ten-year old girl presents with swelling of one knee joint. All of the following conditions can be considered

in the differential diagnosis, except:

1. Tuberculosis

2. Juvenile rheumatoid arthritis

3. Haemophilia.

Page 146: FMGE (MCI Screening test)  preparation material by Gankidi Raghavender Reddy

4. Villonodular synovitis

5.

Correct Answer: 3.

Question 34: Thirty-eight children consumed eatables procured from a single source at a picnic party. Twenty children

developed abdominal cramps followed by vomiting and watery diarrhea 6-10 hours after the party. The most likely etiology

for the outbreak is

1. Rotavirus infection

2. Entero-toxigenic E.Coli infection

3. Staphylococcol toxin

4. Clostridium perfringens infection

5.

Correct Answer: 4.

Question 35: Which one of the folloing is the most common cause of abdominal mass in neonates

1. Neuroblastomas

2. Wilms’ tumour

3. Distended bladder

4. Multicystic dysplastic kidneys

5.

Correct Answer: 4.

Question 36: An ultrasound in a 3 year old child shows a 3 cm circumscribed hypoechoic mass in the abdomen. The

diagnosis is most likely to be

1. Neuroblastoma

2. Wilm’s tumor

3. Oncocytoma

4. Renal cell carcinoma

5.

Correct Answer: 2.

Question 37: All are important causes of post neonatal mortality in India except

1. Respiratory infections

2. Diarrhea

3. Malnutrition

4. Tetanus

5.

Correct Answer: 4.

Question 38: All of the following are chromosomal breakage syndrome except

1. Fanconi’s anaemia

2. Ehlers-Danlos syndrome

3. Bloom syndrome

4. Ataxia telengiectasia

5.

Page 147: FMGE (MCI Screening test)  preparation material by Gankidi Raghavender Reddy

Correct Answer: 2.

Question 39: You are part of a neonatal resuscitation team attending a delivery. A baby weighing 3 kg is born and has not

cried immediately. What sequence of resuscitation steps will you carry out for this baby

1. Dry the baby; position the tactile stimulation; suction of mouth and nose ; free flow oxygen if required

2. Day the baby; free flow oxygen; suction of mouth and nose

3. Position the baby; suction of mouth and nose ;free flow oxygen; tactile stimulation; dry the baby

4. Dry the baby; position the baby; suction of mouth and nose; tactile stimulation; free flow oxygen if required

5.

Correct Answer: 4.

Question 40: A 2 year old child weighing 6.7 kg and with a history of diarrhea and vomiting for 2 days is brought to the

hospital. On examination, the skin when pinched up took almost seconds to return to normal. Which of the following is the

most likely diagnosis

1. No dehydration

2. Some dehydration

3. Severe dehydration

4. Skin pinching is not reliable in making a diagnosis

5.

Correct Answer: 1.

Question 41: A 1-moth old baby presents with frequent vomiting and failure to thrive. There are features of moderate

dehydrtion. Blood sodium is 122mEa/l and potassium is 6.1 mEq/l. The most likely diagnosis is:

1. Gitelman syndrome

2. Bartter syndrome

3. 21-hydroxylase deficiency

4. 11-b hydroxylase deficiency

5.

Correct Answer: 3.

Question 42: Which one of the following is not associated with jaundice in infancy

1. Hereditary fructose intolerance

2. Maple syrup urine disease

3. Galactosemia

4. Crigler Najjar syndrome

5.

Correct Answer: 1.

Question 43: An ultrasound in a 3 year old child shows a 3 cm circumscribed hypoechoic mass in the abdomen. The

diagnosis is most likely to be

1. Neuroblastoma

2. Wilm’s tumor

3. Oncocytoma

4. Renal cell carcinoma

5.

Page 148: FMGE (MCI Screening test)  preparation material by Gankidi Raghavender Reddy

Correct Answer: 2.

Question 44: A 6 month old infant develops high grade fever which subsided after 3 days following which there was

appearance of a generalized rash which too subsided after 48 hours without any residual pigmentation. Which of the

following is the most likely diagnosis

1. Roseola infantum

2. Measles

3. Rubella

4. Dengue fever

5.Correct Answer: 1.

Question 1: Neonatal thymectomy leads to

1. Decreased size of germinal center

2. Decreased size of paracortical areas

3. Increased antibody production by B cells

4. Increased bone marrow production of lymphocytes

5.

Correct Answer: 2.

Question 2: All of the following are associated with low C3 levels except

1. Post streptococcal glomerulonephritis

2. Membrano-proliferative Glomerulonephritis

3. Good pasture's disease

4. Systemic lupus erythematosus

5.

Correct Answer: 3.

Question 3: Which of the following increases the susceptibility to coronary artery disease

1. Type V hyperlipoproteinaemia

2. Von will Brand’s disease

3. Nephritic syndrome

4. Systemic lupus erythematosus

5.

Correct Answer: 1.

Question 4: Recurrent ischemic events following thrombolysis has been patho-physiologically linked to which of the

following factors

1. Antibodies to thrombolytic agents

2. Fibrinopeptide A

3. Lipoprotein A

4. Triglycerides

5.

Correct Answer: 3.

Page 149: FMGE (MCI Screening test)  preparation material by Gankidi Raghavender Reddy

Question 5: All the following can be used to detect mutation except

1. Single chain polymorphism

2. Ligase chain reaction

3. Polymerase chain reaction

4. Genome sequence chain

5.

Correct Answer: 2.

Question 6: Which of the following procedures are wsed as routine technique for karyotyping using light microscopy

1. C- banding

2. G- banding

3. Q- banding

4. B-rd V- staining

5.

Correct Answer: 2.

Question 7: Hypertrophic pulmonary osteoarthropathy is due to

1. Oat cell carcinoma

2. Squmous cell carinoma

3. Adenocarcinoma

4. Adenosqumous carcinoma

5.

Correct Answer: 3.

Question 8: In Henoch-Schonlein purpura, which of the following is seen

1. Blood in stool

2. Recurrent infections

3. Thrombocytopenia

4. Intracranial hemorrhage

5.

Correct Answer: 1.

Question 9: All the following are seen in asbestosis except

1. Mesothelioma

2. Pleural plaque

3. Calcification

4. Diffuse interstitial fibrosis

5.

Correct Answer: 4.

Question 10: Which of the following does not cause interstitial lung disease

1. Tobacco smoke

2. Sulfur dioxide

3. Thermophilic actinomycetes

Page 150: FMGE (MCI Screening test)  preparation material by Gankidi Raghavender Reddy

4. Coal dust

5.

Correct Answer: 1.

Question 11: In a chronic alcoholic all the following are seen in the liver except

1. Fatty degeneration

2. Chronic hepatitis

3. Granuloma formation

4. Cholestatic hepatitis

5.

Correct Answer: 3.

Question 12: Most important prognostic factor of carcinoma is

1. Grade

2. Stage

3. Mitotic index

4. Metastasis

5.

Correct Answer: 3.

Question 13: Serum total lactate dehydrogenase level will NOT be raised in

1. Muscle crush injury

2. Stroke

3. Myocardial infrction

4. Hemolysis

5.

Correct Answer: 2.

Question 14: In AML Chromosomal anomaly is seen at

1. 9 & 22

2. 21 & 9

3. 20 & 10

4. 8 & 14

5.

Correct Answer: 4.

Question 15: Locally malignant tumor is

1. Chordoma

2. Chondroma

3. Choristoma

4. Hamartoma

5.

Correct Answer: 1.

Question 16: Munro micro abcess is seen in ?

Page 151: FMGE (MCI Screening test)  preparation material by Gankidi Raghavender Reddy

1. mycosis fungoides

2. psoriasis

3. lichen planus

4. bacterial infection

5.

Correct Answer: 2.

Question 17: Genotype of Turner Syndrome is

1. XO

2. XXO

3. XXY

4. XY

5.

Correct Answer: 1.

Question 18: All of the following are poor prognostic factors for acute myeloid leukemias, except

1. Age more than 60 years

2. Leucocyte count more than 1,000,000/ml

3. Secondary leukemias

4. Presence of t (8:21)

5.

Correct Answer: 4.

Question 19: MHC class 3 Genes encode

1. Complement Component C3

2. Tumor necrosis factor

3. Interleukin 2

4. Beta 2 microglobulin

5.

Correct Answer: 2.

Question 20: Principal lipoprotein manifestation of heterozygous familial hypercholesterolemia

1. Increased chylomicrons

2. Decreased chylomicrons

3. Increased LDL

4. Decreased LDL

5.

Correct Answer: 3.

Question 21: Kaposi sarcoma is caused by

1. Herpes virus

2. Adeno virus

3. Arbo virus

4. Reo virus

5.

Page 152: FMGE (MCI Screening test)  preparation material by Gankidi Raghavender Reddy

Correct Answer: 1.

Question 22: All of the following conditions may predispose to pulmonary embolism except

1. Protein S deficiency

2. Malignancy

3. Obesity

4. Progesterone therapy

5.

Correct Answer: 4.

Question 23: An albino girls gets married to a normal boy. What are the chances of their having an affected child and

what are the chances of their children being carriers

1. None affected, all carriers

2. All normal

3. 50% carriers

4. 50% affected. 50% carriers

5.

Correct Answer: 1.

Question 24: Which of the following is pan - T laymphocyte carrier

1. CD 2

2. CD 3

3. CD 19

4. CD 25

5.

Correct Answer: 1.

Question 25: In primary tuberculosis, all the following are seen except

1. Cavitation

2. Caseation

3. Calcification

4. Langhan giant cell

5.

Correct Answer: 1.

Question 26: Crescents in renal biopsy is seen in

1. Membranous Glomerulonephritis

2. Membranoproliferative Glomerulonephritis

3. Rapidly progressive Glomerulonephritis

4. Post Streptococcal Glomerulonephritis

5.

Correct Answer: 3.

Question 27: About neutrophills all are incorrect except

Page 153: FMGE (MCI Screening test)  preparation material by Gankidi Raghavender Reddy

1. 10-20% of all leucocytes

2. Phagocytosis

3. Antibody formation

4. Increase in chronic infections

5.

Correct Answer: 2.

Question 28: Which organ is not involved in MEN I

1. Pancreas

2. Adrenals

3. Parathyroid

4. Pitutary

5.

Correct Answer: 2.

Question 29: An intestinal biopsy specimen showed macrophages with partially and completely digested bacteria. Which

of the following conditions could produce such a picture

1. Whipple’s disease

2. Immunoproliferative Small Intestinal Disease

3. Cholera

4. Lymphoma

5.

Correct Answer: 1.

Question 30: The HLA class 3 rejection genes are important elements in

1. Transplant rejection phenomenon

2. Governing susceptibility of autoimmune diseases

3. Immune surveillance

4. Antigen presentation and elimination

5.

Correct Answer: 2.

Question 31: Thromboxane A2 causes

1. Vasodilation

2. Platelet inhibition

3. Both of above

4. None of the above

5.

Correct Answer: 4.

Question 32: Stricture of gut is caused by

1. Typhoid

2. Tuberculosis

3. Crohn’s disease

Page 154: FMGE (MCI Screening test)  preparation material by Gankidi Raghavender Reddy

4. Ulcerative colitis

5.

Correct Answer: 2.

Question 33: In polycythemia vera, all the following are seen except

1. Hyperuricemia

2. Thrombosis

3. Evolution into acute leukemia

4. Spontaneous bacterial infection

5.

Correct Answer: 4.

Question 34: Commonest site for scrofuloderma is

1. Larynx

2. Skin

3. Lymph nodes

4. Lungs

5.

Correct Answer: 3.

Question 35: Example of type IV hypersensitivity

1. Coagulase test

2. Mantoux test

3. Schick test

4. Elek’s gel test

5.

Correct Answer: 2.

Question 36: In granuloma, epithelial and giant cells are derived from

1. T cells

2. B Cells

3. Plasma

4. Macrophages/Monocytes

5.

Correct Answer: 4.

Question 37: All of the following are Autosomal dominant except

1. Hereditary spherocytosis

2. Von Willebrand's disease

3. Hemochromatosis

4. Familial hypercholestrolemia

5.

Correct Answer: 3.

Question 38: A 45 day old infant developed icterus and two days later symptoms and signs of acute liver failure appeared.

Page 155: FMGE (MCI Screening test)  preparation material by Gankidi Raghavender Reddy

Child was found to be positive for HBs Ag. The mother was also HBs Ag carrier. The mother's hepatitis B serological profile

is likely to be

1. HBs Ag positive only

2. HBsAg and HBeAg positivity

3. HBsAg and anti-Hbe antibody positivity

4. Mother infected with mutant HBV

5.

Correct Answer: 2.

Question 39: Necrotizing papillitis is seen in all except

1. Sickle cell disease

2. Tuberculous pyelonephritis

3. Diabetes mellitus

4. Analgesic nephropathy

5.

Correct Answer: 2.

Question 40: DNA analysis of chorionic villus/ amniocentesis NOT likely to detect

1. tay sachs

2. hemophilia A

3. sickle cell disease

4. duchenne muscular dystrophy

5.

Correct Answer: 2.

Question 41: A 10 year old boy, Pappu, died of acute rheumatic fever. All the following can be expected on autopsy

except

1. Aschoff nodules

2. Rupture of chordae tendinae

3. MacCallum patch

4. Fibrinous pericarditis

5.

Correct Answer: 2.

Question 42: IL-1 is involved in

1. T lymphocyte activation

2. Decreased wound healing

3. Gain in body weight

4. Macrophage activation

5.

Correct Answer: 1.

Question 43: All the statement about lactoferrin are true, except

1. It is present in secondary granules of neutrophil

2. It is present in exocrine secretios of body

Page 156: FMGE (MCI Screening test)  preparation material by Gankidi Raghavender Reddy

3. It has great affinity for iron

4. It transports iron for erythropoiesis

5.

Correct Answer: 4.

Question 44: HLA association in Diabetes mellitus is seen in

1. Type I

2. Type II

3. MODY

4. Malnutrition associated

5.

Correct Answer: 1.

Question 45: A 15 year old healthy boy with no major medical problem complaints that he breaks out with blocky areas of

erthema that are pruritic over skin of his arm, leg and trunk every time within an hour of eating sea foods. The clinical

features are suggestive of

1. Localised immune-complex deposition

2. Cell mediated hypersensitivity

3. Locallized anaphylaxis

4. Release of complement C3b

5.

Correct Answer: 3.

Question 46: The inheritance in Dechenne muscular dystrophy is

1. X- linked recessive

2. Autosomal dominant

3. Autosomal recessive

4. Polygenic

5.

Correct Answer: 1.

Question 47: M4 stage of AML is called as

1. Acute myelomonoblastic leukemia

2. Acute monoblastic leukemia

3. Myelomonocytic leukemia

4. Monocytic leukemia

5.

Correct Answer: 1.

Question 48: Lipo-oxygenase mediated chemotoxis is by

1. B4

2. C4

3. D4

4. E4

5.

Page 157: FMGE (MCI Screening test)  preparation material by Gankidi Raghavender Reddy

Correct Answer: 1.

Question 49: Soft, friable node enlargement is seen in

1. Typhoid

2. Thallesemia

3. Syphilis

4. Lymphoma

5.

Correct Answer: 1.

Question 50: Disease or infarction of neurological tissue causes it to be replaced by

1. Fluid

2. Neuroglia

3. Proliferation of adjacent nerve cells

4. Blood vessel

5.

Correct Answer: 2.

Question 51: Histologic hallmark of paget disease of nipple is

1. Comedo necrosis

2. Atypical lobular hyperplasia

3. Infiltration of the epidermis by malignant cells

4. Desmoplasia

5.

Correct Answer: 3.

Question 52: Most sensitve and specific test ofr diagnosis of iron efficiency is

1. Serum iron levels

2. Serum ferritin levels

3. Serum transferrin receptor population

4. Transferrin saturation

5.

Correct Answer: 2.

Question 53: Friedrich type V hyperlipidemia is due to excess of

1. HDL

2. LDL

3. IDL

4. Chylomicrons

5.

Correct Answer: 4.

Question 54: All of the following are morphologic feature of apoptosis except

1. Cell shrinkage

Page 158: FMGE (MCI Screening test)  preparation material by Gankidi Raghavender Reddy

2. Chromatin condensation

3. Inflammation

4. Apoptotic bodies

5.

Correct Answer: 3.

Question 55: Intestinal epithelial cells contain

1. T cells

2. B cells

3. Plasma cells

4. Macrophages

5.

Correct Answer: 3.

Question 56: Leukoerythroblastic picture may be seen in all of the following except

1. Thalassemia

2. Gaucher’s disease

3. Metastatic carcinoma

4. Myelofibrosis

5.

Correct Answer: 1.

Question 57: Karyotype in Klienefilter's syndrome

1. XO

2. XX

3. XY

4. XXY

5.

Correct Answer: 4.

Question 58: What is the size of the microfilter used in blood transfusion sets

1. 50micro meters

2. 150micro meters

3. 170micro meters

4. 250micro meters

5.

Correct Answer: 3.

Question 59: The tumor most commonly metastasizing to bone is

1. Neuroblastoma

2. Wilm’s tumor

3. Glioma

4. Sarcoma

5.

Correct Answer: 1.

Page 159: FMGE (MCI Screening test)  preparation material by Gankidi Raghavender Reddy

Question 60: The most pre malignant lesion among the following is

1. Leukoplakia

2. Erythroplakia

3. Metaplasia

4. Dysplasia

5.

Correct Answer: 2.

Question 61: In early gastric carcinoma malignancy is confined to

1. Mucosa

2. Mucosa and submucosa

3. Gastric wall without lymphnode metaasis

4. Gastric glands

5.

Correct Answer: 2.

Question 62: The most common inheritance pattern of congenital heart disease is

1. Autosomal dominant

2. Autosomal recessive

3. Sporadic mutations

4. Multifactorial

5.

Correct Answer: 4.

Question 63: Gluten sensitve enteropthy is most strongly associated with

1. HLA-DQ2

2. HLA-DQ4

3. HLA-DQ3

4. Blood group ‘B’

5.

Correct Answer: 1.

Question 64: Highly selective proteinuria contains

1. Fibrinogen and albumin

2. Transferrin

3. Transferrin and fibrinogen

4. Fibrinogen

5.

Correct Answer: 1.

Question 65: All the following are causes for granuloma of the liver except

1. Sarcoidosis

2. Candidiasis

3. Hepatic metastasis

Page 160: FMGE (MCI Screening test)  preparation material by Gankidi Raghavender Reddy

4. Halothane

5.

Correct Answer: 3.

Question 66: Cells in M4 AML are

1. Monoblasts & myeloblasts

2. Myeloblasts only

3. Monocytes & myelocytes

4. Erythroid cells

5.

Correct Answer: 1.

Question 67: The intestinal biopsy findings in celiac sprue are all except

1. Infiltration of lymphocytes in the lamina propria

2. Crypt hyperplasia

3. Mucosal thickening

4. Inflammatory cells in submucosa

5.

Correct Answer: 3.

Question 68: People with healthy immune system usually have CD4 cell count of

1. 1000-1200 mm3

2. 800-1000 mm3

3. 400-800 mm3

4. 200-400 mm3

5.

Correct Answer: 1.

Question 69: Clinical symptoms associated with primary chylomicronemia:

1. eruptive xanthomas

2. hepatosplenomegaly

3. hypersplenism

4. lipid-laden foam cells in bone marrow

5. all of the above

Correct Answer: 5.

Question 70: A person missing from home, found wandering purposefully, well groomed, has some degree of amnesia

1. dissociative fugue

2. dissociative amnesia

3. schizophrenia

4. dementia

5.

Correct Answer: 1.

Question 71: Cell of origin of Meningioma

Page 161: FMGE (MCI Screening test)  preparation material by Gankidi Raghavender Reddy

1. Archnoid caps cells

2. Fibroblasts

3. Oligodendrocytes

4. Astrocytes

5.

Correct Answer: 1.

Question 72: Which correlate of protective immune response is measured by Mantoux test

1. Antibody mediated hypersensitivity

2. Delayed type of hypersensitivity

3. Antibody dependent cell mediated hypersensitivity

4. Immune complex mediated hypersensitivity

5.

Correct Answer: 2.

Question 73: Which of the following is not true about Berger’s disease

1. The pathologic changes are proliferative and usually confined to measangial cells

2. Hematuria may be gross or microscopic

3. On immunofluorescence deposits contain both IgA and IgG

4. Absence of associated protyeinuria is pathognomonic

5.

Correct Answer: 1.

Question 74: In thrombotic thrombocytopenic purpura, all the following are seen except

1. Fever

2. Hypertension

3. Thrombocytopenia

4. Anemia

5.

Correct Answer: 2.

Question 75: All the following are seen in paroxysmal nocturnal hemoglobinuria except

1. Increased leukocyte alkaline phosphatase

2. Aplastic anemia

3. Thrombosis

4. Iron deficiency anemia

5.

Correct Answer: 1.

Question 76: Gouty tophi for microscopic examination is transferred in

1. Formalin

2. Alcohol

3. Distilled water

4. Normal saline

5.

Page 162: FMGE (MCI Screening test)  preparation material by Gankidi Raghavender Reddy

Correct Answer: 2.

Question 77: Most common tumour of appendix

1. Carcinoid

2. Squmous cell carcinoma

3. Adenocarcinoma

4. Argentaffin type

5.

Correct Answer: 1.

Question 78: In Giant cell tumour source of cells is

1. Osteoclasts

2. Osteoblasts

3. Osteoid

4. Unknown

5.Correct Answer: 4.

Question 1: In tuberous sclerosis, the lesion is1. Adenoma malignum

2. Adenoma sebaceum

3. Café au lait spots

4. Axillary freckling

5.

Correct Answer: 2.

Question 2: Toxin involved in the streptococcal toxic shock syndrome PJC

1. pyrogenic exotoxin

2. erythrogenic toxin

3. hemolysin

4. M Protien

5.

Correct Answer: 1.

Question 3: A 45 year old woman, presenting with the history of diplopia and dysphagia worsening as the day progrsses,

can be diagnosed to have

1. Thyrotoxicosis

2. Myasthenia gravis

3. Muscular dystrophy

4. Brain tumor

5.

Correct Answer: 2.

Question 4: A couple, with a family history of beta thalassemia major in a distant relative. has come for counseling. The

husband had HbA2 of 4.8% and the wife has HbA2 of 2.3%. The risk of having a child with beta thalassemia major is:

Page 163: FMGE (MCI Screening test)  preparation material by Gankidi Raghavender Reddy

1. 50%

2. 25%

3. 5%

4. 0%

5.Correct Answer: 4.

Question 5: Renal osteodystrophy differs from nutritional and metabolic rickets due to the presence of

1. Hyperphosphatemia

2. Hypercalcemia

3. Hypophosphatemia

4. Hypocalcemia

5.Correct Answer: 1.

Question 6: Basanti, 23 year old pregnant women presents with features of dilated cardiomyopathy. Which of the following drugs you will not give to treat her

1. Digoxin

2. ACE Inhibitors

3. Diuretics

4. vasodilators

5.

Correct Answer: 2.

Question 7: Which of the following behavioral problems would suggest an organic brain lesion

1. Formal thought disorder

2. Auditory hallucinations

3. Visual hallucinations

4. Depression

5.

Correct Answer: 3.

Question 8: An early systolic murmur may be caused by all of the following except

1. Small ventricular septal defect

2. Papillary muscle dysfunction

3. Tricuspid regurgitation

4. Aortic stenosis

5.

Correct Answer: 4.

Question 9: If urine sample darkens on standing, most likely condition is

1. phenylketonuria

2. alkaptonuria

3. maplpe syrup urine disease

Page 164: FMGE (MCI Screening test)  preparation material by Gankidi Raghavender Reddy

4. DM

5.

Correct Answer: 2.

Question 10: Rheumatoid factor in rheumatoid arthritis is important because

1. RA factor is associated with bad prognosis

2. Absent RA factor rules out the diagnosis of Rheumatoid arthritis

3. It is very common in childhood Rheumatoid arthritis

4. It correlates with disease activity

5.

Correct Answer: 1.

Question 11: Absence seizures are characterized on EEG by

1. 3Hz spike & wave

2. 1-2Hz spike & wave

3. Generalized polyspikes

4. Hyparrythmia

5.

Correct Answer: 1.

Question 12: All are opportunistic infections in AIDS except

1. Herpes zoster Ophthalmicus

2. Herpes simplex keratitis

3. TB

4. Toxocariasis

5.

Correct Answer: 4.

Question 13: An eleven month child was found to be very restless and hyperactive in the classroom making it difficult for

the other students in the classroom to concentrate. He runs round the class all the day cannot sit at one place for a long

time. He gets very agitated when the toy in his hand is taken away. The most probable diagnosis is

1. Autism

2. Conduct disorder

3. Attention deficit hyperactivity disorder

4. Conversion

5.

Correct Answer: 3.

Question 14: Most common cause of meningitis in kids 6 months to 2 years of age

1. pneumococcus

2. staph

3. H influenza

4. E.coli

5.

Correct Answer: 3.

Page 165: FMGE (MCI Screening test)  preparation material by Gankidi Raghavender Reddy

Question 15: In hepatolenticular degeneration,main sites of copper accumulation are

1. Liver

2. Brain

3. 1 & 2 are true

4. Only 1 is correct

5.

Correct Answer: 3.

Question 16: All of the following are hepatotoxic except

1. INH

2. Rifampicin

3. Pyrazinamide

4. Ethambutol

5.

Correct Answer: 4.

Question 17: Investigation of choice for detecting Syphilis after 2 course of complete therapy is

1. FTA Abs

2. VDRL

3. TPI

4. Dark ground microscopy

5.

Correct Answer: 1.

Question 18: Decreased cardiac output is due to

1. Decreased venous return

2. Decreased end diastolic volume

3. Increased sympathetic stimulation

4. Decreased vagal tone

5.

Correct Answer: 1.

Question 19: Atrial fibrillation is seen in all except

1. Ventricular Septal defect

2. Constrictive pericarditis

3. Myocardial infarction

4. Atrial septal defect

5.

Correct Answer: 1.

Question 20: Commonest presentation of neurocysticercosis is

1. Seizures

2. Focalneurological deficits

3. Dementia

Page 166: FMGE (MCI Screening test)  preparation material by Gankidi Raghavender Reddy

4. Radiculopathy

5.

Correct Answer: 1.

Question 21: The most common cause of tricuspid regurgitation is secondary to

1. Rheumatic heart disease

2. Dilatation of right ventricle

3. Coronary artery disease

4. Endocarditis due to intravenous drug abuse

5.

Correct Answer: 2.

Question 22: A lady approaches a physician for contraceptive advice. On examination there were two symmetrical ulcers

on vulva, which were well defined with a firm base. Which of the following is the most likely cause

1. Syphilis

2. LGV

3. Herpes

4. Malignancy

5.

Correct Answer: 1.

Question 23: Features of tuberous sclerosis are all except

1. Epilepsy

2. Low IQ or Mental retardation

3. Adenoma sebaceum

4. Scleritis

5.

Correct Answer: 4.

Question 24: A 23- year old woman has experience episod of myalgias, pleural effusion, peicarditis and arthralgias

without joint deformity over course of several years. The best laboratory screening test to diagnose her disease would be

1. CD4 Lymphocyte count

2. Erythrocyte sedimentation rate

3. Antinuclear antibody

4. Assay for thyroid hormones

5.

Correct Answer: 3.

Question 25: Rokitansky Kuster hauser syndrome: most likely condition is

1. ovary agenesis

2. fallopian tubes absent

3. vaginal atresia

4. bicornuate uterus

5.

Correct Answer: 3.

Page 167: FMGE (MCI Screening test)  preparation material by Gankidi Raghavender Reddy

Question 26: All the following are true about loculated pleural effusion except

1. It makes an obtuse angle with the chest wall

2. The margins are diffuse when viewed end on

3. Not confined to any bronchopulmonary segment

4. Air bronchograms are seen within the opacity

5.

Correct Answer: 1.

Question 27: Drug of choice for juvenile rheumatoid arthritis is

1. Aspirin

2. Codiene

3. Colchicine

4. Diazepam

5.

Correct Answer: 1.

Question 28: Following are the signs of Wilson's disease except

1. Hepatomegaly

2. Icterus

3. Kayser Fleischer ring

4. Immature cataract

5.

Correct Answer: 4.

Question 29: Which of the following is the drug of first choice for Non-Gonococcal Urethritis

1. Ceftriaxone

2. Ciprofloxacin

3. Doxycycline

4. Minocycline

5.

Correct Answer: 1.

Question 30: Heparin is the commonly used anticoagulant in cardiac surgery. All of the following are true about heparin

except

1. Weakest acid found in living things

2. Most commercial preparations of heparin now utilize pig intestinal slimes

3. Act via Antithrombin activation

4. Produce thrombocytopenia

5.

Correct Answer: 1.

Question 31: Maladie de Roger is

1. Small V.S.D

2. Large V.S.D

Page 168: FMGE (MCI Screening test)  preparation material by Gankidi Raghavender Reddy

3. Small A.S.D

4. Large A.S.D.

5.

Correct Answer: 1.

Question 32: “Subclavian Steal Syndrome” indicates reversal of blood flow in

1. Subclavian artery

2. Internal carotid artery

3. Vertebral artery

4. Posterior inferior cerebellar artery

5.

Correct Answer: 3.

Question 33: Which of the following features denote a contraindication for surgical resection in a case of carcinoma lung

1. Malignant pleural effusion

2. Involvement of visceral pleura

3. Hilar lymphadenopathy

4. Consolidation of one lobe

5.

Correct Answer: 1.

Question 34: A baby with refusal to feed, skin lesions, seizures, organic acids in urine, lactic acidosis with normal ammoni

1. diagnosis is

2. organic aciduria

3. multiple carboxylase deficiency

4. propionic acidemia

5.

Correct Answer: 5.

Question 35: All of the following statements regarding the ECG in acute pericarditis are true except

1. T-wave inversions develop before ST elevations return to baseline

2. Global ST segment elevation is seen in early pericarditis

3. Sinus tachycardia is a common finding

4. PR segment depression is present in majority of patients

5.

Correct Answer: 1.

Question 36: Delusions are seen in all the following except

1. Depression

2. Mania

3. Schizophrenia

4. Obsessive compulsive disorder

5.

Correct Answer: 4.

Page 169: FMGE (MCI Screening test)  preparation material by Gankidi Raghavender Reddy

Question 37: Which of the following ECG changes is least likely in a patient with left pneumothorax

1. Inversion of T wave

2. Left axis deviation

3. Small R wave

4. Electrical alternans

5.

Correct Answer: 2.

Question 38: Which of the following is not associated with increase in intracranial pressure

1. Increase in blood pressure

2. Respiratory depression

3. Tachycardia

4. Deterioration of consciousness

5.

Correct Answer: 3.

Question 39: The use of which one of the following may cause cardiomyopathy

1. Adriamycin

2. Carboplatin

3. Dactinomycin

4. Etoposide

5.

Correct Answer: 1.

Question 40: Acantholysis is characteristic of

1. Pemphigus vulgaris

2. Pemphigoid

3. Erythema multiforme

4. Dermatitis hepetiformis

5.

Correct Answer: 1.

Question 41: Drug of choice in Bells Palsy is

1. Carbamazepine

2. Dexamethasone

3. Clonazepam

4. Aspirin

5.

Correct Answer: 2.

Question 42: Aplastic crisis in sickle cell anemia is associated in infection with

1. Adenovirus

2. Parvovirus

3. Influenza virus

Page 170: FMGE (MCI Screening test)  preparation material by Gankidi Raghavender Reddy

4. Papilloma virus

5.

Correct Answer: 2.

Question 43: A 60 year old male presented to the emergency with breathlessness, facial swelling and dilated veins on the

chest wall. The most common cause is

1. Thymoma

2. Lung Cancer

3. Hodgkin's lymphoma

4. Superior vena caval obstruction

5.

Correct Answer: 4.

Question 44: All are features of Acromegaly except

1. Carpel Tunnel syndrome

2. Prognathism

3. Decreased sweating

4. Arthropathy

5.

Correct Answer: 3.

Question 45: Exercise testing is absolutely contraindicated in which one week following

1. One week following myocardial infarction

2. Unstable angina

3. Aortic stenosis

4. Peripheral vascular disease

5.

Correct Answer: 3.

Question 46: In a patient, annular erythematous lesions on the trunk were seen circumscribed by collarete scales. The

diagnosis is most likely to be

1. Pityriasis versicolor

2. Pityriasis rosea

3. Pityriasis rubra pilaris

4. Lichen planus

5.

Correct Answer: 2.

Question 47: All of the following are the electrocardiographic features of severe hyperkalemia except

1. Peaked T waves

2. Presence of U waves

3. Sine wave pattern

4. Loss of P waves

5.

Correct Answer: 2.

Page 171: FMGE (MCI Screening test)  preparation material by Gankidi Raghavender Reddy

Question 48: The syndrome of inappropriate antidiuretic hormone is characterized by the following

1. Hyponatremia andurine sodium excretion >20meq/l

2. Hypernatremia andurine sodium exeretion < 20meq/l.

3. Hyponatremia and hyperkalemia

4. Hypernatremia and hypokalemia

5.

Correct Answer: 1.

Question 49: Clinical condition associated with spasticity:

1. hypertension

2. stroke

3. multiple sclerosis

4. hypertension and multiple sclerosis

5.

Correct Answer: 4.

Question 50: Saddle back type of fever is seen in

1. Brucellosis

2. Dengue fever

3. Viral fever

4. Enteric fever

5.

Correct Answer: 2.

Question 51: A patient was on treatment with trifluperazine for some time. He presents with complaints of hyperthermia,

lethargy and sweating. What is the next step in management

1. Do a CT scan brain and a hemogram

2. Hemogram, electrolyte levels and creatinine

3. ECG, chest X ray and hemogram

4. Hemogram, CPK and RFT

5.

Correct Answer: 4.

Question 52: A 25 year old man presented with fever, cough, expectoration and breathlessness of 2 months duration.

Contrast enhanced computed tomography of the chest showed bilateral upper lobe fibrotic lessons and mediastinum had

enlarged necrotic nodes with peripheral rim enhancement. Which one of the following is the most probable diagnosis

1. Sarcoidosis.

2. Tuberculosis.

3. Lymphoma

4. Silicosis

5.

Correct Answer: 2.

Question 53: A patient presents 12 hours following a Myocardial infarction. Which of the following enzymes will be

elevated at this stage

Page 172: FMGE (MCI Screening test)  preparation material by Gankidi Raghavender Reddy

1. Lactate dehydrogenase

2. Serum Glutamate Oxalo acetate Transferase

3. Creatinine Phospho Kinase

4. Myoglobin

5.

Correct Answer: 3.

Question 54: Features of Kallmans syndrome includes all except

1. Selective GnRH failure

2. Congenital

3. FSH production is normal

4. Anosmia

5.

Correct Answer: 3.

Question 55: In the JVP, all the following combinations are true except

1. c wave - atrial systole

2. ax descent - atrial relaxation

3. cy descent - ventricular relaxation

4. ya descent - atrial filling

5.

Correct Answer: 1.

Question 56: Kaposi's sarcoma can affect

1. Skin

2. Mouth

3. GIT

4. All are correct

5.

Correct Answer: 4.

Question 57: “Syringomyelia” may be associated with all except

1. spinal cord injury

2. tubercular arachnoiditis

3. Arnold chiari malformation

4. Demyelination

5.

Correct Answer: 4.

Question 58: An eleven year old boy is having tinea capitis on his sclap. The most appropriate line of treatment is

1. Oral griseofulvin therapy

2. Topical griseofulvin therapy

3. Shaving of the scalp

4. Selenium sulphide shampoo

5.

Page 173: FMGE (MCI Screening test)  preparation material by Gankidi Raghavender Reddy

Correct Answer: 1.

Question 59: In Conn’s syndrome, all the following are seen except

1. Hypokalemia

2. Hypernatremia

3. Hypertension

4. Edema

5.

Correct Answer: 4.

Question 60: Following organs can get affected in Marfan syndrome

1. Heart

2. Lung

3. Eye

4. All are true

5.

Correct Answer: 4.

Question 61: All the following are correct statements regarding findings in JVP except

1. Cannon wave - Complete heart block

2. Slow vy descent - Tricuspid regurgitation

3. Giant cv wave - Tricuspid stenosis

4. Increased JVP with prominent pulsations - SVC obstruction

5.

Correct Answer: 1.

Question 62: Osler;s nodes are typically seen in which one of the follwing

1. Chronic Candida endocarditis

2. Acute Staphylococcal endocarditis

3. Pseudomonas endocarditis

4. Libman sack’s endocarditis

5.

Correct Answer: 2.

Question 63: Acquired extra –cranial infection that causes aqueductal stenosis is

1. Bacterial endocarditis

2. Mumps

3. Measles

4. Staphylococcal septicemia

5.

Correct Answer: 2.

Question 64: Skin contains mainly two glands

1. Sweat & sebaceous

Page 174: FMGE (MCI Screening test)  preparation material by Gankidi Raghavender Reddy

2. Meibomian and Eccrine

3. Apocrine and sweat gland

4. Eccrine and apocrine

5.

Correct Answer: 1.

Question 65: Which of the following is the best method to assess the adequacy of replacement fluid in a case of shock

1. Decrease in thirst

2. Increase in urine out put

3. Blood pressure

4. Increased Pa O2

5.

Correct Answer: 2.

Question 66: Pulmonary hypertension may occur in all of the following conditions except

1. Toxic oil syndrome

2. Progressive systemic sclerosis

3. Sickle cell anaemia

4. Argemone mexicana poisoning

5.

Correct Answer: 4.

Question 67: In which of the following conditions does hyperglycemia occur

1. Addison’s disease

2. Uremia

3. Acromegaly

4. Fever

5.

Correct Answer: 3.

Question 68: A patient with nephrotic syndrome with longstanding corticosteroid therapy may develop all the following

except

1. Hyperglycemia

2. Hypertrophy of muscle

3. Neuropsychiatric symptoms

4. Suppression of the pituitary adrenal axis

5.

Correct Answer: 2.

Question 69: A patient presents with hypertension and dizziness. The X-ray shows anterior rib notching. The diagnosis is

most likely to be

1. Pheochromocytoma

2. Coarctation of the aorta

3. Neurofibromatosis

4. Marfan syndrome

5.

Page 175: FMGE (MCI Screening test)  preparation material by Gankidi Raghavender Reddy

Correct Answer: 2.

Question 70: Which one of the following is not a feature of phenylketonuria

1. Severe mental retardation

2. Reduced tendon reflexes

3. Enamel hypoplasia

4. Vomiting in early infancy

5.

Correct Answer: 4.

Question 71: A 15 year old boy feels that the dirt has hung onto him whenever he passes through the dirty street. This

repetitive thought causes much distress and anxiety. He knows that there is actually no such thing after he has cleaned

once but he is not satisfied and is compelled to think so. This has led to social withdrawal. He spends much of his time

thinking about the dirt and contamination. This has affected his studies also. The most likely diagnosis is

1. Obsessive compulsive disorder

2. Conduct disorder

3. Agarophobia

4. Adjustment disorder

5.

Correct Answer: 1.

Question 72: Erythema nodosum is seen in

1. Sulphonamides

2. Phenylbutazone

3. Digoxin

4. Ciprofloxacin

5.

Correct Answer: 1.

Question 73: In mind relaxed and eyes close the wave is

1. Alpha

2. Beta

3. Theta

4. Delta

5.

Correct Answer: 1.

Question 74: A case of carcinoma cervix found to be in altered sensorium and having hiccups. Cause could be

1. septicemia

2. uremia

3. dementia

4.

5.

Correct Answer: 2.

Page 176: FMGE (MCI Screening test)  preparation material by Gankidi Raghavender Reddy

Question 75: Following cardiovascular abnormalities are seen in Marfan syndrome

1. Mitral valve prolapse

2. Aortic aneurysm

3. Aortic dissection.

4. All are true

5.

Correct Answer: 4.

Question 76: Osteoporosis is caused by all except

1. Methotrxate

2. Glucocorticoides

3. Heparin

4. Estradiol

5.

Correct Answer: 4.

Question 77: A child presents with white patch over the tonsils and resp. distress. A throat swab was taken. It should be

cultured with

1. loeffler serus slope

2. LJ medium

3. Caryblair media

4. Caryblair media

5.

Correct Answer: 1.

Question 78: The most sensitive and practical technique for detection of myocardial ischemia in the perioperative period

is

1. Magnetic Resonance Spectroscopy

2. Radio labelled lactate dehydogenase determination

3. Direct measurement of end diastolic pressure

4. Regional wall motion abnormality detected wth the help of 2 D transoesphagealechocardiography

5.

Correct Answer: 4.

Question 79: Nephrotic syndrome can be caused by all except

1. Malaria

2. Syphilis

3. Hepatitis

4. Mycoplasma

5.

Correct Answer: 4.

Question 80: Uhthoff's phenomenon is seen in

1. Multiple sclerosis

Page 177: FMGE (MCI Screening test)  preparation material by Gankidi Raghavender Reddy

2. Guillian Barie syndrome

3. Marfan's syndrome

4. Duane's syndrome

5.

Correct Answer: 1.

Question 81: Consider the following statements: Dengue is a protozoan disease transmitted by mosquitoes. Retro-orbital

pain is not a symptom of Dengue. Skin rash and bleeding from nose and gums are some of the symptoms of Dengue.

Which of the statements given above is/arecorrect?

1. 1 and 2

2. 3 only

3. 2 only

4. 1 and 3

5.

Correct Answer: 2.

Question 82: Potts shunt is

1. Rt subclavian artery to rt pulmonary

2. Descending aorta to left pulmonary

3. Left subclavian to left pulmonary

4. Ascending aorta to right pulmonary

5.

Correct Answer: 2.

Question 83: A young lady presents with a history of excessive food intake following which she would induce vomiting.

The diagnosis is most likely to be

1. Binge eating disorder

2. Bulimia nervosa

3. Anorexia Nervosa

4. Obsessive compulsive disorder

5.

Correct Answer: 2.

Question 84: A 40 year old male developed persistent oral ulcers followed by multiple flaccid bullae on trunk and

extremities. Direct examination of a skin biopsy immunofluorescence showed intercellular igG deposits inthe epidermis.

The most probable diagnosis is

1. Pemphigus vulgaris

2. Bullous Pemphigoid

3. Bullous Lupus erythematosus.

4. Epidermolysis bullosa acquisita

5.

Correct Answer: 1.

Question 85: A patient has presented to you with pleural effusion. The best site of aspiration of the fluid would be through

the

1. 5th intercostal space in the midclavicular line

Page 178: FMGE (MCI Screening test)  preparation material by Gankidi Raghavender Reddy

2. 2nd intercostal space close to the sternum

3. 7th intercostal space in mid axillary line

4. 10th intercostal space in paravertebral location

5.

Correct Answer: 3.

Question 86: Deleterious effect of ultrasound on small organs is

1. Ionisation

2. Vacoulation

3. Cavitation

4. Disintegration

5.

Correct Answer: 3.

Question 87: Treaement of choice Tinea infection of nail is

1. Griseofulvin+fluconazole

2. Zinc Oxide.

3. Nystatin

4. Itraconazole

5.

Correct Answer: 1.

Question 88: All of the following are antifungals except

1. Metronidazole

2. Griseofulvin

3. Fluconazole

4. Ketoconazole

5.

Correct Answer: 1.

Question 89: Acetazolamide

Ans 4

1. Produces a metallic taste

2. Rx is associated with perioral paraesthesia

3. Is associated with hyperopia

4. 1 & 2 are true,3 is false

5.

Correct Answer: 4.

Question 90: Which of the following is not a complete sterilization agent

1. glutaraldehyde

2. absolut alcohol

3. hydrogen peroxide

4. sodium hypochlorite

5.

Page 179: FMGE (MCI Screening test)  preparation material by Gankidi Raghavender Reddy

Correct Answer: 2.

Question 91: A 20- year-old man has presented with increased alcohol consumption and sexual indulgence, irritability,

lack of sleep, and not feeling fatigued even on prolonged periods of activity. Activity. All these changes have beenpresent

for 3 weeks . The most likely diagnosis is

1. Alcohol dependence

2. Schizophrenia

3. Mania

4. Impulsive control disorder

5.

Correct Answer: 3.

Question 92: Chancre is a lesion see in

1. Primary syphilis & is painless

2. Secondary syphilis & is painless

3. Secondary syphilis & is painful

4. Primary syphilis & is painful

5.

Correct Answer: 1.

Question 93: Which of the following is seen in ITP

1. Thrombocytosis

2. Increased Prothrombin time

3. Increased Bleeding time

4. Increased clotting time

5.

Correct Answer: 3.

Question 94: All are sign of Left ventricular failure except

1. Basal crepts

2. Dyspnoea

3. Orthopnoea

4. Pedal edema

5.

Correct Answer: 4.

Question 95: Complications of Cirrhosis include

1. Edema and ascites

2. Portal hypertension

3. Liver cancer

4. All are true

5.

Correct Answer: 4.

Question 96: Black deposits (melanin) in liver may be seen in

Page 180: FMGE (MCI Screening test)  preparation material by Gankidi Raghavender Reddy

1. Rotors Syndrome

2. Dubin Johnson Syndrome

3. Gilberts syndrome

4. Crigler Najjar syndrome

5.

Correct Answer: 2.

Question 97: Kalloo, a 24 year old occasional alcoholic has got a change in his behavior. He has become suspicious that

people are trying to conspire against him though his father states that there is no reason for his fears. He is getting

hallucinations of voices commenting on his actions. What is the most probable diagnosis

1. Delerium tremens

2. Alcohol induced psychosis

3. Schizophrenia

4. Delusional disorder

5.

Correct Answer: 3.

Question 98: During cardiopulmonary resusitaton intravenous calcium glouconate is indicated under all for the follwing

circumstances except

1. After I min. of arrest routinely

2. Hypocalcemia

3. Calcium channel blocker toxicity

4. Electromechanical dissociaton

5.

Correct Answer: 1.

Question 99: Diagnostic features of allergic broncho-pulmonary aspergillosis (ABPA) include all of the following except

1. Changing pulmonary infiltrates

2. Peripheral eosinophilia

3. Serum precipitins against Aspergillosis fumigatus

4. Occurance in patients with old cavitary lesions

5.

Correct Answer: 4.

Question 100: Exfoliative dermatitis is seen in all the following except

1. Pityriasis Rosea

2. Pityriasis Rubra pilaris

3. Psoriasis

4. Drug reaction

5.

Correct Answer: 1.

Question 1: Effective in management of complex partial seizures:

Page 181: FMGE (MCI Screening test)  preparation material by Gankidi Raghavender Reddy

1. Gabapentin

2. Lamotrigine

3. Ethosuximide

4. Gabapentin and Lamotrigine

5.

Correct Answer: 4.

Question 2: In the heart, the "slow" responses mediated by this ion:

1. sodium

2. calcium

3. magnesium

4. potassium

5. chloride

Correct Answer: 2.

Question 3: Very short-acting; substrate for tissue esterases

1. A) morphine

2. B) levorphanol

3. C) remifentanil

4. D) fentanyl

5.

Correct Answer: 3.

Question 4: Preferred beta-blocking agent to prevent systolic blood pressure increases associated with direct

laryngoscopy in tracheal intubation:

1. IV propranolol (Inderal)

2. IV labetalol (Trandate, Normodyne)

3. IV esmolol (Brevibloc)

4. IV timolol (Blocadren)

5. IV nadolol (Corgard)

Correct Answer: 3.

Question 5: A chronic alcohol and diazepam abuser is hospitalized for management of withdrawal symptoms. Buspirone,

an anxiolytic, is administered to alleviate withdrawal, but the symptoms continue and worsen. Why?

1. Buspirone (BuSpar) dosage too low.

2. Buspirone (BuSpar) does not exhibit cross-tolerance with sedative-hypnotic/benzodiazepines.

3. Management of alcohol or benzodiazepine withdrawal must be managed with a stronger drug, such as haloperidol.

4. Buspirone (BuSpar) is too rapidly inactivated (first-pass effect) to be useful in managing withdrawal.

5.

Correct Answer: 2.

Question 6: Pilocarpine (Pilocar):

1. dry mouth

2. pupillary dilation

3. increased gastrointestinal tone

Page 182: FMGE (MCI Screening test)  preparation material by Gankidi Raghavender Reddy

4. bronchiolar relaxation

5.

Correct Answer: 3.

Question 7: Reduces preload AND afterload

1. A) nitroprusside sodium (Nipride)

2. B) phentolamine (Regitine)

3. C) captopril (Capoten)

4. D) A & C

5.

Correct Answer: 4.

Question 8: Despite strict dietary control, this condition can induce a marked increase in triglyceride serum concentration

in a patient with primary chylomicronemia:

1. chronic obstructive pulmonary disease

2. pregnancy

3. both

4. neither

5.

Correct Answer: 2.

Question 9: Fentanyl (Sublimaze)/sufentanil (Sufenta) & seizure activity:

1. A) high-dose fentanyl (Sublimaze) or sufentanil (Sufenta): use cautiously if patient has seizure disorder

2. B) Patients receiving anticonvulsants often require more fentanyl (Sublimaze) during surgery than patients not

receiving anticonvulsants

3. C) both

4. D) neither

5.

Correct Answer: 3.

Question 10: Cardiac output related to:

1. heart rate x peripheral resistance

2. stroke volume x peripheral resistance

3. heart rate x arterial pressure

4. stroke volume x heart rate

5.

Correct Answer: 4.

Question 11: Primary mechaism for termination of norepinephrine and epinephrine action:

1. metabolic transformation catalyzed by MAO

2. metabolic transformation catalyzed by COMT

3. diffusion away from the synaptic cleft and uptake at extraneuronal sites

4. reuptake into nerve terminals

5.

Correct Answer: 4.

Page 183: FMGE (MCI Screening test)  preparation material by Gankidi Raghavender Reddy

Question 12: Factors enhancing bupivacaine (Marcaine) toxicity

1. A) pregnancy

2. B) presence of calcium channel blockers

3. C) arterial hypoxemia

4. D) acidosis

5. E) All of the Above

Correct Answer: 5.

Question 13: Ketamine (Ketalar) anesthesia:

1. A) awakening may be associated with bad dreams and hallucinations

2. B) cardiovascular system -- typically depressed due to reduced sympathetic outflow

3. C) useful (with diazepam (Valium)) in management of emergency surgery and certain pediatric radiological

procedures

4. D) A & C

5. E) A, B & C

Correct Answer: 4.

Question 14: Highest local anesthetic blood levels associated with this type of regional anesthesia:

1. A) epidural

2. B) brachial plexus

3. C) sciatic

4. D) intercostal

5. E) caudal

Correct Answer: 4.

Question 15: Saturable transport systems:

1. active transport -- carrier mediated

2. facilitated diffusion -- carrier mediated

3. lipid diffusion

4. 1 & 2

5. None of these

Correct Answer: 4.

Question 16: Reverses respiratory depression cause by opioids:

1. A) naloxone (Narcan)

2. B) nalmefene (Revex)

3. C) naltrexone (ReVia)

4. D) All of the Above

5.

Correct Answer: 4.

Question 17: Antilipid medication -- structural analog of HMG-CoA intermediate

1. cholestyramine (Questran, Questran Light)

2. gemfibrozil (Lopid)

Page 184: FMGE (MCI Screening test)  preparation material by Gankidi Raghavender Reddy

3. pravastatin (Pravachol)

4. niacin (vitamin B3)

5.

Correct Answer: 3.

Question 18: Probable site -- NO as neurotransmitter:

1. neuromuscular junction

2. innervation of the corpora cavernosa

3. basal ganglia

4. Renshaw cell

5.

Correct Answer: 2.

Question 19: Droperidol (Inapsine) + fentanyl (Sublimaze):

1. A) neuroleptic analgesia

2. B) dissociative anesthesia

3. C) both

4. D) neither

5.

Correct Answer: 1.

Question 20: Side effects of this antihypertensive agent includes tachycardia, angina, reversible lupus-like syndrome

1. propranolol (Inderal)

2. mecamylamine (Inversine)

3. hydralazine (Apresoline)

4. diazoxide (Hyperstat)

5.

Correct Answer: 3.

Question 21: Principal ion responsible for action potentials in specialized conducting SA nodal and AV nodal tissues:

1. potassium

2. sodium

3. calcium

4. carbon

5.

Correct Answer: 3.

Question 22: Component(s) of balanced anesthesia may include:

1. A) muscle relaxants

2. B) thiopental (Pentothal)

3. C) midazolam (Versed)

4. D) All of the Above

5.

Correct Answer: 4.

Page 185: FMGE (MCI Screening test)  preparation material by Gankidi Raghavender Reddy

Question 23: The mechanism of action of sodium nitroprusside is

1. Increases cAMP

2. Increases guanylate cyclase

3. Calcium channel blocker

4. K channel opener

5.

Correct Answer: 2.

Question 24: Antiemetic drug with least side effects

1. A) chlorpromazine (Thorazine)

2. B) haloperidol (Haldol)

3. C) hydroxyzine (Atarax,Vistaril)

4. D) tropisetron

5.

Correct Answer: 4.

Question 25: Bradycardia is common after injection of

1. Midazolam

2. Succinyl choline

3. Dopamine

4. Isoprenaline

5.

Correct Answer: 2.

Question 26: Aspirin should not be used in

1. Dysmenorrhea

2. Viral infection

3. Ischemic heart disease

4. Inflammation

5.

Correct Answer: 2.

Question 27: A neutral molecule that can reversibly dissociates into an anion (negatively charged molecule) and a proton

(a hydrogen ion):

1. weak acid

2. weak base

3. 1 & 2

4. None of the above

5.

Correct Answer: 1.

Question 28: Hemodynamic responses in anesthetized patients who had been receiving antihypertensive drug treatment:

1. increased venous return cause by positive-pressure ventilation

2. excessive responses associated with body position changes

Page 186: FMGE (MCI Screening test)  preparation material by Gankidi Raghavender Reddy

3. both

4. neither

5.

Correct Answer: 2.

Question 29: Neurological adverse effect associated with antipsychotic drug use: occurs late in therapy

1. A) Parkinson's syndrome

2. B) Akisthisia

3. C) Acute dystonic reactions

4. D) Tardive dyskinesia

5.

Correct Answer: 4.

Question 30: Which is the antagonist of benzodiazepines

1. Flumazenil

2. Neostigmine

3. Naloxone

4. Sugammadex

5.

Correct Answer: 1.

Question 31: Orthostatic (postural) hypotension

1. beta receptor activation

2. alpha receptor activation

3. alpha receptor blocker

4. dopamine receptor blockade

5.

Correct Answer: 3.

Question 32: An attack of acute pancreatitis is likely to be the first presenting symptom of this lipoprotein disorder:

1. familial combined hyperlipoproteinemia

2. familial ligand-defective ApoB

3. familial lipoprotein lipase or cofactor deficiency

4. familial dysbetalipoproteinemia

5.

Correct Answer: 3.

Question 33: i.v. route of administration; few side effects; effective in treating hypertensive crisis:

1. nifedipine (Procardia, Adalat)

2. nitroprusside sodium (Nipride)

3. nicardipine (Cardene)

4. None of the above

5.

Correct Answer: 2.

Page 187: FMGE (MCI Screening test)  preparation material by Gankidi Raghavender Reddy

Question 34: Antiemetic drug that works through serotonin (subtype 3) receptor blockade

1. A) Chlorpromazine (Thorazine)

2. B) Haloperidol (Haldol)

3. C) Ondansetron (Zofran)

4. D) All of the Above

5.

Correct Answer: 3.

Question 35: Preganglionic fibers terminating on adrenal medullary chromaffin cells release:

1. norepinephrine

2. epinephrine

3. acetylcholine

4. dopamine

5. substance P

Correct Answer: 3.

Question 36: Very large polymeric cationic exchange resins:

1. niacin (vitamin B3)

2. colestipol (Colestid)

3. pravastatin (Pravachol)

4. clofibrate (Abitrate, Atromid-S)

5.

Correct Answer: 2.

Question 37: Phase of the cardiac action potential (in SA nodal cells, for example) that is associated with "diastolic

depolarization":

1. phase 0

2. phase 1

3. phase 2

4. phase 3

5. phase 4

Correct Answer: 5.

Question 38: Maintenance of antihypertensive drug treatment during perioperative time frame:

1. associated with increased systemic blood-pressure and heart rate fluctuations during anesthesia

2. decrease likelihood of arrhythmias

3. since previously effective antihypertensive drug therapy typically should be continued during the perioperative

time frame, the pharmacology of the particular antihypertensive agent need not be considered in the development of

the anesthesia plan.

4. usually, is appropriate to maintaina patient on preoperative diuretics when those drugs are used to reduce blood

pressure

5.

Correct Answer: 2.

Question 39: Calcium channel blocker(s) with predominately vascular effects:

Page 188: FMGE (MCI Screening test)  preparation material by Gankidi Raghavender Reddy

1. diltiazem (Cardiazem)

2. amlodipine (Norvasc)

3. verapamil (Isoptin, Calan)

4. All the above

5.

Correct Answer: 2.

Question 40: What is/are the most hazardous response to sympathetic stimulation?

1. A) tachycardia

2. B) arrhythmias

3. C) hypertension

4. D) A & C

5. E) A, B & C

Correct Answer: 5.

Question 41: Contraindications for morphine use:

1. A) Addison's disease

2. B) acute asthma

3. C) increased ICP

4. D) acute ulcerative colitis

5. E) All of the Above

Correct Answer: 5.

Question 42: Interferes with norepinephrine release:

1. alpha-methyltyrosine by preventing synthesis of a protein that promotes fusion of the vesicle and the presynaptic

membrane

2. bretylium (Bretylol) following a transient stimulation of release by displacement

3. reserpine

4. None of the above

5.

Correct Answer: 2.

Question 43: Effects of muscarinic agonists on the gastrointestinal tract

1. A) reduced intestinal peristalsis

2. B) reduced smooth muscle tone

3. C) reduced contraction amplitude

4. D) all of the above

5. E) none of the above

Correct Answer: 5.

Question 44: Anesthesia in patients with severe intracranial hypertension even after steroids, hyperventilation, and

diuretic administration

1. A) isoflurane (Forane) + nitrous oxide

2. B) enflurane (Ethrane) + thiopental (Pentothal)

3. C) thiopental (Pentothal) + fentanyl (Sublimaze) boluses/infusion

Page 189: FMGE (MCI Screening test)  preparation material by Gankidi Raghavender Reddy

4. D) All of the Above

5.

Correct Answer: 3.

Question 45: Cytochrome P450 induction (increased, amount of P450 enzyme protein) is often associated with:

1. A) increased enzyme synthesis rates

2. B) reduced enzyme degradation rates

3. C) both

4. D) neither

5.

Correct Answer: 3.

Question 46: Succinylcholine (Anectine) should be avoided in hemiplegic/paraplegic patients because:

1. A) duration of action may be excessive

2. B) bradycardia may be difficult to manage

3. C) hyperkalemia may occur

4. D) renal excretion may be compromised

5.

Correct Answer: 3.

Question 47: Is there a way that the duration of action of 2% lidocaine (Xylocaine) can be significantly increased?

1. No, but one could use a signficantly higher initial concentration to achieve longer action.

2. add a vasoconstrictor to the lidocaine solution

3. both

4. neither

5.

Correct Answer: 2.

Question 48: In reference to ionic gradients, the term "electrogenic" means:

1. that a large membrane capacitance is present

2. that the membrane is relatively permeable to potassium

3. that net current flows as a result of ionic transport

4. that current flows independent of membrane voltage

5.

Correct Answer: 3.

Question 49: Example(s) of second messenger effect(s):

1. A) increases in cAMP intracellular concentration

2. B) changes in intracellular calcium concentration

3. C) phosphoinositide effects

4. D) all the above

5.

Correct Answer: 4.

Question 50: Pharmacological action(s) of dopamine (Intropin):

Page 190: FMGE (MCI Screening test)  preparation material by Gankidi Raghavender Reddy

1. positive inotropism

2. promotes myocardial norepinephrine release

3. increases glomerular filtration rates (low-dose)

4. vasoconstriction by alpha-1 receptor activation (high-dose)

5. all the above

Correct Answer: 5.

Question 51: Drug-induced severe Parkinsonism associated with the inadvertent synthesis of this pro-toxin.

1. A) selegiline

2. B) MPTP

3. C) primidone

4. D) amantadine

5.

Correct Answer: 2.

Question 52: Considering enflurane (Ethrane), halothane (Fluothane), and isoflurane (Forane) -- cardiac output is well

maintained --

1. A) enflurane (Ethrane)

2. B) halothane (Fluothane)

3. C) isoflurane (Forane)

4. D) All of the Above

5.

Correct Answer: 3.

Question 53: Drug (medicine) of first choice for absence seizures

1. primidone

2. ethosuximide

3. phenytoin

4. i.v. diazepam

5.

Correct Answer: 2.

Question 54: Adverse effects associated with guanethidine and guanadrel:

1. A) hypotension (symptomatic)

2. B) male sexual dysfunction

3. C) diarrhea

4. D) A & C

5. E) A, B & C

Correct Answer: 4.

Question 55: Primary antihypertensive effect due to nitric oxide mediation of smooth muscle relaxation.

1. atropine

2. nitroprusside sodium (Nipride)

3. mecamylamine (Inversine)

Page 191: FMGE (MCI Screening test)  preparation material by Gankidi Raghavender Reddy

4. captopril (Capoten)

5.

Correct Answer: 2.

Question 56: Reason that dopamine itself is not used to treat in Parkinson's disease:

1. too expensive

2. the problem is cholinergic in nature

3. dopamine does not cross the blood-brain barrier

4. levodopa has a higher affinity for the D2 receptor

5.

Correct Answer: 3.

Question 57: Angiotensin II receptor antagonist:

1. captopril (Capoten)

2. losartin (Cozaar)

3. methyldopa (Aldomet)

4. phenoxybenzamine (Dibenzyline)

5.

Correct Answer: 2.

Question 58: Concerning drug-plasma protein binding:

1. A) Drugs are seldom bound to plasma proteins

2. B) volume of distribution is not influenced by protein binding

3. C) diazepam is an example of the drug that is minimally protein-bound

4. D) drug clearance is influenced by the extent of protein binding

5.

Correct Answer: 4.

Question 59: Plasma half life of drug indicates

1. Maximum effective dose of drug

2. Maximum lethal effect

3. Potency of drug

4. Dose frequency interval

5.

Correct Answer: 4.

Question 60: Hyperventilation and ICP reduction:

1. A) effective for both subacute and acute management

2. B) decreases CBF by cerebral vasodilation

3. C) effectiveness independent of cerebrovascular CO2 reactivity

4. D) All of the Above

5.

Correct Answer: 1.

Question 61: Location(s) of cholinergic synaptic sites:

Page 192: FMGE (MCI Screening test)  preparation material by Gankidi Raghavender Reddy

1. A) neuromuscular junction

2. B) autonomic effector sites innervated by post-ganglionic sympathetic fibers

3. C) some CNS synapses

4. D) A & C

5. E) B & C

Correct Answer: 4.

Question 62: Stress developed in the wall of ventricle during ejection -- dependencies:

1. ventricular dimension

2. aortic pressure

3. both

4. neither

5.

Correct Answer: 3.

Question 63: Corneal microdeposits, blurred vision, and photophobia:

1. adenosine (Adenocard)

2. encainide (Enkaid)

3. amiodarone (Cordarone)

4. diltiazem (Cardiazem)

5.

Correct Answer: 3.

Question 64: Reflex bradycardia secondary to an abrupt increase in blood pressure may be blocked by:

1. atropine

2. mecamylamine (Inversine)

3. both

4. neither

5.

Correct Answer: 3.

Question 65: Major neurotransmitter released at end organ effectors of the thoracolumbar division of the autonomic

nervous system:

1. dopamine (Intropin)

2. epinephrine

3. norepinephrine

4. acetylcholine

5.

Correct Answer: 3.

Question 66: Combination drug therapy useful in management of absence seizures:

1. Propofol and chlorpromazine

2. Diazepam and phenytoin

3. Phenytoin and ethosuximide

Page 193: FMGE (MCI Screening test)  preparation material by Gankidi Raghavender Reddy

4. Fentanyl and propofol

5. Thiopental and diazepam

Correct Answer: 3.

Question 67: Examples of "high-extraction" drugs -- related to hepatic drug clearance:

1. A) morphine

2. B) meperidine

3. C) verapamil

4. D) diltiazem

5. E) all the above

Correct Answer: 5.

Question 68: Types of glaucoma:

1. primary

2. secondary

3. congenital

4. all the above

5.

Correct Answer: 4.

Question 69: Examples of epinephrine metabolic effects

1. insulin secretion reduced by beta2 adrenergic receptor activation

2. glucagon secretion: diminished by beta adrenergic receptor activation

3. free fatty acids: increased

4. minimal calorigenic effect

5. glycolysis inhibition

Correct Answer: 3.

Question 70: itric oxide mediates this effect on vascular smooth muscle:

1. A) smooth muscle relaxation

2. B) smooth muscle contraction

3. C) no effect

4. N/A

5.

Correct Answer: 1.

Question 71: Concerning transfer of basic drugs such as nonionized local anesthetics from mother to fetus is:

1. A) fetal pH is higher than maternal pH

2. B) in fetal distress alkalosis contributes to local anesthetic accumulation

3. C) concerning maternal blood: fetal blood -- gradient is maintained for continual local anesthetic transfer from

maternal circulation to fetal circulation

4. D) None of the Above

5.

Correct Answer: 3.

Page 194: FMGE (MCI Screening test)  preparation material by Gankidi Raghavender Reddy

Question 72: Isoflurane (Forane) and blood pressure:

1. A) neonates: increase blood-pressure, increased heart rate

2. B) elderly patients: reduced heart rate responses

3. C) younger patients: reduced heart rate

4. D) direct pharmacological effects

5.

Correct Answer: 2.

Question 73: Drug not appropriate for antagonist-assisted neuromuscular-blockade reversal, because the dosage

requirement is excessive:

1. A) edrophonium (Tensilon)

2. B) physostigmine (Antilirium)

3. C) neostigmine (Prostigmin)

4. D) pyridostigmine (Mestinon)

5. E) vecuronium (Norcuron)

Correct Answer: 2.

Question 74: Competitive inhibition:

1. at high concentrations of agonist, a maximal physiological responds is not possible even in the presence of the

antagonist

2. with competitive inhibition, the dose effect curve is shifted to the right

3. is irreversible

4. is produced by antagonists that have the ability to activate receptors

5.

Correct Answer: 2.

Question 75: Which of the following is least likely to cause interstitial nephritis on chronic use

1. Methicillin

2. Heparin

3. Ampicillin

4. Cephalothin

5.

Correct Answer: 2.

Question 76: Factors that argue against initial use of benztropine in a patient with mild Parkinson's disease:

1. A) old age

2. B) prostatic hypertrophy

3. C) narrow angle glaucoma

4. D) All of the Above

5.

Correct Answer: 4.

Question 77: Which type of glaucoma response to anticholinesterase treatment?

1. primary

Page 195: FMGE (MCI Screening test)  preparation material by Gankidi Raghavender Reddy

2. secondary

3. third

4. congenital

5.

Correct Answer: 1.

Question 78: Neurogenic hypertension:

1. pheochromocytoma

2. aortic coarctation

3. familial dysautonomia

4. chronic pyelonephritis

5.

Correct Answer: 3.

Question 79: Other than TB,rifampicin can be used for

1. Drug resistant Leprosy

2. Neiserria infections

3. Legionella pneumonia

4. All are correct

5.

Correct Answer: 4.

Question 80: Glaucoma category responding to anticholinesterase treatment

1. A) primary (narrow angle -- acute, congestive)

2. B) secondary (aphakic --no lens; following cataract surgery)

3. C) congenital

4. D) None of the Above

5.

Correct Answer: 1.

Question 81: ANS (autonomic nervous system) manifestations in Parkinson's disease:

1. A) tremor

2. B) rigidity

3. C) micrographia

4. D) sialorrhea

5.

Correct Answer: 4.

Question 82: Normal cardiac impulse propagation sequence:

1. AV node, His-Purkinje, ventricle, SA node

2. SA node, His-Purkinje, AV node, ventricle

3. SA node, AV node, His-Purkinje, ventricle

4. AV node, SA node, His-Purkinje, ventricle

5.

Correct Answer: 3.

Page 196: FMGE (MCI Screening test)  preparation material by Gankidi Raghavender Reddy

Question 83: Enzyme catalyzing the conversion of norepinephrine to epinephrine:

1. A) dopamine beta-hydroxylase

2. B) phenylethanolamine N-methyltransferase

3. C) tyrosine hydroxylase

4. D) dopa decarboxylase

5.

Correct Answer: 2.

Question 84: which antimuscarinic agent might be used in combination with an anticholinesterase when desiring reversal

of neuromuscular-blockade and opioid-based maintenance anesthesia has been used:

1. edrophonium (Tensilon)

2. high-dose atropine (10-15 ug/kg)

3. neostigmine (Prostigmin)

4. None of the above

5.

Correct Answer: 2.

Question 85: Most potent at beta adrenergic receptors

1. epinephrine

2. isoproterenol (Isuprel)

3. norepinephrine

4. dopamine

5.

Correct Answer: 2.

Question 86: Depolarization in the specialized conduction system, the SA. and AV nodal tissue, is mediated by this ion

movement:

1. sodium

2. calcium

3. potassium

4. chloride

5. magnesium

Correct Answer: 4.

Question 87: Most common side effect of zidovudine is

1. Hemolytic Anemia

2. Myopathy

3. Bone marrow depression

4. Hepatotoxicity

5.

Correct Answer: 3.

Question 88: Tizanidine (Zanaflex) pharmacology:

1. advantages for patients with spasticity (several types)

Page 197: FMGE (MCI Screening test)  preparation material by Gankidi Raghavender Reddy

2. facilitates nociceptor transmission in dorsal horn

3. significantly more efficacious compared to diazepam (Valium)

4. adverse effects: hypertension

5.

Correct Answer: 1.

Question 89: Drug-drug interaction involving anticonvulsant agents and agents used in anesthesia protocols:

1. A) phenytoin (Dilantin): may cause increased patient resistance to nondepolarizing muscle relaxant action

2. B) Carbamazepine (Tegretol) because increased patient resistance to the action of nondepolarizing muscle

relaxants

3. C) both

4. D) neither

5.

Correct Answer: 3.

Question 90: Produces epinephrine reversal, converting a pressor response to a deep pressor response:

1. losartin (Cozaar)

2. propranolol (Inderal)

3. metoprolol (Lopressor)

4. phentolamine (Regitine)

5. clonidine (Catapres)

Correct Answer: 4.

Question 91: Bond type that is seen in some drug-receptor interactions and tends to very strong, often nearly irreversible:

1. hydrophobic

2. electrostatic

3. covalent

4. 1 & 2

5. 1 & 3

Correct Answer: 3.

Question 92: A seizure which varies in its presentations depending upon which part of the cortex is affected; last between

20 and 60 seconds; does not impair the patient's state of consciousness:

1. Complex partial

2. Simple partial

3. Absence

4. Myoclonic

5.

Correct Answer: 2.

Question 93: Utility of loop diuretics and congestive heart failure:

1. A) rapid increase in venous capacitance

2. B) rapid and substantial natriuresis

3. C) both

4. D) neither

5.

Page 198: FMGE (MCI Screening test)  preparation material by Gankidi Raghavender Reddy

Correct Answer: 3.

Question 94: Centrally-acting antihypertensive drug

1. nitroprusside sodium (Nipride)

2. clonidine (Catapres)

3. methoxamine (Vasoxyl)

4. captopril (Capoten)

5.

Correct Answer: 2.

Question 95: Highest nicotinic receptor activity among choline esters:

1. acetylcholine

2. atropine

3. methacholine (Provocholine)

4. carbachol

5.

Correct Answer: 4.

Question 96: True about Isoniazid include all except

1. Kills intracellular bacilli

2. Kills dormant bacilli

3. Most dangerous complication is peripheral neuritis

4. It penetrates all body tissues

5.

Correct Answer: 2.

Question 97: About succinyl choline all are true except

1. Neostigmine reversal

2. Persistant depolarization

3. Short acting

4. Metabolised by pseudocholinesterase

5.

Correct Answer: 2.

Question 98: Probable cause of myasthenia gravis:

1. A) defect in acetylcholine synthesis

2. B) decreased receptor turnover

3. C) binding of anti-nicotinic receptor antibodies to the nicotinic receptor

4. D) None of the Above

5.

Correct Answer: 3.

Question 99: EC50 mainly reflexs a drug's:

1. A) maximal effect

Page 199: FMGE (MCI Screening test)  preparation material by Gankidi Raghavender Reddy

2. B) potency

3. C) lethality

4. D) ease of elimination

5. E) safety

Correct Answer: 2.

Question 100: Metoclopramide (Reglan):

1. A) dopamine antagonist

2. B) causes pylorus constriction

3. C) causes duodenal constriction

4. D) increases gastroesophageal sphincter tone

5.Correct Answer: 4.

Question 1: Inability to understand, speak or read is

1. Dyslexia

2. Aphasia

3. Disconnection syndrome

4. Ataxia

5.

Correct Answer: 1.

Question 2: Synaptic conduction is mostly orthodromic because

1. Dendrities cannot be depolarized

2. Once repolarised, an area cannot be depolarized

3. The strength of antidromic impulse is less in presynaptic ter minal

4. Chemical mediator is located only in the presynpatic terminal

5.

Correct Answer: 4.

Question 3: Nitrogen Narcosis is caused due to?

1. Increased susceptibility of Neurons for Nitrogens

2. Nitrogen Inhibits Dismutase Enzyme

3. Decrease in Oxygen free Radicals

4. Increased production of Nitrous Oxide

5.

Correct Answer: 1.

Question 4: Measurement of intravascular pressure by a pulmonary catheter should be done

1. At end expiration

2. At peak of inspiration

3. During mid expiration

4. During mid inspiration

5.

Page 200: FMGE (MCI Screening test)  preparation material by Gankidi Raghavender Reddy

Correct Answer: 2.

Question 5: The system internationale (SI) unit for blood pressure is

1. Torr

2. mm Hg

3. K Pa

4. cm H2O

5.

Correct Answer: 3.

Question 6: Sertoli cells in the testis have receptors for:

1. FSH

2. LH

3. Inhibin

4. None of these

5.

Correct Answer: 1.

Question 7: Kernicterus is due to

1. Albumin bound bilirubin

2. Bilirubin diglucuroinde

3. Bilirubin monoglucuronide

4. Free bilirubin

5.

Correct Answer: 4.

Question 8: The temperature at which the MgO reacts with carbon powder in the presence of chlorine gas is ______.

1. 100ºC

2. 500ºC

3. 600ºC

4. 1000ºC

5.

Correct Answer: 4.

Question 9: Bicarbonate is maximally absorbed from

1. PCT

2. DCT

3. Collecting duct

4. Thick ascending loop of Henle

5.

Correct Answer: 1.

Question 10: Normal testicular development requires

1. XY

Page 201: FMGE (MCI Screening test)  preparation material by Gankidi Raghavender Reddy

2. XX

3. Y chromosome

4. X chromosome

5.

Correct Answer: 1.

Question 11: Positive feedback action of estrogen for inducing lulteinizing hormone surge is associated with one of the

follwing steroid hormone ratios in peripheral circulation

1. High estrogen: low progesterone

2. Low estrogen: high progesterone

3. Low estrogen: low progesterone

4. High estrogen: high progesterone

5.

Correct Answer: 1.

Question 12: As we go down in the period table from beryllium to radium, the principal quantum number of valence

electrons increases from __________ to _____

1. 2º, 8

2. 3º, 7

3. 2º, 7

4. 4º, 7

5.

Correct Answer: 3.

Question 13: Increased cerebral blood flow:

1. A) increases the amount of intracranial blood

2. B) decreases the amount of intracranial blood

3. C) increases ICP under all conditions

4. D) decreases ICP under all conditions

5.

Correct Answer: 1.

Question 14: The correct sequence of cell cycle is

1. G0-G1-S-G2-M

2. G0-G1-G2-S-M

3. G0-M-G2-S-G1

4. G0-G1-S-M-G2

5.

Correct Answer: 1.

Question 15: who is the father of blood groups?

1. Landsteiner

2. Paul berg

3. Leibig

4. Bateson

5.

Page 202: FMGE (MCI Screening test)  preparation material by Gankidi Raghavender Reddy

Correct Answer: 1.

Question 16: Cerebellar Ataxia is characterized by all except

1. Resting tremor

2. Dysdiadokokinesis

3. Ataxia

4. Hypotonia

5.

Correct Answer: 1.

Question 17: Avoiding response is due to

1. Ispilateral frontal lobe lesions

2. Contra lateral frontal lobe lesions

3. Ipsilateral parietal lobe lesions

4. Contralateral parietal lobe lesions

5.

Correct Answer: 2.

Question 18: Early stage of trauma is characterized by

1. Catabolism

2. Anabolism

3. Glycogenesis

4. Gluconeogenesis.

5.

Correct Answer: 4.

Question 19: The formula used to calculate the diameter of a wire (When there is no zero error)

1. P.S.R + H.S.R

2. P.S.R. + H.S.R. X L.C

3. P.S.R. +(H.S.R. X L.C.) X error

4. P.S.R +(H.S.R X L.C *correction

5.

Correct Answer: 2.

Question 20: Which of the following is the correct order for Pathway a sperm

1. Straight tubules - Rete testis - Efferent tubules

2. Rete Tubules - Efferent Tubules - Straight Tubule

3. Efferent Tubule - Rete Testis - Straight Tubules

4. Straight Tubule - Efferent Tubules - Rete Tubules

5.

Correct Answer: 1.

Question 21: Polydipsia, polyuria, increased dilution of urine is due to deficiency of

1. Vasopressin

Page 203: FMGE (MCI Screening test)  preparation material by Gankidi Raghavender Reddy

2. Insulin

3. Glucagon

4. All of the above

5.

Correct Answer: 1.

Question 22: The weight of 400 gram stone is

1. 0.041N

2. 0.4

3. 3920N

4. 3.9N

5.

Correct Answer: 4.

Question 23: The electromagnetic radiations used in RADAR

1. Radio Waves

2. Micro Waves

3. X rays

4. Y rays

5.

Correct Answer: 2.

Question 24: PR interaval is

1. 0.12- 0.16 seconds

2. 0.3 – 0.4 sec

3. 0.6 – 0.8 sec

4. 0.12 – 4 sec

5.

Correct Answer: 1.

Question 25: Which of the following is true regarding a system which favors oscillatory responses

1. Proportional component

2. Has a greater gain

3. Has a lesser gain

4. Positive feedback system

5.

Correct Answer: 2.

Question 26: In the presence of vasopressin the greatest fraction of filtered water is re-absorbed which part of the

nephron

1. Proximal tubule

2. Distal tubule

3. Loop of Henle

4. Collecting duct

5.

Page 204: FMGE (MCI Screening test)  preparation material by Gankidi Raghavender Reddy

Correct Answer: 1.

Question 27: In a intrinsic semi-conductor, the Fermi level is __________

1. Near the conduction bond

2. Near valence bond

3. in the middle of conduction bond and valence bond

4. within the conduction bond

5.

Correct Answer: 3.

Question 28: Anabolic hormones are all except

1. Cortisol

2. Testosterone

3. Growth Hormone

4. ) Insulin

5.

Correct Answer: 1.

Question 29: If a screw gauge moves 1.mm distance in two rotations then the pitch of the screw gauge is _______

1. 1.mm

2. 2.mm

3. 0.5mm

4. 3.mm

5.

Correct Answer: 3.

Question 30: Decreased O2 carrying capacity & Normal PO2 is seen is

1. Hypoxic hypoxia

2. Anemic hypoxia

3. Histotoxic hypoxia

4. Stagnant hypoxia

5.

Correct Answer: 2.

Question 31: Posterior pituitary hormones are

1. ADH and Oxytocin

2. TRH and FSH

3. Vasopressin and CRH

4. Oxyticin and ACTH

5.

Correct Answer: 1.

Question 32: Capnography basically monitors

1. Central venous pressure

Page 205: FMGE (MCI Screening test)  preparation material by Gankidi Raghavender Reddy

2. Blood pressure during anesthesia

3. Concentration of exhaled CO2

4. Concentration of inhaled O2

5.

Correct Answer: 3.

Question 33: Starling's Law and related matters:

1. As blood volume returning to the hard increases,ventricular dilation is reduced

2. Generally, increase preloaded an increase contractility lead to decreased stroke volume

3. According to Starling's Law, increased ventricular stretch usually leads to increased myocardial contractility

4. all of the above are true

5.

Correct Answer: 3.

Question 34: Double barr body are seen in

1. XX

2. XXY

3. XY

4. XXX

5.

Correct Answer: 4.

Question 35: All of the following transport processes follow'saturation kinetics' except

1. Facilitated diffusion

2. Na+-Ca2+ exchanger

3. Simple diffusion

4. Na+ coupled active transport

5.

Correct Answer: 3.

Question 36: Non shivering thermogenesis in adults is due to

1. Thyroid hormone

2. Brown fat between the shoulders

3. Noradrenaline

4. Muscle metabolism

5.

Correct Answer: 1.

Question 37: True about Action potential is

1. Decremental phenomenon

2. Doesn’t obey all or none phenomenon

3. K+ goes from ECF to ICF

4. Threshold stimulus is required

5.

Correct Answer: 4.

Page 206: FMGE (MCI Screening test)  preparation material by Gankidi Raghavender Reddy

Question 38: All of the following statements regarding bio availability of a drug are true except

1. It is the proportion (fraction) of unchanged drug that reaches the systemic circulation

2. Bioavailability of an orally administered drug can be calculated by comparing the Area Under Curve (0- ) after oral

and intravenous (iv) administration

3. Low oral bioavailability always and necessarily mean poor absorption

4. Bioavailability can be determined from plasma concentration or urinary excretion data

5.

Correct Answer: 3.

Question 39: Following are the reatures of corticospinal involvement except

1. Cog-wheel rigidity

2. Spasicity

3. Planter extensor response

4. Exaggerated deep tenedon reflexes

5.

Correct Answer: 1.

Question 40: Which of the following is true regarding renal clearance

1. Renal blood flow is 400 ml/minute

2. Inulin is very useful for determination of renal blood flow

3. Para amino hippuric acid is used for measuring renal plasma flow

4. Normal value of inulin clearance is 1.25 litres/minute

5.

Correct Answer: 3.

Question 41: Mechanism of Action (MOA) of Nitric Oxide is

1. cox 1 inhibition

2. cox 2 inhibition

3. lipo-oxygenase inhibition

4. phospholipase inhibition

5.

Correct Answer: 2.

Question 42: Regarding nitric oxide, false is

1. Seen in the lung of smokers

2. Increases cAMP levels

3. Used to treat hypertension

4. None of these

5.

Correct Answer: 2.

Question 43: Absorption of H2O in PCT is characteristic by all except

1. Vasopressin dependent

2. 60% of water is absorbed

Page 207: FMGE (MCI Screening test)  preparation material by Gankidi Raghavender Reddy

3. Depends on solutes

4. Vasopressin independent

5.

Correct Answer: 1.

Question 44: Which of the following cell type , can undergo both hypertrophy and hyperplasia

1. Endometrial gland cell

2. Neuron

3. Smooth muscle

4. Hepatocytes

5.

Correct Answer: 1.

Question 45: A lesion of ventrolateral part of spinal cord will lead to loss (below the level of lesion) of

1. Pain sensation on the ipsilateral side

2. Proprioception on the contralateral side

3. Pain sensation on the contralateral side

4. Propriception on the ispilateral side

5.

Correct Answer: 3.

Question 46: One unit of fresh blood raises the HB% concentration by

1. 0.1gm%

2. 1 gm%

3. 2 gm%

4. 2.2 gm%

5.

Correct Answer: 2.

Question 47: The time taken by the freely falling body to touch the ground is called

1. time of ascent

2. time of flight

3. time of descent

4. none

5.

Correct Answer: 3.

Question 48: Parasympathetic stimulation of tissues in walls of bronchioles causes

1. Bronchoconstriction

2. Bronchodilation

3. Increased blood flow

4. Dilatation of alveoli

5.

Correct Answer: 1.

Page 208: FMGE (MCI Screening test)  preparation material by Gankidi Raghavender Reddy

Question 49: As a part of space-research program, a physiologist was asked to investigate the effect of flight-induced

stress on blood pressure. Accordingly, the blood pressure of the cosmonauts were to be measured twice: once before the

take-off, and once after the spacecraft entered the designated orbit around the earth. For a proper comparison, the pre-

flight blood pressure should be recorded in

1. The lying down position

2. The sitting position

3. The standing position

4. Any position, as long as the post-flight recording is made in the same position

5.

Correct Answer: 1.

Question 50: Vitamin B12 and folic acid supplementation in megaloblastic anemia leads to the improvement of anemia

due to

1. Increased DNA synthesis in bone marrow

2. Increased Hemoglobin production

3. Erythroid hyperplasia

4. Increased iron absorption

5.

Correct Answer: 1.

Question 51: Which of the following statements is true for excitatory postsynaptic potentials [EPSP];

1. Are self propagating

2. Show all or none response

3. Are proportional to the amount of transmitter released by the presynaptic neuron

4. Are inhibitory at presynaptic terminal

5.

Correct Answer: 3.

Question 52: Frequency of Alpha waves is

1. 4-6 Hz

2. > 13 Hz

3. 8-13 Hz

4. 2-4 Hz

5.

Correct Answer: 3.

Question 53: All of the following statements are correct about potassium balance, except

1. Most of potassium is intracellular

2. Three quarter of the total body potassium is found in skeletal muscle

3. Intracellular potassium is released into extra-cellular space in response to severe injury

4. Acidosis leads to movement of potassium from extracellular to intracellular fluid compartment

5.

Correct Answer: 4.

Question 54: Minority charge carries in n-type semiconductor _________

Page 209: FMGE (MCI Screening test)  preparation material by Gankidi Raghavender Reddy

1. electrons

2. holes

3. positive ions

4. do not exist

5.

Correct Answer: 2.

Question 55: Function of cochlea is

1. Auditory function

2. Nerve deafness

3. Basilar membrane is tuned

4. Equilibrium is maintained

5.

Correct Answer: 1.

Question 56: In metabolic acidosis, which of the following changes are seen

1. Increased K+ excretion

2. Increased K+ reabsorption from tubules

3. Increased Na+ excretion

4. Increased Na+ reabsorption

5.

Correct Answer: 2.

Question 57: Tropomyosin is involved in

1. Helps in the fusion of actin and myosin

2. Covers myosin and prevents attachment of actin and myosin

3. Slides over myosin

4. Causes Ca2+ release

5.

Correct Answer: 2.

Question 58: Salivary secretion does all except

1. Lubrication

2. Taste sensation

3. Digestion of carbohydrates

4. Digestion of proteins

5.

Correct Answer: 4.

Question 59: An increase in which of the following parameters will shift the O2 dissociation curve to the left

1. Temperature.

2. Partial pressure of CO2

3. 2,3 DPG concentration

4. Oxygen affinity of haemoglobin

5.

Page 210: FMGE (MCI Screening test)  preparation material by Gankidi Raghavender Reddy

Correct Answer: 4.

Question 60: Sudden decrease in serum calcium is associated with

1. Increased thyroxin and PTH

2. Increased phosphate

3. Increased sensitivity of muscle and nerve

4. Cardiac conduction abnormality

5.

Correct Answer: 3.

Question 61: Neurotransmitter at the sympathic Ganglion is

1. Adrenaline

2. Noradrenaline

3. Acetylcholine

4. Dopamine

5.

Correct Answer: 3.

Question 62: Absorption of vitamin D in intestine is helped by

1. Calcium

2. Potassium

3. Magnesium

4. Iron

5.

Correct Answer: 1.

Question 63: A low lander goes to the mountains. When he reaches about 50,000 feet high, he develops dyspnea. Which

of the following is the correct explanation for his symptom

1. Carbon dioxide wash out

2. O2 deficiency

3. Increased blood flow to the pulmonary tissues

4. CNS depression

5.

Correct Answer: 1.

Question 64: According to myogenic hypothesis of renal autoregulation, the afferent arterioles contract in response to

strech induced by

1. No release

2. Noradrenaline release

3. Opening of Ca2+ channels

4. Adenosine release

5.

Correct Answer: 3.

Question 65: Ablation of the "somatosensory area 1" of the cerebral cortex leads to

Page 211: FMGE (MCI Screening test)  preparation material by Gankidi Raghavender Reddy

1. Total loss of pain sensation

2. Total loss of touch sensation

3. Loss of tactile localization but not two point discrimination

4. Loss of tactile localization and two point discrimination

5.

Correct Answer: 4.

Question 66: Glucose mediated release of insulin is mediated through

1. ATP dependent K channels

2. cAMP

3. Carrier modulation

4. Receptor phosphorylation

5.Correct Answer: 1.

Question 1: A patient presented with a 3.5 cms size lymph node enlargement, which was hard and presented in submandibular region. Examination of the head and neck did not yield any lesion. The next investigation to be done

1. CXR

2. Triple endoscopy

3. Supravital staining of oral mucosa

4. Laryngoscopy

5.

Correct Answer: 2.

Question 2: A 50-year old male. Working as a hotel cook, has four dependent family members. He has been diagnosed

with an early stage squamous cell cancer of anal canal. He has more than 60% chances of cure. The best treatment option

is

1. Abdomino-perineal resection

2. Combined surgery and radiotherapy

3. Combined chemotherapy and radiotherapy

4. Chemotherapy alone

5.

Correct Answer: 3.

Question 3: A patient on same evening after thyroidectomy has swelling and difficulty breathing. Next step of

management will be

1. Open immediatly

2. Intubate oro tracheally

3. Wait and watch

4. Oxygen by mask

5.

Correct Answer: 1.

Question 4: Most common cause of aneurysm of abdominal aorta is

Page 212: FMGE (MCI Screening test)  preparation material by Gankidi Raghavender Reddy

1. Trauma

2. Atherosclerosis

3. Syphilis

4. Cysitic medial necrosis

5.

Correct Answer: 2.

Question 5: Post cricoid carcinoma of the oesophagus are best treated by

1. Radiotherapy

2. Total oesophagectomy

3. Pharyngolaryngectomy with gastric transposition

4. Intubation through growth

5.

Correct Answer: 3.

Question 6: Pitutary regoin can be best viewed by

1. Plain AP view

2. Caldwell

3. Townes

4. Basal view

5.

Correct Answer: 3.

Question 7: Isotope selectively concentrated in abscess cavities

1. Gallium

2. Technitium

3. Selenium

4. Chromium

5.

Correct Answer: 1.

Question 8: Neural tube defects occur during gestational age

1. 18-22 days

2. 26-30 days

3. 22-26 days

4. 30-34 days

5.

Correct Answer: 2.

Question 9: The muscle pair which rotates radius without flexion and extension at arm is

1. Pronator quadratus and pronator teres

2. Supinator and Anconeus

3. Bachioradialis and brachialis

4. Triceps and Biceps brachii

5.

Page 213: FMGE (MCI Screening test)  preparation material by Gankidi Raghavender Reddy

Correct Answer: 2.

Question 10: A lady presents with a history of fracture radius, which was put on plaster of paris cast for 4 weeks. After

that she developed swelling of hands with shiny skin.What is the most likely diagnosis

1. Rupture of extensor policis longus tendon

2. Myositis ossificans

3. Reflex sympathetic dystrophy

4. Malunion

5.

Correct Answer: 3.

Question 11: The Hunterian Ligature operation is performed for

1. Varicose veins

2. Arteriovenous fistulae

3. Aneurysm

4. Acute ischemia

5.

Correct Answer: 3.

Question 12: The most common malignancy after cholecystectomy is of

1. Colon

2. Stomach

3. Pancreas

4. Ileum

5.

Correct Answer: 2.

Question 13: All the following indicates early gastric cancer except

1. Involvement of mucosa

2. Involvement of mucosa and submucosa

3. Involvement of mucosa, submucosa and muscularis

4. Involvement of mucosa, submucosa and adjacent lymph nodes

5.

Correct Answer: 3.

Question 14: Trans-cranial Doppler Ultrasonography(TCD) uses all of the following windows except

1. Trans-temporal

2. Trans-orbital

3. Foramen magnum

4. Trans-frontal

5.

Correct Answer: 4.

Question 15: In a motor vehicle accident the seat belt leads to following EXPECTED

Page 214: FMGE (MCI Screening test)  preparation material by Gankidi Raghavender Reddy

1. Reduced incidence of severe thoracic injury

2. Occurrence of small intestine and mesenteric injury

3. Increased severity of decelerating head injury

4. Trauma to major intra abdominal

5.

Correct Answer: 4.

Question 16: After appendicectomy, fecal fistula may be due to

1. Crohn’s disease

2. Ulcerative colitis

3. TB

4. Amebic colitis

5.

Correct Answer: 2.

Question 17: In a patient having heavy dense bones, penetration is best achieved by

1. Increase in mA

2. Increase in kVp

3. Increased exposure time

4. Increased developing time

5.

Correct Answer: 2.

Question 18: Which type of hypospadias does not need any treatment

1. Galndular

2. Coronal

3. Penile

4. Peno-scrotal

5.

Correct Answer: 1.

Question 19: On her third of hospitalization, a 70 year old woman who is being treated for acute cholecystitis develops

increased pain and tenderness in the right upper quadrant with a palpable mass. Her temperature rises to 104 F and her BP

falls to 80/60.Heametamesis and malena ensue and petechiae are noted. Laboratory studies reveal thrombocytopenia,

prolonged PT, and decreased fibrinogen level. The most important step in the correction of this patients coagulopathy

1. Administration of heparin

2. Administration of fresh frozen plasma

3. Administration of Epsilon amino caproic acid

4. Exploratory laparotomy

5.

Correct Answer: 4.

Question 20: The treatment modality of achalasia which has the maximum probability of causing a recurrence is

1. Pneumatic dilatation

2. Laparoscopic myotomy

Page 215: FMGE (MCI Screening test)  preparation material by Gankidi Raghavender Reddy

3. Botulinum toxin

4. Open surgical myotomy

5.

Correct Answer: 3.

Question 21: All the following are correct about radiological evaluation of a patient with Cushing’s syndrome except

1. Adrenal CT scan distinguishes adrenal cortical hyperplasia from an adrenal tumor

2. CT of sella tursica is diagnostic when a pituitary tumor is present

3. MRI of the adrenals may distinguish adrenal adenoma from carcinoma

4. Petrosal sinus sampling is the best way to distinguish tumor from an ectopic ACTH producing tumor

5.

Correct Answer: 2.

Question 22: Most physiological treatment of duodenal ulcer is

1. Highly selective vagotomy

2. Antrectomy

3. Vagotomy & gastrojejunostomy

4. Gastrojejunostomy

5.

Correct Answer: 1.

Question 23: A 30 year old male was brought to the casualty following a road traffic accident. His physical examination

revealed that his right lower limb was short, internally rotated, and flexed and adducted at the hip. The most likely

diagnosis is

1. Posterior dislocation of hip

2. Central Fracture dislocation of hip

3. Trochanteric fracture

4. Fracture neck of femur

5.

Correct Answer: 1.

Question 24: Incontinence in urinary tract infection is

1. False

2. True

3. Stress

4. Urge

5.

Correct Answer: 4.

Question 25: Inversion injury of foot is associated with damage to all the following except

1. 1. Lateral malleolus

2. Base of 5th metatarsal bone

3. Sustentaculum tali

4. Extensor digitorum brevis

5.

Page 216: FMGE (MCI Screening test)  preparation material by Gankidi Raghavender Reddy

Correct Answer: 4.

Question 26: A man aged 60 yrs has h/o IHD and atherosclerosis. He presents with abdominal pain and maroon stools,

likely diagnosis is

1. Acute intestinal obstruction

2. Acute mesenteric ischemia

3. Peritonitis

4. Appendicitis

5.

Correct Answer: 2.

Question 27: An anterolateral corodotomy relieving pain in right leg is effective because it interrupts the:

1. Left dorsal column

2. Left ventral spinothalmic tract

3. Left lateral spinothalmic tract

4. Right lateral spinothalmic tract

5.

Correct Answer: 3.

Question 28: Thickness of stomach in ultrasound is

1. 2mm

2. 5mm

3. 7mm

4. 10mm

5.

Correct Answer: 2.

Question 29: A 27 year old man presents with a left testicular tumor with a 10 cm retroperitoneal lymph node mass. The

treatment of choice is

1. Radiotherapy

2. Immunotherapy with interferon and interleukins

3. Left high inguinal orchiectomy plus chemotherapy

4. Chemotherapy alone

5.

Correct Answer: 3.

Question 30: Sympathectomy is indicated in all the following conditions except

1. Ischaemic ulcers

2. Anhidrosis

3. Intermittent claudication

4. Acrocyanosis

5.

Correct Answer: 2.

Question 31: Most common cause of acute pancreatitis is

Page 217: FMGE (MCI Screening test)  preparation material by Gankidi Raghavender Reddy

1. Gall stone

2. Alcohol

3. Trauma

4. Cyst

5.

Correct Answer: 1.

Question 32: True intracavernous aneurysms present with

1. Ophthalmological problems

2. Headache

3. Nasal intonation of voice

4. Vomiting

5.

Correct Answer: 1.

Question 33: Littre’s hernia includes

1. Meckel’s diverticulum

2. Intestinal margins

3. Umblicus

4. Omentum

5.

Correct Answer: 1.

Question 34: The commonest site for congenital “Arachnoid cyst” is

1. Sylvian fissure

2. Cerebello-pontine angle

3. Cerebello-pontine angle

4. Cerebellar vermis

5.

Correct Answer: 1.

Question 35: A post thyroidectomy patient presents with tingling and perioral paresthesia. Serum calcium level was 7

mg/dL. Which of the following is the best line of management

1. Oral vitamin D3

2. Oral vitamin D3 with calcium

3. I.V.calcium gluconate

4. Observation

5.

Correct Answer: 3.

Question 36: In which of the following conditions is paradoxical respiration observed

1. Stove-in chest

2. Flail chest

3. Pneumothorax

Page 218: FMGE (MCI Screening test)  preparation material by Gankidi Raghavender Reddy

4. Haemopneumothorax

5.

Correct Answer: 2.

Question 37: Which one of the following radio-isotopes is commonly used as a source for external beam radiotherapy in

the treatment of cancer patients

1. Strontium-89

2. Radium-226

3. Cobalt-59

4. Cobalt-60

5.

Correct Answer: 4.

Question 38: Shepherd crook deformity is seen in

1. Osteopetrosis

2. Fibrous dysplasia

3. Rheumatoid arthritis

4. Osteoarthritis

5.

Correct Answer: 2.

Question 39: Fraying and cupping of metaphyses of long bones in an child does not occur in

1. Rickets

2. Lead poisoning

3. Metaphyseal dysplasia

4. Hypophosphatasia

5.

Correct Answer: 2.

Question 40: “CSF Rhinorrhoea” may occur because of all except

1. Trauma

2. Chronically Raised Intracranial pressure

3. Acoustic tumor

4. Pituitary surgery

5.

Correct Answer: 3.

Question 41: Osteomyelitis is common in

1. Frontal sinus

2. Ethmoid sinus

3. Sphenoid sinus

4. Maxillary sinus

5.

Correct Answer: 1.

Page 219: FMGE (MCI Screening test)  preparation material by Gankidi Raghavender Reddy

Question 42: Chemotherapy against tubercular meningitis may be started on the basis of all except

1. Radiological criteria for tubercular meningitis

2. CSF culture positivity for tuberculosis

3. CSF positivity for PCR and ELISA

4. CSF positivity for PCR and ELISA

5.

Correct Answer: 4.

Question 43: Loss of 90% of ileum and jejunum causes all of the following except

1. Anemia

2. Osteomalacia

3. Fluid loss

4. Hypoprotienemias

5.

Correct Answer: 1.

Question 44: During surgical exploration for hydatid cyst of the liver , any of the following agents can be used as scolicidal

agent EXCEPT

1. Hypertonic sodium chloride

2. Formalin

3. Cetrimide

4. Povidone Iodine

5.

Correct Answer: 1.

Question 45: Sciatic nerve palsy may occur in the following injury:

1. Posterior dislocation of hip joint

2. Anterior dislocation of hip

3. Trochanteric fracture

4. Fracture neck of femur

5.

Correct Answer: 1.

Question 46: A patient is having benign prostatic hyperplasia. He is having a blood pressure of 180/110mmHg and a

serum Creatinine of 3.5. Which of the following drugs is not to be used to treat him

1. Prazocin

2. Enlapril

3. Amlodipine

4. Metoprolol

5.

Correct Answer: 2.

Question 47: In a case of blunt trauma, in shock, not responding to IV crystalloids, next step will be

1. Immediate laparotomy

Page 220: FMGE (MCI Screening test)  preparation material by Gankidi Raghavender Reddy

2. Blood transfusion

3. Albumin transfusion

4. Abdominal compression

5.

Correct Answer: 1.

Question 48: The first Laparoscopy was done by

1. Using a vaginal speculum introduced through the fornix

2. Using a cystoscope through a subumbilical incision

3. Using a hollow reed through a subumbilical incision

4. None of the above

5.

Correct Answer: 1.

Question 49: A patient presents with features suggestive of chronic pancreatitis. ERCP examination showed "chain of

Lakes" appearance. Which of the following is the most appropriate management for this patient

1. Side to side pancreatico jejunostomy

2. Distal pancreatic resection and end to side anastomosis

3. Whipple's resection

4. Sphincterotomy

5.

Correct Answer: 1.

Question 50: Most common surgical complication of typhoid is

1. Perforation of ulcer

2. Cholecystitis

3. Hernia

4. Peritonitis

5.

Correct Answer: 1.

Question 51: Trans-cranial Doppler Ultrasonography(TCD) uses all of the following windows except

1. Trans-temporal

2. Trans-orbital

3. Foramen magnum

4. Trans-frontal

5.

Correct Answer: 4.

Question 52: The best treatment for cystic hygroma is

1. Surgical excision

2. Radiotherapy

3. Sclerotherapy

4. Chemotherapy

5.

Page 221: FMGE (MCI Screening test)  preparation material by Gankidi Raghavender Reddy

Correct Answer: 1.

Question 53: Pipe stem appearance in barium enema is seen in

1. Crohns disease

2. Ulcerative colitis

3. Schistosomiasis

4. Carcinoma colon

5.

Correct Answer: 2.

Question 54: Epithelium of Barret’s mucosa is

1. Ciliated columnar

2. Columnar

3. Stratified squamous

4. Squamous

5.

Correct Answer: 2.

Question 55: Diastomatomyelia is due to

1. Gap in the spinal cord

2. Spur in the spinal cord

3. Both of the above

4. None of the above

5.

Correct Answer: 1.

Question 56: Steeple sign is seen in

1. Croup

2. Acute epiglottitis

3. Laryngomalacia

4. Quinsy

5.

Correct Answer: 1.

Question 57: The substances present in the gall bladder stones or the kidney stoens can be best identified by the

following technique

1. Fluorescence spectroscopy

2. Electron microscopy.

3. Nuclear magnetic resonance

4. X-ray diffraction

5.

Correct Answer: 4.

Question 58: Which one of the following emits gamma rays

Page 222: FMGE (MCI Screening test)  preparation material by Gankidi Raghavender Reddy

1. Caesium

2. Strontium

3. Technetium

4. Iodine

5.

Correct Answer: 1.

Question 59: Lunate bone osteoarthritis is called as

1. Kohler’s

2. Keinbock

3. Freeburg

4. Perthes

5.

Correct Answer: 1.

Question 60: “Subclavian Steal Syndrome” indicates reversal of blood flow in

1. Subclavian artery

2. Internal carotid artery

3. Vertebral artery

4. Posterior inferior cerebellar artery

5.

Correct Answer: 3.

Question 61: An edentulous patient has carcinoma of the oral cavity infiltrating into the alveolar margin. Which of the

following would not be indicated in managing the case

1. Segmental mandibulectomy

2. Marginal mandibulectomy with removal of the outer table

3. Marginal mandibulectomy with removal of upper half of mandible

4. Radiotherapy

5.

Correct Answer: 2.

Question 62: CT scan of a patient with history of head injury shows a biconvex hyperdense lesion displacing the grey-

white matter interface. The most likely diagnosis is

1. Subdural hematoma

2. Diffuse axonal injury

3. Extradural hematoma

4. Hemorrhagic contusion

5.

Correct Answer: 4.

Question 63: A patient had an accident and sustained fracture of his femur. 48 hours later, he developed tachypnea,

disorientation and conjunctival petechiae. The diagnosis is most likely to be

1. Hypovolumia

2. Air embolism

Page 223: FMGE (MCI Screening test)  preparation material by Gankidi Raghavender Reddy

3. Fat embolism

4. Hyponatremia

5.

Correct Answer: 3.

Question 64: A posteriorly perforating ulcer in the pyloric antrum of the stomach is most likely to produce initial localized

peritonitis or abscess formation the following

1. Omental bursa [lesser sac].

2. Greater sac.

3. Right subphrenic space

4. Hepatorenal space [ pouch of Morison].

5.

Correct Answer: 1.

Question 65: Bowel does not get strangulated in which of the following

1. Recto uterine pouch

2. Omental bursa

3. Ileo-colic recess

4. Para duodenal and peritoneal recess

5.

Correct Answer: 1.

Question 66: The most common type of esophageal atresia with tracheoesophageal fistula is

1. Side to side

2. Proximal

3. Distal end to trachea

4. Proximal and distal end to trachea

5.

Correct Answer: 3.

Question 67: A patient presents with a pancreatic pseudocyst 5cm in size, 3 weeks duration. What is the best method to

manage this case

1. External drainage

2. USG and follow up

3. Cystogastrostomy

4. Needle aspiration

5.

Correct Answer: 2.

Question 68: All of the following vessels form a part of “circle of willis” except

1. Anterior cerebral artery

2. Middle cerebral artery

3. Posterior cerebral artery

4. Posterior communicating artery

5.

Page 224: FMGE (MCI Screening test)  preparation material by Gankidi Raghavender Reddy

Correct Answer: 2.

Question 69: Ulcerative colitis is characterized by all except:

1. Crypt abcess

2. Submucosal inflammation

3. Pseudopolyps seen

4. Fistula formation

5.

Correct Answer: 4.

Question 70: 20. A 9 month old infant presented with features of intestinal obstruction. On barium enema, the diagnosis

was confirmed to be intussusception. The most likely etiology would be

1. Meckel’s diverticulum

2. Hypertrophic Peyer’s patch

3. Mucosal polyp

4. Lipoma

5.

Correct Answer: 2.

Question 71: The following statement/s about Littre’s hernia is/are true 1. usually occurs in the groin 2. more common in

men than women 3. more common on the right side

1. Only 1 is correct

2. 2 is correct

3. 1 and 2 are correct

4. All the Above

5.

Correct Answer: 4.

Question 72: At the end of a balanced anaesthesia technique with non-depolarizing muscle relaxant, a patient recovered

spontaneously from the effect of muscle relaxant without any reversal. Which is the most probable relaxant the patient had

received

1. Pancuronium

2. Gallamine

3. Atracurium

4. Vecuronium

5.

Correct Answer: 3.

Question 73: Cranio-Vertebral Junction instability may result from all except

1. Rheumatoid arthritis

2. Tuberculosis

3. Trauma

4. Sarcoidosis

5.

Correct Answer: 4.

Page 225: FMGE (MCI Screening test)  preparation material by Gankidi Raghavender Reddy

Question 74: Tamoxifen is used in management of

1. Breast carcinoma

2. Cholecystitis

3. Gastric carcinoma

4. Hernia

5.

Correct Answer: 1.

Question 75: Which of the following is seen in Pindborg’s tumor

1. Sunray appearance

2. Onion peel appearance

3. Driven snow appearance

4. Cherry Blossom appearance

5.

Correct Answer: 3.

Question 76: Seminoma testis is seen in

1. Always in undescended testis

2. Occurs in elderly

3. Occurs in younger age

4. Always bilateral

5.

Correct Answer: 1.

Question 77: In hand surgery which area is called "No man's land"

1. Proximal phalanx

2. Distal phalanx

3. Wrist

4. Between distal palmar crease and proximal phalanx

5.

Correct Answer: 3.

Question 78: What is most characteristic of congenital hypertrophic pyloric stenosis

1. Affects the first born female child

2. The pylori tumour is best feld during feeding

3. The patient is commonly marasmic

4. Loss of appetite occurs early

5.

Correct Answer: 2.

Question 79: Chronic Subdural Hematoma presents

1. 48-72 hours after injury

2. 3 – 20 days after injury

3. more than 3 weeks of injury

Page 226: FMGE (MCI Screening test)  preparation material by Gankidi Raghavender Reddy

4. may present any time after injury

5.

Correct Answer: 3.

Question 80: For breast reconstruction which of the following is not used

1. Transverse rectus abdominus myocutaneous flap

2. Transverse rectus abdominus free flap

3. Pectoralis major myocutaneous flap

4. Lattismus dorsi myocutaneous flap

5.

Correct Answer: 3.

Question 81: All of the following are the sites of portosystemaic shunts except

1. Lower end of esophagus

2. Umbilicus

3. Bare area of liver

4. Anal canal

5.

Correct Answer: 4.

Question 82: Carcinoma prostate commonly occurs in the____ lobe

1. Anterior

2. Posterior

3. Lateral

4. Middle

5.

Correct Answer: 2.

Question 83: Gastric malignancy is predisposed with

1. Duodenal ulcer

2. Gastric hyperplasia

3. Intestinal metaplasia III

4. Blood group O

5.

Correct Answer: 3.

Question 84: Charcot joints include

1. Destruction of bone

2. Dense bone

3. Deformity

4. Debris

5.

Correct Answer: 4.

Question 85: In gastric outlet obstruction in a duodenal ulcer patient, the site of obstruction is most likely to be

Page 227: FMGE (MCI Screening test)  preparation material by Gankidi Raghavender Reddy

1. Antrum

2. Duodenum

3. Pylorus

4. Pyloric canal

5.

Correct Answer: 1.

Question 86: All of the following are risk factors for deep vein thrombosis (DVT) except

1. Duration of Surgery more than thirty minutes

2. Obesity

3. Age less than forty years

4. Use of the oestrogen-projesterone contraceptive pills

5.

Correct Answer: 3.

Question 87: A 6 month old child was found to have grade IV vesicoureteric reflux. The bladder wall is normal. The

treatment of choice would be

1. Ureteric reimplantation

2. Teflon injection to ureteric orifices

3. Antibiotics and observe

4. Endoscopic ureteric resection

5.

Correct Answer: 1.

Question 88: A patient operated for carcinoma colon 4 months back now presents with a 2 cm solitary mass in the liver.

The best line of management is

1. Radiotherapy

2. Radiofrequency ablation

3. Resection

4. CT scan

5.

Correct Answer: 3.

Question 89: In “Communicating hydrocephalus”, the site of CSF obstruction is

1. Aqueduct

2. Foramen of Magendie and Lushka

3. Foramen of Monro

4. Subarachnoid space

5.

Correct Answer: 4.

Question 90: The most common cause of hypoxia during one lung ventilation is:

1. Malposition of the double lumen tube

2. Increased shunt fraction

3. Collapse of one lung

Page 228: FMGE (MCI Screening test)  preparation material by Gankidi Raghavender Reddy

4. Soiling of lung by secretions

5.

Correct Answer: 2.

Question 91: Angiographic demonstration of “puff of smoke” appearance is diagnostic of

1. Polyarteritis Nodosa

2. Moya – Moya disease

3. Takayasu’s arteritis

4. Arterio-Venous Fistula

5.

Correct Answer: 2.

Question 92: Most common complication of acute appendicitis is

1. Wound infection

2. Appendicular mass

3. Subphrenic abscess

4. Perforation

5.

Correct Answer: 2.

Question 93: Which of the following lasers is used for treatment of bening prostatic hyperplasia as well as urinary calculi?

1. CO2 laser

2. Ho: YAG laser

3. Nd: YAG laser

4. Excimer laser

5.

Correct Answer: 2.

Question 94: CT scan may detect air in all of the following conditions except

1. Trauma

2. Post-operative scan

3. Lumbar puncture

4. Pyogenic meningitis

5.

Correct Answer: 4.

Question 95: Which of the following is an absolute indication for surgery in cases of benign prostatic hyperplasia

1. Bilateral hydroureteronephrosis

2. Nocturnal frequency

3. Recurrent urinary tract infection

4. Voiding bladder pressures> 50 cm of water

5.

Correct Answer: 1.

Question 96: In aortic dissection the investigation of choice is

Page 229: FMGE (MCI Screening test)  preparation material by Gankidi Raghavender Reddy

1. ECG

2. CT scan

3. Aortography

4. MRI

5.

Correct Answer: 4.

Question 97: Treatment of duodenal ulcer which has least recurrence is

1. Highly selective vagotomy

2. Vagotomy with antrectomy

3. Vagotomy with gastrojejunostomy

4. Gastrojejnostomy

5.

Correct Answer: 2.

Question 98: All are features of acute peritonitis except

1. Bowel sound present

2. Rigidity

3. Muscle guarding

4. Tenderness

5.

Correct Answer: 1.

Question 99: Symptomatology of metastatic brain abscess includes all except

1. Headache

2. Altered sensorium

3. Hemiparesis

4. Conductive hearing loss

5.

Correct Answer: 4.

Question 100: Strong correlation with colorectal cancer is seen in

1. Peutz-Jegher's polyp

2. Familial polyposis coli

3. Juvenile polyposis

4. Hyperplastic polyp

5.Correct Answer: 2.